Вы находитесь на странице: 1из 107

INTEGRAL DIGEST

1011
Find the minimum value of the area bounded by the curve y =
a + 1 (a > 0), and the x-axis.

e2x ex + 1
(x > 0), the lines x = a, x =
ex 1

1010 2016 The University of Tokyo


Let p, q be real numbers and t be a positive real number.
2 parabolas on a coordinate plane
A : y = x2
B : y = x2 + px + q
touch at the point (1, 1). Let C be a parabola obtained by parallel translation of the parabola B toward the
positive direction of the x and y axis by 2t and t, respectively.
(1) Find the values of p, q.
(2) Let denote S(t) as the area of the region bounded by the parabolas A and C. Find the maximum value of S(t),
letting t vary in a range t > 0.
1009 2016 Kyoto University
In the xyz space consider the figure D given by
|x|

ey + ey
1, 0 y ln a
2

in the plane y = z, where a > 1 is a constant.


Find the volume of the solid generated by rotating the figure D about the y-axis
1008 2016 Tokyo Medical and Dental University
Consider a function
f (x) =<< x >> 2 << x 1 >> + << x 2 >> .
Note that
<< u >>=

u + |u|
2

for a real number u.


(1) Draw the graph of f (x).
(2) Find the maximum value of
g(x) =

1
0

f (x t) dt.

(3) Find the minimum value of


p(s) =

(x s)2 g(x) dx.

1007 2016 Tohoku University


Find the minimum value of the integrated value
I=

(x2 a cos 2x)2 dx.

1006 1991 Hitotsubashi University

In a space, given are fixed points A, B such that |AB| = 4.


Find the volume of the domain in which there exist the points P (x, y, z) satisfying the conditions below simultaneously :

(i) |AP | 4.
1


(ii) 4 AP AB 8.


(iii) AP AB 2 2|AP |.
1005 1969 Kyoritsu College of Pharmacy
Given a parabola y = x2 (a + b)x + ab. Find the relation on a, b for which the area of the region bounded by
the parabola, the x-axis and the y-axis is equal to the half of the area of the region bounded by the parabola and the
x-axis.
Note that : 0 < a < b.
1004 1977 Tokyo University of Science
The cubic function f (x) = x3 + ax2 + bx + c has local maximum at x = and local minimum at x = . If the
graph of f (x) passes through the point (2, 3) and has slope of 9 of the tangent line at the point, f () f () = 4,
then find the values of a, b and c.
1003 1966 Yohohama National University
Find the domain of the points (p, q) satisfying the condition such that a quartic in x : x4 + (1 2q)x2 + p2 + q 2 1
can be decomposed into the product of four linear expressions having real coefficients, also find the area of the domain.
1002
1
1
Suppose two tangent lines l, m can be drawn from the point (0, 1) to the parabola C : y = x2 + . Find the
2
2
volume of the solid generated by rotating the figure bounded by C, l, m about one of the tangents as a rotation axis.
1001 1979 Fukushima Medical University
Z 2
sin x ln sin x dx.
Find lim
a+0

1000
On the coordinate plane, given a quartic function C : y = f (x), cubic functions C1 : y = g(x), C2 : y = h(x) and
a linear function L : y = l(x) such that:
f (x) l(x) = (x )2 (x )2
l(x) g(x) = (x )2 (x )
l(x) h(x) = (x )(x )2
where < 0 < and < 2.
Let s < < t < < u. Denote by S1 , S2 , S3 and S4 each area of each part bounded by C, C2 in [s, ], C1 , C2
in [, t], C1 , C2 in [t, ] and C, C1 in [, u], respectively.
S4
S1
and
.
Find the minimum values of
S2
S3
999

2015 The University of Tokyo

Let n be a positive integer. Answer the following questions.


(1) Define a function g(x) as below.

cos(x) + 1

(|x| 1)

2
g(x) =

0
(|x| > 1)

Let f (x) be a continuous function and let p, q be real numbers.


If p f (x) q holds for x satisfying |x| n1 , then show that :
pn

g(nx)f (x)dx q.

(2) Define a function h(x) as below.

h(x) =

2 sin(x) (|x| 1)

Find the limit

lim n2

(|x| > 1)

h(nx) ln(1 + ex+1 )dx.

998 2015 Ochanomizu Women University


Z 1c
t2n

dt (0 < c < 1, n = 0, 1, ).
Let I2n, c =
1 t2
1+c

(1) Let f (x) be a function satisfying sin(f (x)) = x 2 < x < 2 .
Z x
1

Show that f (x) =


holds on the interval of 1 < x < 1.
1 t2
0
(2) Find lim I0, c .
c+0

1
,
(3) Using the relation tk 1 t2 = tk (1 t2 )
1 t2
Find a2n (n = 0, 1, 2, ) such that lim (I2n+2, c a2n I2n, c ) = 0.
c+0

lim I2n,

(4) For each n, find the value of

c+0

2n Cn

997 2015 University of Miyazaki

For 0 6 , let r() = 2 cos 2. On the coordinate plane, suppose that the circle x2 + y 2 = {r()}2 and the
line y = (tan )x have two points of intersection of which, let P (f (), g()) be the point of intersection with positive

x-coordinate. When ranges in 0 , denote by C the locus of the point P .


6
(1) Find f (), g().
(2) Find the maximum value of g().

(3) Find the area of the part bounded by the curve C, the x-axis and the line x = f
.
6
996 2015 Shizuoka University
Z 2
|et x2 | dt. When x ranges in 0 x e ,find the maximum value and the minimum value, also
Let f (x) =
0

the values of x giving the maximum and minimum.


995 2015 Doshisya University
Let n be a positive integer. Given
In =

xn cos( ln x) dx, Jn =

Find

xn sin( ln x) dx.

lim

Jn+1
Jn
In+1
, lim
and lim
.
n In
In n Jn

994 2015 Ryukyu University


Find the functions f (x), g(x) such that
Z

f
(x)
+

x
0

g(t) dt = 2 sin x 3


f (x)g(x) = cos2 x

993 2015 Nagoya City University

x+
4



|2 cos2 t + 2 sin t cos t 1| dt 0 x
Let f (x) =
.
2
 x 
.
(1) Find f
2
(2) Find f (x).
(3) Find the maximum and the minimum value of f (x), also the values of x giving the values.
992 2015 Meijo University
Define a sequence {an } by an = 2n+1 3 (n = 1, 2, 3, ).
Find
Z an+1
{ln(x + 3) n ln 2} dx (n = 1, 2, 3, ).
In =
an

991 2015 Shibaura Institute of Technology






Let k be a positive constant. Given the curves C1 : y = k sin 2x 0 x
, C2 : y = sin x 0 x
.
2
2
Answer the following questions.
(1) Find the area S1 of the part bounded by the curve C1 and the x-axis.
(2) Find the range of k such that two curves C1 , C2 intersect other than the origin. When k satisfies the range, if
the x-coordinate other than the origin is , then express cos in terms of k.
(3) Let k be the values found in (2). Denote by S2 the area of the part bounded by C1 , C2 . Find the value of k
such that S1 = 3S2 .
990 2015 Osaka University
1
The line l : y = kx + m (k > 0) touches the both the circle C1 : x2 + (y 1)2 = 1 and the parabola C2 : y = x2 .
2
(1) Find the values of k and m.
(2) Find the area of the figure bounded by the line l, the parabola C2 and the y-axis.
989 2015 Kyoto University
Draw the domain of the points (p, q) such that a line y = px + q intersects with the graph of y = x2 x , but
dont intersect with the graph of y = |x| + |x 1| + 1, also find the area of the domain.
988 2015 Tohoku University
Let n be a positive integer and let
In =

(1) Show that for a real number t 0,

dx

, Jn =
1 + xn

ln( 1 + xn + 1) dx.

1+t+1
t
ln
.

2
2( 1 + t + 1)

(2) Show that


0 Jn ln 2

1
.
4(n + 1)

d
ln( 1 + xn + 1)dx.
dx
(4) Find lim n(1 In ).

(3) Find

987 2015 Kyusyu University


Prove that
n
X

k=3

1
1
<
.
k(ln k)2
ln 2

986 2015 Toyama University

Let f (x) = ln x (x > 0) and let l be the tangent line at the point (t, f (t))
on the curve C1 : y = f (x). Let S be
the area of the figure bounded by the line l and the curve C2 : y = (x 2)2 . Find the value of t for which S is
minimized.
985
Evaluate
Z

1
(1 +

2016
x2015 ) 2015

dx.

984 2015 Kanazawa University

1
cos x sinn1 x dx = n .
n2
0

(2) Let n be a positive integer. In the interval of 0 x ,


6
Show that
(1) Show that for a positive integer n,

X
1
.
(3) Find
n
n2
n=1

1
1
(1 + sin x + sin2 x + + sinn x) n .
1 sin x
2

983 2015 Waseda University


Let D be the part bounded by a parabola y = x2 + 2x + 2 and the x-axis.
(1) Find the volume of the solid generated by rotating D about the x-axis.
(2) Find the volume of the solid generated by rotating D about the y-axis.
982

2015 Osaka University


x
x
For a positive integer n, define a function by fn (x) =
(x 0).
ln 1 +
n(1 + x)
n
Z n
Z 1
(1) Show that
fn (x) dx
ln(1 + x) dx.
0
0
Z n
fn (x) dx. Using the fact ln(1 + x) ln 2 for x 0, show that the sequence
(2) Define a sequence {In } by In =
0

{In } converges, then find the limit value.


981 2015 Kochi University

Find the area of the domain of the points (x, y) satisfying 3|1x| (1 + |y|) 3.
980 2015 Hirosaki University
Let n be a positive integer.
(1) Find the maximum value of a function f (x) = xn+1 ex in x 0.
(2) Find lim xn ex .
x
Z x
(3) Show that for each positive integer n, lim
tn et dt = n!.
x

979 2015 Hokkaido University


Z e
Consider a function f (x) =
tx1 ln t dt (x > 0). Find the local minimum of a function g(x) = x2 f (x) x2 in
1

x > 0, also find the value of x giving the value.

978 2015 Kanazawa Institute of Technology


Let n be a positive integer. Find the limits below.
Z
Z n
2x
e
cos 5x dx, lim
lim
n

e2x sin 5x dx.

977 2015 Osaka Prefectural University


Z
Let m be an integer. Compute I = xm ln x dx.
976 2015 Hiroshima University
On the coordinate plane, let C1 be a circle passing through the origin O centered on P (1, 1). For a positive
k
constant k, let the curve C2 : y = (x > 0). Suppose that C1 and C2 intersect at two points Q, R and the line P Q
x
is paralell to the x axis. Let q, r be the x coordinates of Q, R, respectively.
(1) Find the values of k, q and r.
(2) Find the area
Z q pof the part bounded by the curve C2 , line segments OQ and OR.

(3) Evaluate
2 (x 1)2 dx by setting x = 1 + 2 sin .
r

(4) Find the volume V generated by rotating the figure enclosed by arc QR not including the origin O of the circle
C1 and the curve C2 about the x-axis.
975

2015 Mie University

Find the area of the figure bounded by the curve y = e

x1

x (x 0) , the x-axis and y-axis.

974 2015 Tokyo University


Let f (x) = cos x + sin x 1 and let g(x) is a continuous function such that
Z 2

1
g(x) = |f (x)| 2
tg(t) dt 3 .
4
0
(1) Find the range of x in the interval 0 x 2 such that f (x) > 0.
Z 2
(2) Evaluate
t|f (t)| dt.
(3) Find g(x).

973 2015 Chiba University


For non-negative integer n, define a polynomial Tn (x) by
T0 (x) = 1, T1 (x) = x, Tn (x) = 2xTn1 (x) Tn2 (x) (n = 2, 3, 4, ).
(1) Show that for each non-negative integer n, we have cos n = Tn (cos ).
Z 1
(2) Evaluate
Tn (x) dx.
1

972 2015 Daido University


Z 2
Z 2
sin x
cos x
Evaluate I =
dx, J =
dx.
3
sin
x
+
cos
x
3
sin
x + cos x
0
0
971 2015 Sinsyu University

Z x

1

2
cos 2(x t)
Let f (x) =
dt 0 x
.
4
2
0
Find the maximum and minimum value of f (x).
970 2015 Kumamoto University
Let r be a positive real number. Define a sequence {an } by
Z n
an =
erx | sin x|dx (n = 1, 2, 3, ).
0

Let f (r) = lim an . Find lim rf (r).


n

r+0

969 2015 Toyama University


p
Prove that n!(n 1)! < nn en+1 (n = 2, 3, ).
6

968 2015 Oita University


Find the area of the part enclosed by the figure expressed by the equation y 2 = x6 (1 x2 ).
967 2015 Tottori University
Let f (x) be a continuous function such that f (x) = 1 +
Answer the questions below.
(1) Let (x) = f (x) + f (x). Find

(x t)f (t) dt.

(x)
.
(x)

(2) Find f (x).


966 2015 Mie University
Prove that

e2

1
1
1
3

dx e4 e2 + .
8
8
4
ln x

965 2015 Kyoto Institute of Technology


1 p
n
(n2 + 12 )(n2 + 22 ) (n2 + n2 ).
n n2

Find the limit lim

964 2015 Yokohama National University

Suppose a, b and c in this order beZ the three outcomes when a fair die is thrown three times.

(cos ax)(cos bx)(cos cx) dx > 0.


What is the probability of getting
0

963 2015 The University of Miyazaki


Evaluate
Z the integrals below.
(1)
| sin x cos x| dx.
Z 012 3
x + 2x2 3
(2)
dx.
x2 1
0
!
r
Z 1
3
1

dx.
+
(3)
4 3x2
4 x2
0
Z 2
(4)
x3 ln x dx.
1

962 2015 Kyoto University

Find the volume of the solid generated by a rotation of the region bounded by the part of 0 x of the curves
2



y = sin x +
and y = sin 2x about the x-axis, where the lines x = 0, x = arent be considered as the lines that
8
2
enclose the region.
961
960 2015 Tokyo Institute of Technology
4an 9
(n = 1, 2, 3, ) and
an 2
a1 + 2a2 + + nan
a sequence {bn } as bn =
(n = 1, 2, 3, ).
1 + 2 + + n
(1) Find the general term {an }.
4
holds.
(2) Prove that for each n the inequality bn 3 +
n+1
(3) Find the limit value lim bn .
Define a sequence {an } as a1 = 5, an+1 =

959 2015 The University of Tokyo for Medicine


Let a be a positive real number and let p be a positive rational number.

Consider two curves y = axp (x > 0) and y = ln x (x > 0). Suppose the curves have just one common point Q of
intersection.
xp
Answer the questions as below. If necessary, you may use lim
= without proving.
x ln x
(1) Express a and the x-coordinate of the point Q in terms of p.
(2) Express the volume of the solid generated by revolving the figure enclosed by the two curves and the x-axis in
terms of p.
(3) Find the value of p such that the volume of the solid obtained in (2) is 2.
958 2015 The University of Tokyo
Consider two points A(1, 1), B(1, 1) on the coordinate plane. Let P be a point on the coordinate plane such
that the absolute value of the x-coordinate
of P is less than or equal to 1. Draw the domain of the points P satisfying the condition (i) or (ii) as below, then
find the area.
(i) There exists a parabola having the absolute value of the x-coordinate of the vertex which is more than or equal
to 1 and passes through all points of A, P, B.
(ii) The points A, P, B are collinear.
957 2015 Waseda University
For real numbers a, b let f (x) = x2 + ax + 1, g(x) = x2 bx + 1.
Answer the questions as below.
(1) Find the condition in a, b such that the equations f (x) = 0 and g(x) = 0 have some common root.
(2) Let a, b move satisfying the condition found in (1), a 0, b 0. Let be the common root of f (x) = 0 and
g(x) = 0. Denote by l the tangent line drawn at the point (, 0) on the graph of y = f (x). Find the minimum area
of the part bounded by the graph of y = g(x) and l.
956 2015 Waseda University

In the xy-plane, the points A( 2a, 0), B( 2a, 0) (a > 0) are fixed. A mobile point P (x, y) satisfies the
condition AP + BP = 4a.
Answer the questions as below.
(1) Find the equation of the points P (x, y) as the locus.

(2) Find the volume V ofthe solid generated


by revolving the figure enclosed by the part 2a x 2a of the

curve in (1), the lines x = 2a, x = 2a about the x-axis.


(3) Find the surface area of the solid in (2).
955 2015 Japan Womens University
In the coordinate plane, find the area of the region bounded by the parabolas C1 : y = 4x2 +12x+2, C2 : y = x2 +2
and the common tangent line with a positive slope touching both of C1 , C2 .
954 2015 Keio University
Find the area of the figure bounded by the curve C : y = f (x) := (x 1)|x 3| 4x + 12 and the tangent line l
drawn at P (1, f (1)).
953 2015 Keio University
For a real number a, let f (x) be a quadratic function satisfying, for all real numbers x,
Z
x
4
f (t) dt = xf(x) x3 + ax2 and f (0) = a2 a 6.
3
0
Find the possible value of a such that the quadratic equation f (x) = 0 has at least one positive real solution.
952 2015 Showa University School of Medicine
n
X
n + 3k
.
n
n2 + k 2

Find lim

k=1

951 2015 Tokyo Womens Medical University


Z ln 2 5x
e
dx.
Evaluate
x
e +1
0
950 2014 National Defense Medical College

Z
(1)k x
x sin x
2
Define ln fk (x) =
and gk (x) =
fk (x),
fk1 (t) dt (k = 2, 3, 4, ), f1 (x) =
x
2
1
+
e
4

cos2 x
0
Z
Z
2n+1
XZ
sin x
x sin x
dx, K =
dx.
In =
gk (x) dx, J =
2x
4

cos
4

cos2 x
0
0
k=1
(1) Express fk (x) without using integration.
(2) Express In in terms of J.
(3) Express J in terms of K.
(4) Find In .
949
Evaluate, for a > b > 0,
Z

ab

948

x3
p

dx.
( a x2 + b + x2 )2 (a x2 )(b + x2 )

Find the continuous function {fn (x)}n1 defined by f1 (x) = cos x,


fn+1 (x) =

cos x(sin x sin t)fn (t) dt (n = 1, 2, 3, ).

Note: cos x(sin x sin t)fn (t) expresses the product of cos x, sin x sin t and fn (t).
947 1987 Tokyo University of Science
In the xyz-space, let K be the solid figure enclosed by the surface of revolution Q formed by rotating the curve
z2
= 1 on the yz-plane around the z-axis and two planes z = y + 1, z = y 1.
y
2
(1) Find the area S(t) of the section of K cut by the plane : z = y + t (1 t 1).
(2) Find the volume of K.
2

946 1987 Nagoya University


Let C be the part of thefirst quadrant of a circle with radius 1 centered on the origin O. Consider a mobile point

, denote by S1 () the area of the sector bounded by line segment OP , the x-axis and the
P (cos , sin ) 0 < <
2
curve C, and denote by S2 () the area of the part bounded by the line passing through the point P and touching C,

the y-axis and the curve C. Let S() be the smaller value among S1 (), S2 (). Note : When = 0 or = , then we
2
Z 2
S() sin d.
regard S() as 0. Evaluate
0

945 1985 Nagoya University


Given a circle C with radius 1 which touches externally a circle C with radius 2. Let A be a fixed point on C ,
suppose that at first the centers of C, C and A are collinear in this order. When C travels around C at once without
slipping and touching C, find the length of the curve on which A traces.
944 1981 Nagoya University
If a differentiable function f (x) satisfies that for x 0 always f (x) > 0,
for x > 0.

f (t) dt x, then prove that f (x) > 1

943
Find lim n2
n

enx sin x dx (n = 1, 2, ).

942 1990 Osaka University


The sphere O2 with radius r2 centered on B(1, 0, 0) is contained inside the sphere O1 with radius r1 centered on
A(1, 0, 0). Let S be the curved surface generated by all of centers P of the spheres which touches the sphere O1
internally and the sphere O2 externally.
(1) Find the equation, on the xy-plane, of the section C of the curved surface S cut by xy-plane.
9

(2) Find the volume V of the part enclosed by the curved surface S.
941
Evaluate

cos x

dx.
(sin x + 3 cos x)3

940 2014 Tokyo Institute of Technology

When the point P (t, s) moves on the x-y plane with constrain s = 2t2 2t, denote by C the curve of the locus
of the point Q obtained by a rotation of 45 anticlockwise for the point P about the origin and denote by D the figure
bounded by the curve C and the x-axis.
(1) Express the coordinate of Q(x, y) in terms of t.
(2) Find the value of a such that the line y = a has just one point of intersection with the curve C.
(3) Find the volume of V of the solid generated by a rotation of the figure D about the y-axis.
939
Evaluate

1
dt.
4 + sin t

938
Evaluate

1
dt.
sin t + tan t

937 2014 Fukushima Medical University


1 a cos x
(0 x ), answer the following questions:
1 + sin x
dx
cos x
(0 < x < ). Express
in terms of t.
(1) Let t =
1 + sin x
dt
(2) Determine the range of a such that f (x) has extreme value in the range of 0 < x < , then express the extreme
value in terms of a.
(3) When a lies in the range determined in (2), find the volume of the solid generated by a rotation of the figure
bounded by the line passing through the points (0, f (0)), (, f ()) and the graph of y = f (x) in terms of a.
Let a be a constant. For a function f (x) =

936 2014 Shinshu University


A function f (x) satisfies f (x) < 0. For t 0, show that the following inequalities hold:
(1) f (0) + f (t)t f (t) f (0) + f (0)t.
Z t
f (0)t + f (t)t
f (0) 2
(2)

f (u) du f (0)t +
t .
2
2
0
935 2014 Osaka City University
For a constant > 0, take the points O(0, 0), A(, ln ( + 1))) on the curve C : y = ln (x + 1). Consider a
moving point P (t, ln (t + 1)) with 0 < t < on the curve C. Denote by S1 the area of the part bounded by the curve
C and the line OP , and by S2 that of the part bounded by the curve C and the line P A.
x
(1) Show that for x > 0,
< ln (x + 1) < x holds.
x+1
(2) Express S1 , S2 in terms of , t.
(3) Find t such that S1 + S2 is minimized.
934 2014 Hitotsubashi University
Z 1
For an integer n 0, let an =
xn ex dx.

(1) Find a1 , a2 , a3 and a4 .


30
8
.
(2) Show that < e <
3
11

933 2014 Tokyo University


For a positive constant p, let f (x) =

{p ln(1 + |t|)} dt.

10

(1) Find the extreme value of f (x).


(2) Find the range of p such that the curve y = f (x) has two point of intersections with the positive part of the x
axis.
932 2014 Hokkaido University
Z x+ 3
Let f (x) =
| sin | d.
x

(1) Find f (x).


(2) Find the maximum and minimum value of f (x) on 0 x .
931 2014 Osaka City University

When a, b move on all of real numbers, find the minimum value of I(a, b) =

(x a b cos x)2 dx.

930 2014 Yokohama City University


Let a be a positive real number. Take two points 0(0, 0), A(x1 , y1 ) (y1 > 0) on the parabola y 2 = 4ax.

1
(1) For > 0, let F (t) = {t t2 + + ln(t + t2 + )}.
2
Find F (t).
(2) Express the length of the curve L from the point O to the point A as the function of x1 .
929 2014 National Defense Academy


Z ax p
1
2
.
For a > 1, let f (x) =
1 t dt 0 x
a
  Zx 1 p
a
1
(1) Evaluate f
1 t2 dt.
+
a
0

3
.
(2) Find the value of M such that f (x) has maximum value M at x =
2a
928 2014 Shibaura Institute of Technology


t2
x2
Let l be a tangent line at the point T t,
. A circle S touches the curve C
(t > 0) on the curve C : y =
2
2
Z 1
and the x axis. Denote by (x(t), y(t)) the center of the circle S. Find lim
x(t)y(t) dt.
r+0

927 2014 Nagoya Institute of Technology


Z ln 3
n
X
Evaluate
x lim
kekx dx.
n

ln 2

k=1

926 2014 Setsunan Universuty


Z 6
Z
3x
Evaluate I =
e
sin 4x dx, J =
0

e3x cos 4x dx.

925 2014 Miyazaki University


Evaluate
integrals as follows.
Z 2 xthe definite
e + ex
dx.
(1)
ex ex
Z1 6
sin 3x sin 5x dx.
(2)
Z0 1
x3 + 3x2
dx.
(3)
x2 + 3x + 2
Z0 2
3
(4)
x5 ex dx.
1

924 2014 The University of Tokyo

11

Let u be a real number. On the coordinate plane, consider two parabolas C1 : y = x2 + 1, C2 : y = (x u)2 + u.
The range of u such that C1 and C2 have at least one point of intersection is expressed by a u b for some real
numbers a, b.
(1) Find the values of a, b.
(2) When u satisfies a u b, let P1 (x1 , y1 ), P2 (x2 , y2 ) be the point of intersections of C1 and C2 .
Note : if the point of intersection is just one, then we regard the intersection as P1 = P2 .
Express 2|x1 y2 x2 y1 | in terms of u.
Z b
f (u) du.
(3) Let f (u) be the expression obtained in (2), evaluate I =
a

923 2014 Yokohama National University


Let a be a positive real numbers. Let t, u (t < u) be the x coordinates of the point of intersections of the curves :
C1 : y = | cos x| (0 x ), C2 : y = a sin x (0 x ). Denote by S1 the area of the part bounded by C1 , C2 and
y-axis in 0 x t, and by S2 the area of the part bounded by C1 , C2 in t x u. When a moves over all positive
real numbers, find the minimum value of S1 + S2 .
922 2014 Aichi University
Find the value of t such that S(t) =

|x t|

ln x
dx (1 t e) is minimized.
x

921 2014 Kobe University



1

Let a be the x coordinate of the point of intersection of the curves y = f (x) =
.
, y = g(x) = 2 tan x < x <
cos x
2
2
Z a
xf (x)g(x) dx.
Evaluate
0

920 2014 Osaka University


Find the integer part of

40000
X
n=1

1
.
n

919
Find

Z
X

n=1

sin x sin nx dx (n = 1, 2, ).

918 2014 Chiba University


For m, n be non-negative integers, let
(1) For n 2, express In,

cosn sinm d.

in terms of In2, m+2 .


Z
1 1 n
(2) Show that I2n+1, 2m+1 =
x (1 x)m dx.
2 0
C0m
C1
Cm
n!m!
m
=
m + + (1)m
.
(3) Show that
(n + m + 1)!
n+1 n+2
n+m+1
m

917 2014 Tohoku University

1 

Consider a line l : 2x 3y = 0 and a parameterized curve C : x = tan t, y =
0t<
.
cos t
2
Find the area of the part bounded by l, C and the y-axis.
916 2014 Fukui University
Z 2
sin 3x
dx.
Evaluate
sin x + cos x
0
915
Find the area of the region bounded by a function y = x4 + 16x3 78x2 + 50x 2 and the tangent line which is
tangent to the curve at exactly two distinct points.
914 2014 Nagoya University
12

In a space, let B be a sphere (including the inside)


with radius of 1. Line intersects with B, the length of the
common part is the line segment with the length of 3.
(1) Find the distance between the center of B and .
(2) Find the volume of the solid generated by rotating B about .
913 2014 Tohoku University
Z 2
cos(2n + 1)x
dx.
For integer n, let In =

sin x
4
(1) Find I0 .
(2) For each positive integer n, find In In1 .
(3) Find I5 .
912
Which is larger,
Z

2014

x
0

2014

sin

x dx

2013

2013

sin

x dx

or Z

x2011 sin2011 x dx
?
2012

2012

sin

x dx

911 2014 Tohoku University


Let 1 , 2 be the tangent lines at P (a, a2 ), Q(b, b2 ) (a < b) on the curve C : y = x2 respectively. Let R be the point
of intersection of 1 , 2 . Denote by S the area of the figure enclosed by the segments P R, QR and the curve C. If 1
is perpendicular to 2 , then find the minimum value of S.
910 2014 Nagoya University
For a real number t, consider two points P (t, t2 ), Q(t + 1, (t + 1)2 ). When t moves in the range of 1 t 0,
draw the figure swept out by the segment P Q, then find the area of the figure.
909 2014 Tokyo Institute of Technology
In the xy-plane, consider the curve C : y = x3 + x2 + 1, let P0 (1, 3). For k = 1, 2, 3, , let Pk (xk , yk ) is the
point of intersection other than the point Pk1 between the tangent line of C at the point Pk1 (xk1 , yk1 ) and C.
Denote by Sk the area of the part enclosed by the segment Pk1 Pk and C.
(1) Find S1 .
(2) Express xk in terms of k.

X
1
(3) Find
.
Sk
k=1

908 2014 Kyoto University


1
, a circle centered on the the origin, respectively.
x

Let C1 , C2 intersect at the points A, B and let the angle of the tangent line at A on C1 and the segment OA is .
6
Find the area of the figure enclosed by C1 , C2 .
Let C1 , C2 be parts of the first quadrant of the parabola y =

907 2014 Hitotsubashi University


Let be the tangent line at the point (t, t2 ) (0 < t < 1) on the parabola C : y = x2 . Denote by S1 the area of the
part enclosed by C, and the x-axis, denote by S2 of the area of the part enclosed by C, and the line x = 1. Find
the minimum value of S1 + S2 .
906 2014 Waseda University
For a > 1,
(1) Prove that:
n1
X

k=0

(2) Prove that:

k+1
n

k
n

1
a

k+1
n

<

13

n1
 1
X  k+1
k
dx
<
a n an
k .
x
an
k=0

lim

n1
X

k=0

k+1
n

an

1
a

k+1
n

n1

 1
X  k+1
k
1
a n an
dx = lim
k .
n
x
an
k=0

905 2014 Tokyo University


Z n+ 1
X
4
e4x cos 2x dx.
Find
n=1

n 41

904 2014 Ritsumeikan University


For f (t) = t cos t, g(t) = t sin t, find the maximum value of

and evaluate

f (t)g (t) dt.

f (t)f (t) + g (t)g (t)


p
p
f (t)2 + g (t)2 f (t)2 + g (t)2

903 2014 Showa University

Let {an }n1 be a sequence defined by an =

x2 (1 x)n dx. Find the real value of c such that

(n + c)(an an+1 ) = 2.

n=1

902 2014 Sophia University


For a 0, find the minimum value of S(a) =

|x2 + 2ax + a2 1| dx.

901 2014 Tokyo Womens Medical University


d
Given the polynomials P (x) = px4 + qx3 + rx2 + sx + t, Q(x) =
P (x), find the real numbers p, q, r, s, t such
dx
Z 1

52
P (x)dx = .
that P ( 5) = 0, Q( 2) = 0 and
5
0
900 2014 Kwansei Gakuin University
Find

n
X
(1)k k
C .
2k + 1 n
k=0

899 1969 Shibaura Institute of Technology


Find the limit as below:
(12 + 22 + + n2 )(13 + 23 + + n3 )(14 + 24 + + n4 )
n
(15 + 25 + + n5 )2
lim

898

Let a, b be positive constants. Evaluate

1
0

(x + a)x+a
(x + b)x+b
dx.
(x + a)(x + b) ln(x + a) ln(x + b)
ln

897 2013 Tsuda College


Find the volume V of the solid formed by a rotation of the region enclosed by the curve y = 2x 1 and two lines
x = 0, y = 1 around the y axis.
896 2013 Tokyo University of Science

14

1
1p
Given sequences an = n An2n , bn = 2
n
n

q
n
A2n
4n and cn =

s
n

A4n
8n
, find lim an , lim bn and lim cn .
n
n
n
A4n
6n

895 2013 Yokohama City University


p
In the coordinate plane, suppose that the parabola C : y = x2 + q (p > 0, q > 0) touches the circle with radius
2
1 centered on the origin at distinct two points. Find the minimum area of the figure enclosed by the part of y 0 of
C and the x-axis.
894 2013 Chiba University
Let a be non zero real number. Find the area of the figure enclosed by the line y = ax, the curve y = x ln(x + 1).
893 2013 Ochanomizu University
Z 4
Find the minimum value of f (x) =
| tan t x| dt.
0

892 2012 Kobe University


Z 2
sin x cos x
dx.
Evaluate
1 + cos x
0
891 2013 Osaka University
Given a triangle OAB with the vetices O(0, 0, 0), A(1, 0, 0), B(1, 1, 0) in the xyz space. Let V be the cone
obtained by rotating the triangle around the x-axis. Find the volume of the solid obtained by rotating the cone V
around the y-axis.
890 2013 Osaka Prefecture University
A function fn (x) (n = 1, 2, ) is defined by f1 (x) = x and
Z
e 1
fn1 (t)ext dt (n = 2, 3, ).
fn (x) = x +
2 0
Find fn (x).
889 2013 Okayama University



1
Find the area S of the region enclosed by the curve y = x (x > 0) and the line y = 2.
x
888 2013 Meijyo University

In the coordinate plane, given a circle K : x2 + y 2 = 1, C : y = x2 2. Let be the tangent line of K at


P (cos , sin ) ( < < 2). Find the minimum area of the part enclosed by and C.
887 2013 Shibaura Institute of Technology
Z x
1
dt, answer the questions as follows:
For the function f (x) =
1
+
t2
0

(1) Find f ( 3).


Z 3
(2) Find
xf (x) dx.
0
 
1
is constant, then find the value.
(3) Prove that for x > 0, f (x) + f
x
Z
Note: Please solve the problems without using directly the formula

School students those who dont study arcsin x, arccos x, arctan x.


886 2013 Nagoya City University
Find the functionsZ f (x), g(x) such that

f (x) = ex sin x +
ug(u) du

and

15

1
dx = tan1 x + C for Japanese High
1 + x2

g(x) = ex cos x +

uf (u) du.

885
Find the infinite integrals as follows
(1)
Hiroshima City University
Z 2013
x2
dx.
2 x2
(2) 2013 Kanseigakuin University
Z
x4 ln x dx.

(3)
Shinsyu University
Z 2013
cos3 x
dx.
sin2 x

884 2013 Shinsyu University


Prove that :
(e 1) <

e| cos 4x| dx < 2(e 2 1).

883 2013 Fukushima University


Prove that for each positive integer n
Z
4n2 + 1 x
(2n + 1)2
e e 2
x
ln
.
(e

e
)
cos
2nx
dx
>
4n2 1 0
4
(2n 1)(n + 3)
882 2013 Aichi University
Find lim

n
X

k=1

1
(ln(n + k) ln n).
n+k

881 2013 Kagoshima University


!2
Z 2013
X
sin kx
dx.
Evaluate

k=1

880 2013 Aichi Prefectural University



sin2 at + t2
sin2 at t2 
0 < |t| <
and
, g(t) =
at sin at
2a

at sin at
4
2
let C : x2 y 2 = 2 x
. Answer the questions as follows:
a
a
(1) Show that the point (f (t), g(t)) lies on the curveC.

(2) Find the normal line of the curve C at the point lim f (t), lim g(t) .
For a > 2, let f (t) =

t0

t0

(3) Let V (a) be the volume of the solid generated by a rotation of the part enclosed by the curve C, the nornal
line found in (2) and the x-axis. Express V (a) in terms of a, then find lim V (a).
a

879 2013 Hiroshima City University


Evaluate
the integrals as follows:
Z
x2
dx.
(1)
2x
Z p
3
(2)
x5 + x3 dx.
Z 1
(3)
(1 x) cos x dx.
0

878 2013 National Defense Academy of Japan


A cubic function f (x) satisfies the equation sin 3t = f (sin t) for all real numbers t.

16

Evaluate

f (x)2

p
1 x2 dx.

877 2005 Tokyo Gakugei University

sinn+2 x + cosn+2 x
for 0 x .
Let f (x) = lim
n
n
n
sin x + cos x
2
Z 2
Evaluate
f (x) dx.
0

876
Suppose a function f (x) is continuous on [1, 1] and satisfies the condition :
1) f (1) f (1).
2) x + f (x) is non decreasing function.
Z 1
3)
f (x) dx = 0.
1
Z 1
2
Show that
f (x)2 dx .
3
1
875
Evaluate

x2 + x + 1
dx.
x4 + x3 + x2 + x + 1

874 2013 Hitotsubashi University


Given a parabola C : y = 1 x2 in xy-palne with the origin O. Take two points P (p, 1 p2), Q(q, 1 q 2 ) (p < q)
on C.
(1) Express the area S of the part enclosed by two segments OP, OQ and the parabalola C in terms of p, q.
(2) If q = p + 1, then find the minimum value of S.
(3) If pq = 1, then find the minimum value of S.
873 2013 Tokyo Institute of Technology
Let a, b be positive real numbers. Consider the circle C1 : (x a)2 + y 2 = a2 and the ellipse C2 : x2 +

y2
= 1.
b2

(1) Find the condition for which C1 is inscribed in C2 .


1
(2) Suppose b = and C1 is inscribed in C2 . Find the coordinate (p, q) of the point of tangency in the first
3
quadrant for C1 and C2 .
(3) Under the condition in (1), find the area of the part enclosed by C1 , C2 for x p.
872 2013 Tsukuba University
Let n be a positive integer.
(1) For a positive integer k such that 1 k n, Show that :
Z

k
2n

k+1

sin 2nt cos t dt = (1)


k1
2n

2n
4n2 1



k
k1
cos + cos
.
2n
2n

(2) Find the area Sn of the part expressed


by a parameterized
curve Cn : x = sin t, y = sin 2nt (0 t ).

If necessary, you may use

n1
X

cos

k=1

(3) Find lim Sn .


n

1
k
=
2n
2

tan
4n

1
(n 2).

871 2013 Tohoku University


Define sequences {an }, {bn } by
an =

en sin d, bn =

en sin cos d (n = 1, 2, 3, ).

17

(1) Find bn .

2
(2) Prove that for each n, bn an bn .
3
1
(3) Find lim
ln(nan ).
n n
870 2013 Hokkaido University

Consider the ellipse E : 3x2 + y 2 = 3 and the hyperbola H : xy =

3
.
4

(1) Find all points of intersection of E and H.


(2) Find the area of the region expressed by the system of inequality

3x2 + y 2 3
3
xy .
4
869 2013 Showa University
Z
1
Let In =
x(sin nx + n cos nx) dx (n = 1, 2, ).
n+1 0
Answer the questions below.
(1) Find In .

X
(2) Find
In .
n=1

868 2013 The University of Tokyo


In the coordinate space, define a square S, defined by the inequality |x| 1, |y| 1 on the xy-plane, with four
vertices A(1, 1, 0), B(1, 1, 0), C(1, 1, 0), D(1, 1, 0). Let V1 be the solid by a rotation of the square S about
the line BD as the axis of rotation, and let V2 be the solid by a rotation of the square S about the line AC as the axis
of rotation.
(1) For a real number t such that 0 t < 1, find the area of cross section of V1 cut by the plane x = t.
(2) Find the volume of the common part of V1 and V2 .
867 2013 Waseda University
Z 2
f (x) dx for any quadratic functions f (x) in terms of f (0), f (1) and f (2).
Express
0

866 2013 Waseda University


Given a solid R contained in a semi cylinder with the hight 1 which has a semicircle with radius 1 as the base.
The cross section at the hight x (0 x 1) is the form combined with two right-angled triangles as attached figure
as below. Answer the following questions.
(1) Find the cross-sectional area S(x) at the hight x.
(2) Find the volume of R. If necessary, when you integrate, set x = sin t.

bc
bc

E
bc

bc

bc

bc

bc

bc

865 2013 Ritsumeikan University


Find the volume of the solid generated by a rotation of the region enclosed by the curve y = x3 x and the line
y = x about the line y = x as the axis of rotation.
18

864 2013 Nippon Medical School


Let m, n be positive integer such that 2 m < n.
(1) Prove the inequality as follows.
1
1
1
n+1m
1
n+1m
+ +
+ 2 <
< 2+
.
m(n + 1)
m
(m + 1)2
(n 1)2
n
n(m 1)

(2) Prove the inequality as follows.



3
1
1
lim 1 + 2 + + 2 2.
2 n
2
n
(3) Prove the inequality which is made precisely in comparison with the inequality in (2) as follows.


29
61
1
1
lim 1 + 2 + + 2
.
18 n
2
n
36
863 1981 Tokyo Institute of Technology
Z
1 2t
For 0 < t 1, let F (t) =
| cos 2x| dx.
t 0
(1) Find lim F (t).
t0

(2) Find the range of t such that F (t) 1.


862 1969 Hitotsubashi University
Draw a tangent with positive slope to a parabola y = x2 + 1. Find the x-coordinate such that the area of the figure
11
bounded by the parabola, the tangent and the coordinate axisis is
.
3
861 2013 Showa University
Answer the questions as below:
2
(1) Find the local minimum of y = x(1 x2 )ex .
(2) Find the total area of the part bounded the graph of the function in (1) and the x-axis.
860 2013 Kyorin University
e

f (t)
dt, answer the questions as below:
t
1
(a) Find f (x) and the y-coordinate of the inflection point of the curve y = f (x).
(b) Find the area of the figure bounded by the tangent line of y = f (x) at the point (e, f (e)), the curve y = f (x)
and the line x = 1.
2

For a function f (x) (x 1) satisfying f (x) = (ln x)

859
2

In the x-y plane, for t > 0, denote by S(t) the area of the part enclosed by the curve y = et x , the x-axis, y-axis
1
4
and the line x = . Show that S(t) > . If necessary, you may use e3 > 20.
t
3
858 1982 Saint Paul University
On the plane S in a space, given are unit circle C with radius 1 and the line L. Find the volume of the solid
bounded by the curved surface formed by the point P satifying the following conditions:
(a) The point of intersection Q of the line passing through P and perpendicular to S are on the perimeter or the
inside of C.
(b) If A, B are the points of intersection of the line passing through Q and parallel to L, then P Q = AQ BQ.
857 1970 Tokyo Medical and Dental University
1

Let f (x) = lim (cosn x + sinn x) n for 0 x


n

.
2

(1) Find f (x).


(2) Find the volume of the solid generated by a rotation of the figure bounded by the curve y = f (x) and the line
y = 1 around the y-axis.
856
19

On the coordinate plane, find the area of the part enclosed by the curve C : (a + x)y 2 = (a x)x2 (x 0) for
a > 0.
855
Let f (x) be a function which is differentiable
twice and f (x) > 0)on [0, 1].
(Z
n1  
1
k
1X
.
f
For a positive integer n, find lim n
f (x) dx
n
n
n
0
k=0

854
y2
= 1 on the coordinate plane. Denote by Sn the area of the common part of the n + 1 s
3
k
(k = 0, 1, 2, , n) radians counterclockwise about the origin. Find lim Sn .
figures formed by rotating F of
n
2n
Given a figure F : x2 +

853
Let 0 < a <

. Find lim 3
a+0 a
2

ln (1 + tan a tan x) dx.

852
Let f (x) be a polynomial. Prove that if

for each case as follows.


(1) gn (x) = (1 + x)n ;
(2) gn (x) = sin nx;
(3) gn (x) = enx .

f (x)gn (x) dx = 0 (n = 0, 1, 2, ), then all coefficients of f (x) are 0

851 2012 National Defence Medical College


Let T be a period of a function f (x) = | cos x| sin x (, ). Find lim

nT

ex f (x) dx.

850
Evaluate

{(1 x sin 2x)ecos

+ (1 + x sin 2x)esin

} dx.

849
Evaluate

e2

(2x2 + 2x + 1)ex

dx.
x

848

Evaluate

1 sin
sin 2 ln
r cos
d.
1 + sin
(1 + cos 2) ln
cos

847 2012 Tohoku University

Consider a right-angled triangle with AB = 1, AC = 3, BAC = . Let P1 , P2 , , Pn1 (n 2) be the


2
points which are closest from A, in this order and obtained by dividing n equally parts of the line segment AB. Denote
by A = P0 , B = Pn , answer the questions as below.
(1) Find the inradius of Pk CPk+1 (0 k n 1).
(2) Denote by Sn the total sum of the area of the incircle for Pk CPk+1 (0 k n 1).
n1
1
3
1X
In , then find the limit lim In .
Let In =
 2 , show that nSn
n
n
4
k
k=0 3 +
n
(3) Find the limit lim nSn .
n

20

846 2012 Saitama University


Z 1
For a > 0, let f (a) = lim
|ax + x ln x| dx. Let a vary in the range 0 < a < +, find the minimum value of
t+0

f (a).

845 2012 Iwate University


Consider for a real number t > 1, I(t) =
Find the minimum value of I(t) (t > 1).

4t4

(x 4) x + 4 dx.

844 2012 Mie University


Let be a solution satisfying the equation |x| = ex . Let In =

lim n2 In .

(xenx + xn1 ) dx (n = 1, 2, ). Find

843
Let f (x) be a continuous function such that

f (x) dx = 1. Find f (x) for which

minimized.

(x2 + x + 1)f (x)2 dx is

842 2012 Yamanashi University


Z
sinn x dx (n = 1, 2, , ). Find lim nSn Sn+1 .
Let Sn =
n

841 2012 Kobe University


Z x1
Z x
1
1
dt
+
dt (x > 0).
Find
2
2
0 1+t
0 1+t
840 2012 Ehime University
Let x, y be real numbers. For a function f (t) = x sin t + y cos t, draw the domain of the points (x, y) for which
the following inequality holds:
Z
Z



{f (t)}2 dt.
f (t) cos t dt

839 2012 Tokyo City University


Z 1p
Evaluate
1 x2 dx.
1
2

838 2012 Toyama University


Z 1
2

e1
<
ex dx < .
Prove that :
e
4
0
837 2012 Shiga University of Medical Science

 2k1
n
X
x2k
x
.
+
Let fn (x) =
(1)k+1
2k 1
2k
k=1
Find lim fn (1).
n

836
Evaluate

esin x cos2 (sin x) cos x dx.

835 2012 Miyazaki University


Evaluate the following definite integrals:

21

x1
dx;
2 2x + 2
x
Z1 1
4x
e
dx;
(b)
2x
Z 0e e + 2

(c)
x ln x dx;

Z1 3 
1
2
(d)
cos x sin 3x sin 5x dx.
4
0
(a)

834 2012 Wakayama Medical


Find the maximum and minimum areas of the region enclosed by the curve y = |x|e|x| and the line y = a (0 a e)
at [1, 1].
833 2012 Kumamoto University
Z x
Z
Let f (x) =
et (cos t + sin t) dt, g(x) =
0

For a real number a, find

0
2a

et (cos t sin t) dt.

e
.
(n) (a)}2 + {g (n) (a)}2
{f
n=1

832 2012 Osaka University of Education


Find the limit
1
n n ln n
lim

(n+1)

(sin2 t)(ln t) dt.

831 1977 Tsukuba Univrersity


Let n be a positive integer. Answer the following questions:
(1) Find the maximum value of fn (x) = xn ex for x 0.
(2) Show that lim fn (x) = 0.
Z x
x
fn (t) dt. Find lim In (x).
(3) Let In =
x

830 2012 Tottori University


n
1 X ln k
Find lim
.
n (ln n)2
k
k=3

829 2012 Kyoto Institute of Technology


Let a be a positive constant. Find the value of ln a such that
Z e
ln(ax) dx Z e
ln(ax)
1Z
=
dx.
e
x
1
x dx
1

828 2012 Miyagi University of Education

Find a function f (x), which is differentiable and f (x) is continuous, such that
827 2012 Kyusyu University
Z (2k+1)
X
xex sin x dx.
Find lim
n

k=0

2k

826 2012 The University of Tokushima

22

x
0

f (t) cos(x t) dt = xe2x .

Let k (k = 1, 2, , n), 1 < 2 < < n be the solution of such that


n
k
1X
lim
cos .
n n
2

3 sin n + cos n = 0. Find

k=1

825 2012 Hokkaido University


Answer the following questions.
x3
sin x x.
(1) For x 0, show that x
6
Z x
3
5
x
x
x3
(2) For x 0, show that

t sin t dt
.
3
30
3
0
(3) Find the limit
lim

x0

sin x x cos x
.
x3

824 1999 Nara Medical University


In the xy-plane, for a > 1 denote by S(a) the area of the figure bounded by the curve y = (a x) ln x and the
x-axis.
S(a)
Find the value of integer n for which lim n
is non-zero real number.
a a ln a
823 2007 Kobe University
Let C be the curve expressed by x = sin t, y = sin 2t



.
0t
2

(1) Express y in terms of x.


(2) Find the area of the figure D enclosed by the x-axis and C.
(3) Find the volume of the solid generated by a rotation of D about the y-axis.
822 2012 Iwate University
For nZ= 0, 1, 2, , let
n+1
{xex (n + 1)en1 (x n)} dx,
an =
Z nn+1
bn =
{xex (n + 1)en1 } dx.
n

Find lim

n
X

k=0

(ak bk ).

821 1996 Yamaguchi University


Z 43
11
1
7
Prove that ln
<
dx < ln .
1
27
ln(1 x)
15
4
820 1994 Seikei University
k
Let Pk be a point whose x-coordinate is 1 + (k = 1, 2, , n) on the curve y = ln x. For A(1, 0), find the
n
n
1X
2
limit lim
AP k .
n n
k=1

819 2012 Waseda University of Education


For real numbers a, b with 0 a , a < b, let I(a, b) =
Determine the value of a such that lim I(a, b) = 0.

ex sin x dx.

818 2012 Tokyo Medical and Dental University


Z
For a function f (x) = x3 x2 + x, find the limit lim
n

2n

23

f 1 (x)3

1
dx.
+ |f 1 (x)|

817 2012 Osaka University




1
1
t+
, g(t) = t2 2 ln t. When real number t moves in the range of t > 0, denote
Define two functions f (t) =
2
t
by C the curve bywhich 
the point (f (t), g(t)) draws on the xy-plane. Let a > 1. Find the area of the part bounded
1
1
by the line x =
a+
and the curve C.
2
a
816 2012 Kyusyu University
Find the volume of the solid of a circle x2 + (y 1)2 = 4 generated by a rotation about the x-axis.
815


n 
X
i
i

1 sin
Prove that :
cos
< 1.

4n
4n
i=0

814 2012 Nagasaki University

Find the area of the region bounded by C : y = x4 + 8x3 18x2 + 11 and the tangent line which touches C at
distinct two points.
813
Z

Let a be a real number. Find the minimum value of

|ax x3 | dx.

812 2012 Kwansei Gakuin University


Let f (x) =

f (x)
1
cos 2x (a + 2) cos x + a + 1
. For constant a such that lim
= , evaluate
x0 x
sin x
2

1
dx.
f (x)

811 2012 Okayama Prefectural University


Z a+
Let a be real number. Evaluate
|x| cos x dx.
a

810 2012 Keio University


x

Given the functions f (x) = xe + 2x

|g(t)| dt 1, g(x) = x x

f (t) dt, evaluate


0

|g(t)| dt.

809 2012 Kobe University


For a > 0, denote by S(a) the area of the part bounded by the parabolas y =
Find the maximum area of S(a).

1
1 2
x 3a and y = x2 +2axa3 a2 .
2
2

808 2012 Kyusyu University


For a constant c, a sequence an is defined by an =

Find lim an .

  n
1
nxn1 ln
dx (n = 1, 2, 3, ).
x

807 2012 Osaka University


Define a sequence an satisfying :
a1 = 1, an+1 =

Find lim

2m
X

n=m+1

nan
(n = 1, 2, 3, ).
2 + n(an + 1)

an .

806 2012 Osaka Prefecture University

24

Let n be positive integers and t be a positive real number. Evaluate

2n
t

|x sin tx| dx.

805 2012 Yokohama National University


Prove the following inequalities:
(1) For 0 x 1,

1
(2)
3
6

3
2

1
1
1.
1 x
3
1 + x2

dx .
3
1 x4

804 2012 Hokkaido University


For a > 0, find the minimum value of I(a) =

| ln ax| dx.

803 2012 Yokohama National University


Answer the following
questions:
Z 1
(1) Evaluate
(1 x2 )e2x dx.
1
1

(2n)! n
.
(2) Find lim
n n!nn
802 2012 Osaka Prefecture University
Let k and a are positive constant. Denote by V1 the volume of the solid generated by a rotation of the figure
x
(x 0), the line x = a and the x-axis around the x-axis, and denote by V2 that
enclosed by the curve C : y =
x+k
a
of the solid by a rotation of the figure enclosed by the curve C, the line y =
and the y-axis around the y-axis.
a+k
V2
.
Find the ratio
V1
801 2012 Chiba University
Answer the following questions:
(1) Let f (x) be a function such that f (x) is continuous and f (a) = f (b) = 0 for some a < b.

Z b
a+b
Prove that f (b) f (a) =
x f (x) dx.
2
a
(2) Consider the running a car on straight road. After a car which is at standstill at a traffic light started at time
0, it stopped again at the next traffic light apart a distance L at time T . During the period, prove that there is an
4L
instantfor which the absolute value of the acceleration of the car is more than or equal to 2 .
T
800 2012 Kumamoto University
For a positive constant a, find the minimum value of f (x) =

| sin t ax cos t| dt.

799 2012 Tokyo Institute of Technology


Let n be positive integer. Define a sequence {ak } by
k

1
1
nX
a1 =
ai (k = 1, 2, 3, ).
, ak+1 =
+
n(n + 1)
k + n + 1 k i=1
(1) Find a2 and a3 .
(2) Find the general term ak .
n
X

(3) Let bn =
ak . Prove that lim bn = ln 2.
k=1

798 2012 Tokyo Institute of Technology

25

Denote by C, l the graphs of the cubic function C : y = x3 3x2 + 2x, the line l : y = ax.
(1) Find the range of a such that C and l have intersection point other than the origin.
(2) Denote S(a) by the area bounded by C and l. If a move in the range found in (1), then find the value of a for
which S(a) is minimized.
797 2012 Tokyo Institute of Technology

In the xyz-space take four points P (0, 0, 2), A(0, 2, 0), B( 3, 1, 0), C( 3, 1, 0). Find the volume of the
part satisfying x2 + y 2 1 in the tetrahedron P ABC.
796 2012 Yokohama National University
Answer the following questions:
(1) Let a be non-zero constant. Find

x2 cos(a ln x) dx.

(2) Find the volume of the solid generated by a rotation of the figures enclosed by the curve y = x cos(ln x), the

x-axis and the lines x = 1, x = e 4 about the x-axis.


795 2012 Yokohama National University
Z 2
2 + sin x
dx.
Evaluate

1 + cos x
3
794 2012 Tohoku University
Z 2
cos |t x|
dt for 0 x .
Define a function f (x) =
1 + sin |t x|
0
Find the maximum and minimum value of f (x) in 0 x .
793 2012 Kyoto University
Find the area of the figure bounded by two curves y = x4 , y = x2 + 2.
792 2012 Kyoto University
Answer the following questions:
1
(1) Let a be positive real number. Find lim (1 + an ) n .
n
Z 3
p
1
ln 1 + x2 dx.
(2) Evaluate
2
x
1
791 2012 Tokyo University

Let S be the domain in the coordinate plane determined by two inequalities:


y

1
1 2 x2
x ,
+ 4y 2 .
2
4
8

Denote by V1 the volume of the solid by a rotation of S about the x-axis and by V2 , by a rotation of S about the
y-axis.
(1) Find the values of V1 , V2 .
V2
(2) Compare the size of the value of
and 1.
V1
790 2012 Tokyo University
Define a parabola C by y = x2 + 1 on the coordinate plane. Let s, t be real numbers with t < 0. Denote by l1 , l2
the tangent lines drawn from the point (s, t) to the parabola C.
(1) Find the equations of the tangents l1 , l2 .
(2) Let a be positive real number. Find the pairs of (s, t) such that the area of the region enclosed by C, l1 , l2 is
a.
789 2012 Waseda University
Find the non-constant function f (x) such that f (x) = x2
788 2012 Waseda University
26

(f (t) + x)2 dt.

For a function f (x) = ln(1 + 1 x2 ) 1 x2 ln x (0 < x < 1), answer the following questions:
(1) Find f (x).
(2) Sketch the graph of y = f (x).
(3) Let P be a mobile point on the curve y = f (x) and Q be a point which is on the tangent at P on the curve
y = f (x) and such that P Q = 1. Note that the x-coordinate of Q is les than that of P . Find the locus of Q.
787 2012 Waseda University
Take two points A (1, 0), B (1, 0) on the xy-plane. Let F be the figure by which the whole points P on the

plane satisfies AP B and the figure formed by A, B.


4
Answer the following questions:
(1) Illustrate F .
(2) Find the volume of the solid generated by a rotation of F around the x-axis.
786 2012 Chuo University
For each positive integer n, define Hn (x) = (1)n ex

dn x2
e
.
dxn

(1) Find H1 (x), H2 (x), H3 (x).


d
(2) Express
Hn (x) in terms of Hn (x), Hn+1 (x). Then prove that Hn (x) is a polynomial with degree n by
dx
induction.
Z a
2
xHn (x)ex dx in terms of Hn1 (a), Hn2 (a), Hn2 (0).
(3) Let a be real number. For n 3, express Sn (a) =
0

(4) Find lim S6 (a).


a

If necessary, you may use lim xk ex = 0 for a positive integer k.


x

785 2012 Gakusyuin University


For a positive real number x, find the minimum value of f (x) =

2x

(t ln t t) dt.

784 2012 Tokyo University of Science


ln x
(x > 0). In the coordinate plane, denote by Sn the area of
xn
the figure enclosed by y = fn (x) (x t), the x-axis and the line x = t and denote by Tn the area of the rectagle with
four vertices (1, 0), (t, 0), (t, fn (t)) and (1, fn (t)).
(1) Find the local maximum fn (x).
(2) When t moves in the range of t > 1, find the value of t for which Tn (t) Sn (t) is maximized.
(3) Find S1 (t) and Sn (t) (n 2).
(4) For each n 2, prove that there exists the only t > 1 such that Tn (t) = Sn (t).
ln x
= 0.
Note that you may use lim
x x
Define for positive integer n, a function fn (x) =

783 2012 Gakusyuin University


Define a sequence a1 = 0,
(1) Find an . r

1
1

= 2n + 1 (n = 1, 2, 3, ).
1 an+1
1 an

k+1

(1 ak+1 ) for k = 1, 2, 3, .
k
n
X

Prove that
bk < 2 1 for each n.
(2) Let bk =

k=1

782 2012 Saint Pauls University




1
1
(t > 0) on C.
Let C be the part of the graph y = (x > 0). Take a point P t,
x
t
(i) Find the equation of the tangent l at the point A(1, 1) on the curve C.
(ii) Let m be the line passing through the point P and parallel to l. Denote Q be the intersection point of the line
m and the curve C other than P . Find the coordinate of Q.
(iii) Express the area S of the part bounded by two line segments OP, OQ and the curve C for the origin O in
terms of t.
27

(iv) Express the volume V of the solid generated by a rotation of the part enclosed by two lines passing through
the point P and pararell to the y-axis and passing through the point Q and parallel to y-axis, the curve C and the
x-axis in terms of t.
S
.
(v) lim
t10 V
781 2012 Sophia University
Let , m be the tangent lines passing through the point A(a, a 1) on the line y = x 1 and touch the parabola
y = x2 . Note that the slope of is greater than that of m.
(1) Express the slope of in terms of a.
(2) Denote P, Q be the points of tangency of the lines , m and the parabola y = x2 . Find the minimum area of
the part bounded by the line segment P Q and the parabola y = x2 .
(3) Find the minimum distance between the parabola y = x2 and the line y = x 1.
780 2012 The Jikei University School of Medicine
Let n 3 be integer. Given a regular n-polygon P with side length 4 on the plane z = 0 in the xyz-space.Let G
be a circumcenter of P . When the center of the sphere B with radius 1 travels round along the sides of P , denote by
Kn the solid swept by B.
Answer the following questions.
(1) Take two adjacent vertices P1 , P2 of P . Let Q be the intersection point between the perpendicular dawn from
G to P1 P2 , prove that GQ > 1.
(2) (i) Express the area of cross section S(t) in terms of t, n when Kn is cut by the plane z = t (1 t 1).
(ii) Express the volume V (n) of Kn in terms of n.
(3) Denote by l the line which passes through G and perpendicular to the plane z = 0. Express the volume W (n)
of the solid by generated by a rotation of Kn around l in terms of n.
V (n)
.
(4) Find lim
n W (n)
779 2012 Sophia University
Consider parabolas Ca : y = 2x2 + 4ax 2a2 + a + 1 and C : y = x2 2x in the coordinate plane. When Ca and
C have two intersection points, find the maximum area enclosed by these parabolas.
778 2012 Nippon Medical School
In the xyz space with the origin O, Let K1 be the surface and inner part of the sphere centered on the point
(1, 0, 0) with radius 2 and let K2 be the surface and inner part of the sphere centered on the point (1, 0, 0) with
radius 2. For three points P, Q, R in the space, consider points X, Y defined by
1
OX = OP + OQ, OY = (OP + OQ + OR).
3
(1) When P, Q move every cranny in K1 , K2 respectively, find the volume of the solid generated by the whole
points of the point X.
(2) Find the volume of the solid generated by the whole points of the point R for which for any P belonging to K1
and any Q belonging to K2 , Y belongs to K1 .
(3) Find the volume of the solid generated by the whole points of the point R for which for any P belonging to K1
and any Q belonging to K2 , Y belongs to K1 K2 .
777 2012 Nippon Medical School
Given two points P, Q on the parabola C : y = x2 x 2 in the xy plane. Note that the x coodinate of P is less
than that of Q.
(a) If the origin O is the midpoint of the linesegment P Q, then find the equation of the line P Q.
(b) If the origin O divides internally the line segment P Q by 2 : 1, then find the equation of P Q.
(c) If the origin O divides internally the line segment P Q by 2 : 1, find the area of the figure bounded by the
parabola C and the line P Q.
776
Evaluate

1+ 5
2

1 5
2

(2x2 1)e2x dx.

775 2012 Tokyo Womens Medical University


28

Let a be negative constant. Find the value of a and f (x) such that
numbers t.

t
2
a
2

f (x) dx = t2 + 3t 4 holds for any real

774 2012 Tokyo Womens Medical University


Z a x
12
e + ex
dx =
.
Find the real number a such that
2
5
0
773 2012 Showa University
For x 0 find the value of x by which f (x) =

x
0

3t (3t 4)(x t)dt is minimized.

772 2012 Aichi Medical University


Given are three points A(2, 0, 2), B(1, 1, 0), C(0, 0, 3) in the coordinate space. Find the volume of the solid
of a triangle ABC generated by a rotation about z-axis.
771 1997 Tokyo University
(1) Find the range of a for which there exist two common tangent lines of the curve y =

8 3
x and the parabola
27

y = (x + a)2 other than the x axis.


(2) For the range of a found in the previous question, express the area bounded by the two tangent lines and the
parabola y = (x + a)2 in terms of a.
770
Find the value of a such that :
101a = 6539

x12 + 31
dx.
1 + 2011x

769
In xyz space, find the volume of the solid expressed by x2 + y 2 z

3y + 1.

768
Let r be a real such that 0 < r 1. Denote by V (r) the volume of the solid formed by all points of (x, y, z)
satisfying
x2 + y 2 + z 2 1, x2 + y 2 r2
in xyz-space.
(1) Find V (r).
(2) Find lim

V (1) V (r)
3

(1 r) 2
V (r)
(3) Find lim

r+0 r2
r10

767
For 0 t 1, define f (t) =

| sin x t| dx. Evaluate

f (t) dt.

766
Let f (x) be a continuous function defined on 0 x and satisfies f (0) = 1 and
Z

Evaluate

2
Z
(sin x + cos x)f (x) dx =
{f (x)}2 dx.
0

{f (x)}3 dx.

765

29

Define two functions g(x), f (x) (x 0) by g(x) =


Z 1
2
Now we know that f (x) =
e(1+s )x ds.

t2

dt, f (x) =

1
0

e(1+s )x
ds.
1 + s2

(1) Find f (0).

(2) Show that f (x) ex (x 0).


42
(3) Let h(x) = {g( x)} . Show that f (x) = h (x).
(4) Find lim g(x).
x+

764
Find lim

ex | sin nx| dx.

763
Evaluate

x2
dx.
(x2 + 4) x

762
Define a function fn (x) (n = 0, 1, 2, ) by
f0 (x) = sin x, fn+1 (x) =

fn (t) sin(x + t)dt.

(1) Let fn (x) = an sin x + bn cos x. Express an+1 , bn+1 in terms of an , bn .


X
(2) Find
fn
.
4
n=0
761
1
Find lim
n n

s
n

(4n)!
.
(3n)!

760
Prove that there exists a positive integer n such that

x sin (x2 x + 1)dx

n+2
n
sin
.
n+1
n+3

759
Given a regular tetrahedron P QRS with side length d. Find the volume of the solid generated by a rotation around
the line passing through P and the midpoint M of QR.
758
Find the slope of a line
passing through the point (0, 1) with which the area of the part bounded by the line and
5 5
2
.
the parabola y = x is
6
757
Evaluate
Z

2
(x2 + x + 1)3 {ln(x2 + x + 1) + 2}
(2x + 1)ex +x+1 dx.
(x2 + x + 1)3

756
Let a be real number. A circle C touches the line y = x at the point (a, a) and passes through the point (0, 1).
Denote by P the center of C. When a moves, find the area of the figure enclosed by the locus of P and the line y = 1.
755

30



Given mobile points P (0, sin ), Q(8 cos , 0) 0
on the x-y plane. Denote by D the part in which line
2
segment P Q sweeps. Find the volume V generated by a rotation of D around the x-axis.
754
Let Sn be the area of the figure enclosed by a curve y = x2 (1 x)n (0 x 1)and the x-axis. Find lim

n
X

Sk .

k=1

753
Find lim

2n
X

2n2

1
n

1
n

k=1

n
.
+ 3nk + k 2

752
Find lim


1
ln n .
2

(sin2 x) ln(x + n)dx

751
Find lim


1
(sin x) ln(x + n)dx ln n .
2
2

750 2011 Tsukuba University


Let an (n 1) be the value for which

2x

et dt (x 0) is maximal. Find lim ln an .


n

749 2011 Gakusyuin University


Let m be a positive integer. A tangent line at the point P on the parabola C1 : y = x2 + m2 intersects with the
parabola C2 : y = x2 at the points A, B. For the point Q between A and B on C2 , denote by S the sum of the areas
of the region bounded by the line AQ,C2 and the region bounded by the line QB, C2 . When Q move between A and
B on C2 , prove that the minimum value of S doesnt depend on how we would take P , then find the value in terms
of m.
748 2011 Hirosaki University
Evaluate
Z the following integrals:
cos mx cos nx dx (m, n = 1, 2, ).
(1)

Z0 3 
1
x
(2)
(ln x)2 dx.
x
1
747 2011 Kanazawa University
Z 4
x  x
e dx 2e2 + 30.
Prove that
1 cos
2
0
746 2011 Mie University
Prove the following inequality:
nn en+1 n!

1
(n + 1)n+1 en+1 .
4

745 2011 Toyama University


When real numbers a, b move satisfying

(a cos x + b sin x)2 dx = 1, find the maximum value of

b sin x)2 dx.

(ex a cos x

744
Let a, b be real numbers. If
is minimized.

(ax b) dx 3 holds, then find the values of a, b such that

31

(x 3)(ax b) dx

743
Evaluate

ln(1 +

3
sin ) cos d.

742
Evaluate
Z

1 x2

dx.
(1 + x2 ) 1 + x4

741
Evaluate
Z

(x 1)2 (cos x + 1) (2x 1) sin x

dx.
(x 1 + sin x)2

740

2x2
, C2 : y = r2 x2 have each tangent line at their point
+1
of intersection and at which their tangent lines are perpendicular each other, then find the area of the figure bounded
by C1 , C2 .
Let r be a positive constant. If 2 curves C1 : y =

x2

739
Find the function f (x) such that :
f (x) = cos x +

f (y) sin(x y) dy.

738
Answer the following questions:
(1) Find the value of a for which S =

(x a sin 3x)2 dx is minimized, then find the minimum value.

(2) Find the vlues of p, q for which T =

(sin 3x px qx2 )2 dx is minimized, then find the minimum value.

737
Let a, b real numbers such that a > 1, b > 1. Prove the following inequality:

Z 1
1 + b|x| 1 + a|x|
dx < a + b + 2.
+
1 + ax
1 + bx
1
736
Evaluate
Z

(ex + 1){ex + 1 + (1 + x + ex ) ln(1 + x + ex )}


dx.
1 + x + ex

735
Evaluate
the following definite integrals:
Z 2
(a)
x tan(x2 ) dx;
0
Z 13
xe3x dx;
(b)
0 e
Z e
1
dx;
(c)
x
ln
x
Ze 3 2
x +1
(d)
dx.
x(x
+ 1)
2
32

734
Find the extremum of f (t) =

ln x
dx (t > 0).
x+t

733
Find lim

x2 e( n ) dx.
x

732
Let a be parameter such that 0 < a < 2. For 0 < x < 2, find the extremum of F (x) =

x+a

1 cos d.

731
Let C be the point of intersection of the tangent lines l, m at A(a, a2 ), B(b, b2 ) (a < b) on the parabola y = x2
1
respectively. When C moves on the parabola y = x2 x 2, find the minimum area bounded by 2 lines l, m and
2
the parabola y = x2 .
730
Let an be the local maximum of fn (x) =
1

a2n n
.
Find lim ln
n
an

xn ex+n
(n = 1, 2, ) for x > 0.
n!

729
Evaluate

ln x 1
dx.
x2 (ln x)2

728
Evaluate
Z

12

sin x cos x x(sin x + cos x) + 1


dx.
x2 x(sin x + cos x) + sin x cos x

727
a x
x
(e a + e a ). Denote by (t) the length of the part a y t for C and denote
2
S(t)
.
by S(t) the area of the part bounded by the line y = t (a < t) and C. Find lim
t (t) ln t
For positive constant a, let C : y =

726
Let P (x, y) (x > 0, y > 0) be a point on the curve C : x2 y 2 = 1. If x =
bounded by the line OP , the x axis and the curve C in terms of u.
725
For a > 1, evaluate

a
1
a

1
(ln x) ln (x2 + 1) dx.
x

724
Find lim

(1 + n)

1
n

1

n  n3
n  nn n
n  n2 
1+
... 1 +
.
1+
2
3
n

723
Evaluate

{1 (x 1)ex } ln x
dx.
(1 + ex )2

33

eu + eu
(u 0), then find the area
2

722
Find the continuous function f (x) such that :
Z t

Z x
1
f (t)
f (t) dt dt = f (x) + .
2
0
0
721
For constant a, find the differentiable function f (x) satisfying

(ex aet )f (t) dt = 0.

720
Evaluate

|x2 2 sin2 x| dx.

719
Compute

sin t cos t sin(2 cos t) dt.

718
Z 1

X
1
Find
(1 x)2 (1 + x)n dx (n 1).
n
2
1
n=1
717
Let an be the area of the part enclosed by the curve y = xn (n 1), the line x =
0 ln 2

1
and the x axis. Prove that :
2

1
1
(a1 + a2 + + an ) n+1 .
2
2

716
Prove that :
Z

n1

(ln x) dx = (1)

 m
n
X
1
nm n!
(1)
.
n! + e
m! 2
m=0

715
Find the differentiable function f (x) with f (0) 6= 0 satisfying f (x + y) = f (x)f (y) + f (x)f (y) for all real numbers
x, y.
714
Find the area enclosed by the graph of a2 x4 = b2 x2 y 2 (a > 0, b > 0).
713
If a positive sequence {an }n1 satisfies

an

xn dx = 2, then find lim an .


n

712
Evaluate


1
tan x
dx.
+
tan x (ln sin x) ln cos x

711
Evaluate

e2

4(ln x)2 + 1
3

(ln x) 2

dx.

710
34

Evaluate

sin (sin cos + 2)


d.
cos4

709
Evaluate

708
Find lim

x p
1 x2 dx.
1+x
1

x2 | sin nx| dx (n = 1, 2, ).

707

1 
0<
. For a constant t > 0, let the
cos
2
line l pass through the point P (t, 0) and is perpendicular to the x-axis,intersects with the curve C at Q. Denote by
S1 the area of the figure bounded by the curve C, the x-axis, the y-axis and the line l, and denote by S2 the area of
S1 S2
OP Q. Find lim
.
t
ln t
Consider a curve C on the x-y plane expressed by x = tan , y =

706
In the xyz space, consider a right circular cylinder with radius of base 2, altitude 4 such that
 2
x + y2 4
0z4
Let V be the solid formed by the points (x, y, z) in the circular cylinder satisfying

z (x 2)2
z y2
Find the volume of the solid V .
705
The parametric equations of a curve are given by x = 2(1 + cos t) cos t, y = 2(1 + cos t) sin t (0 t 2).
(1) Find the maximum and minimum values of x.
(2) Find the volume of the solid enclosed by the figure of revolution about the x-axis.
704
A function fn (x) (n = 0, 1, 2, 3, ) satisfies the following conditions:
(i) f0 (x) = eZ2x + 1.
x
2xn+1
(n = 1, 2, 3, ).
(ii) fn (x) =
(n + 2t)fn1 (t) dt
n+1
0 


X
1
Find
fn
.
2
n=1
703
Given a line segment P Q with endpoints on the parabola y = x2 such that the area bounded by P Q and the
4
parabola always equal to . Find the equation of the locus of the midpoint M .
3
702
f (x) is a continuous function defined in x > 0. For all a, b (a > 0, b > 0), if
c
then prove that f (x) = d (c: constant).
x
701

35

f (x) dx is determined by only

b
,
a

Evaluate
Z

#
x
(1 + cos x) 1 tan tan(x + sin x) tan(x sin x)
2
"

tan(x + sin x)

dx.

700
Evaluate

x2 cos2 x x sin x cos x 1


dx.
(1 + x sin x)2

699
Find the volume of the part bounded by z = x + y, z = x2 + y 2 in the xyz space.
698
For a positive integer n, let denote Cn the figure formed by the inside and perimeter of the circle with center the
origin, radius n on the x-y plane.
Denote by N (n) the number of a unit square such that all of unit square, whose x, y coordinates of 4 vertices are
integers, and the vertices are included in Cn .
N (n)
= .
Prove that lim
n n2
697
Find the volume of the solid of the domain expressed by the inequality x2 x y x, generated by a rotation
about the line y = x.
696
Let P (x), Q(x) be polynomials such that :
Z

{P (x)} dx = 14,

P (x) dx = 4,

Find the maximum and the minimum value of

{Q(x)} dx = 26,

P (x)Q(x)dx.

695
For a positive integer n, let
Sn =

Answer the following questions:


(1) Show the following inequality.

X (1)k1
1 (x)n
dx, Tn =
.
1+x
k(k + 1)
k=1


Z

Sn

(2) Express Tn 2Sn in terms of n.


(3) Find the limit lim Tn .



1
1
dx
1+x
n+1

694
Prove the following inequality:
Z

1
(ln x)2009
dx >
.
x2
2010 2011 2012

693
Evaluate

p
4
1 + | cos x| dx.
36

Q(x) dx = 2.

692
Evaluate

12

tan2 x 3
dx.
3 tan2 x 1

691 2011 Yokohama National Universty


ln x
Let a be a constant. In the xy palne, the curve C1 : y =
touches C2 : y = ax2 . Find the volume of the solid
x
generated by a rotation of the part enclosed by C1 , C2 and the x axis about the x axis.
690
Find the maximum value of f (x) =

t sin(x + t) dt.

689
Let C : y = x2 + ax + b be a parabola passing through the point (1, 1). Find the minimum volume of the figure
enclosed by C and the x axis by a rotation about the x axis.
688 2011 Tokyo Institute of Technology
Z 2
| cos t x sin 2t| dt.
For a real number x, let f (x) =
0

(1) Find the minimum value of f (x).


Z 1
f (x) dx.
(2) Evaluate
0

687 2011 Tokyo University


(1) Let x > 0, y be real numbers. For variable t, find the difference of Maximum and minimum value of the
quadratic function f (t) = xt2 + yt in 0 t 1.
(2) Let S be the domain of the points (x, y) in the coordinate plane forming the following condition:
For x > 0 and all real numbers t with 0 t 1 , there exists real number z for which 0 xt2 + yt + z 1 .
Sketch the outline of S.
(3) Let V be the domain of the points (x, y, z) in the coordinate space forming the following condition:
For 0 x 1 and for all real numbers t with 0 t 1, 0 xt2 + yt + z 1 holds.
Find the volume of V .
686 2011 Tokyo University
Let L be a positive constant. For a point P (t, 0) on the positive part of the x axis on the coordinate plane, denote
Q(u(t), v(t)) the point at which the point reach starting from P proceeds by distance L in counter-clockwise on the
perimeter of a circle passing the point P with center O.
(1) Find u(t), v(t).
Z 1p
(2) For real number a with 0 < a < 1, find f (a) =
{u (t)}2 + {v (t)}2 dt.
a

f (a)
.
(3) Find lim
a+0 ln a

685 2011 Tokyo University


Suppose that a cubic function with respect to x, f (x) = ax3 + bx2 + cx + d satisfies all of 3 conditions:
Z 1
f (1) = 1, f (1) = 1,
(bx2 + cx + d) dx = 1.
1

Find f (x) for which I =

1
2

{f (x)}2 dx is minimized, the find the minimum value.

684 2011 Kyoto University



3
On the xy plane, find the area of the figure bounded by the graphs of y = x and y = x2 3
4
683 2011 Kyoto University

37



2.

Evaluate

1
2

p
(x + 1) 1 2x2 dx.

682 2011 Waseda University of Education

On the x-y plane, 3 half-lines y = 0, (x 0), y = x tan (x 0), y = 3x (x 0) intersect with the circle with

the center the origin O, radius r 1 at A, B, C respectively. Note that .


6
3
If the area of quadrilateral OABC is one third of the area of the regular hexagon which inscribed in a circle with
Z 3
radius 1, then evaluate
r2 d.

681 2011 University of Occupational and Environmental Health


Z 2 p
1 2 sin 2x + 3 cos2 x dx.
Evaluate
0

680 2011 Keio University


Z a 
x a
dx.
Let a > 0. Evaluate
x2 1
a
0
679 2011 Hosei University
Find

3n
X

k=1

1
Z

.
k

x(1 x) dx

678 2011 Doshisya University


Evaluate
Z

1+

n
X

!2

k cos kx

k=1

dx (n = 1, 2, ).

677 2011 Tokyo University of Science


Let a, b be positive real numbers with a < b. Define the definite integrals I1 , I2 , I3 by I1 =
Z b
Z b
cos (x2 )
sin (x2 )
dx,
I
=
dx.
3
x2
x4
a
a
1
(1) Find the value of I1 + I2 in terms of a, b.
2
3
(2) Find the value of I2 I3 in terms of a, b.
2
Z 2(n+1)
Z 2(n+1)
3
sin (x2 )
dx.
(3) For a positive integer n, define Kn =
sin (x2 ) dx +
4 2n
x4
2n

Find the value of lim 2n 2nKn .

sin (x2 ) dx, I2 =

676 2011 Tokyo University of Science


Z 2
4
4
Let f (x) = cos x + 3 sin x. Evaluate
|f (x)| dx.
0

675 2011 Ritsumeikan University


In the coordinate plane with the origin O, consider points P (t+2, 0), Q(0, 2t2 2t+4) (t 0). If the y-coordinate
of Q is nonnegative, then find the area of the region swept out by the line segment P Q.
674 2011 Doshisya University
Z 1
x2 + 5
dx.
Evaluate
2
0 (x + 1) (x 2)
673 2011 Ritsumeikan University
38

Let f (x) =


 r
1+x
1
.
dt. For 1 x < 1, find cos 2f
1 + t2
1x

672 2011 Tokyo Womens Medical University


Find the limit




1
1
1
1
1

.
+
+
+ +
sin
n
n
n+1
n+2
n+3
2n
lim

671 2011 Showa University


For real number k find the minimum value of I =

(x + 1 + k sin x)2 dx.

670 2011 Nippon Medical School


Let a, b be constants. Suppose that the following equation holds for all real numbers x. Answer the questions as
follows.
Z x
Z a
f (y)dy x2 6x + b =
(x3 + x2 y)f (y)dy.
a

Q1. For a = 1, find the values of

f (y)dy,

yf (y)dy and b.

Q2. For b = 0, find f (x), a.


Q3. Find the condition of a3 for which there exists only one pair of f (x) and b.
669
Find the differentiable function defined in x > 0 such that

f (x)

f 1 (t) dt =


1 3
x2 8 .
3

668 2011 Kyorin University


Consider two curves y = sin x, y = sin 2x in 0 x 2.
(1) Let (, ) (0 < < ) be the intersection point of the curves. If sin x sin 2x has a local minimum at x = x1
and a local maximum at x = x2 , then find the values of cos x1 , cos x1 cos x2 .
(2) Find the area enclosed by the curves, then find the volume of the part generated by a rotation of the part of
x for the figure about the line y = 1.
667 1993 Hiroshima University

Let a > 1, 0 x . Find the volume of the solid generated by a rotation of the part bounded by two curves
4

2 sin x
1
, y=
about the x-axis.
y=
sin 2x + a
sin 2x + a
666 1987 Sapporo Medical University

Let f (x) be a function defined in 0 < x < satisfying:


2
 
(i) f
=0
6
Z x
2 cos t

dt.
(ii) f (x) tan x =

sin t
6
Find f (x).
665 1992 Japan Womens University
Z
Find lim
x| sin 2nx|dx (n = 1, 2, ).
n

664 1992 University of Fukui


Z
For a positive integer n, let In =

Find I1 + I2 + I3 + I4 .


2


|x| cos nx dx.

39

663 1978 Nagasaki University


Given are the curve y = x2 + x 2 and a curve which is obtained by tranfering the curve symmetric with respect
to the point (p, 2p). Let p change in such a way that these two curves intersects, find the maximum area of the part
bounded by these curves.
662 1987 Sophia University
In xyz space, let A be the solid generated by a rotation of the figure, enclosed by the curve y = 2 2x2 and the
x-axis about the y-axis.
(1) When the solid is cut by the plane x = a (|a| 1), find the inequality which expresses the figure of the
cross-section.
(2) Denote by L the distance between the point (a, 0, 0) and the point on the perimeter of the cross-section found
in (1), find the maximum value of L.
(3) Find the volume of the solid by a rotation of the solid A about the x-axis.
661 2010 National Defense Medical College
Consider a sequence 10.01 , 20.02 , 20.02 , 30.03 , 30.03 , 30.03 , 40.04 , 40.04 , 40.04 , 40.04 , .
(1) Find the
Z 36th term.
x2 ln x dx.

(2) Find

(3) Let A be the product of from the first term to the 36th term. How many digits does A have integer part?
If necessary, you may use the fact 2.0 < ln 8 < 2.1, 2.1 < ln 9 < 2.2, 2.30 < ln 10 < 2.31.
660

Let a, b be given positive constants.


Evaluate
Z

1
0

ln (x + a)x+a (x + b)x+b
dx.
(x + a)(x + b)

659
Evaluate

ln(x + 2)
dx.
x+1

658 1997 Kyoto University




.
Consider a parameterized curve C : x = et cos t, y = et sin t 0 t
2
(1) Find the length L of C.
(2) Find the area S of the region enclosed by the x, y axis and C.
Please solve the problem without using the formula of area for polar coordinate for Japanese High School Students
who dont study it in High School.
657
A sequence an is defined by
Find lim

an
.
n3

an+1
an

(1 + | sin x|)dx = (n + 1)2 (n = 1, 2, ), a1 = 0.

656
Find lim n
n

1
dx (n = 1, 2, ).
(1 + cos x)n

655
Find the area of the region of the points such that the total of three tangent lines can be drawn to two parabolas
y = x x2 , y = a(x x2 ) (a 2) in such a way that there existed the points of tangency in the first quadrant.
654 1997 Hokkaido University
A function f (x) defined in x 0 satisfies lim

f (x)
= 1.
x
40

Find

{f (x) f (x)}ex dx.

653 1975 Yokohama National University


Z
xp
dx (0 < p < 1).
Evaluate
2
x +1
0
652 1981 Tokyo University
Let a, b, c be positive real numbers such that b2 > ac. Evaluate
Z
dx
.
4 + 2bx2 + c
ax
0
651 1985 Tohoku University
Find
lim

2n

650 1995 Hitotsubashi University

2x





x 2 x + 1 dx.

2

Find the values of p, q, r (1 < p < q < r < 1) such that for any polynomials with degree 2, the following
equation holds:
Z

p
1

f (x) dx

f (x) dx +

f (x) dx

f (x) dx = 0.

649 2009 Tokyo Institute of Technology


p
n
(r = x2 + y 2 ),
Let fn (x, y) =
2
3
r cos r + n r
Z Z

In =

r1

fn (x, y) dxdy (n 2).

Find lim In .
n

648 2010 Kyoto University, Master Course in Mathematics


Consider a function real-valued function with C -class on R such that:
d2 f
df
(0) 6= 0.
(0) = 0,
(a) f (0) =
dx
dx2
(b) For x 6= 0, f (x) > 0.
Judge whether the following integrals (i), (ii) converge or diverge, justify your answer.
(i)
Z Z
dx1 dx2
.
f
(x
2
2
1 ) + f (x2 )
|x1 | +|x2 | 1
(ii)
Z Z

|x1 |2 +|x2 |2 +|x3 |2 1

dx1 dx2 dx3


.
f (x1 ) + f (x2 ) + f (x3 )

647
Evaluate
Z

xpx cos qx dx,

xpx sin qx dx (p > 0, p 6= 1, q N+ )

646
Evaluate
Z

a cos bx dx,

ax sin bx dx (a > 0, a 6= 1, b N+ )
41

645
Prove the following inequality.
Z

ex + ex
1
dx < e
x
e
eee

644
For a constant p such that

ex dx = 1, prove that

1
p

Z

2 Z
e cos x dx +
x

2
1
e sin x dx > .
2
x

643
Evaluate
Z

642

x
p
{(3 cos x + 4 sin x) sin2 x + 4}dx.
3
1 + sin x

Evaluate
Z

(tan2 2x) cos 2x + 2

dx.
(cos2 x) cos 2x

641
Evaluate
e

ee


ln(ln(ln x)) +

ee

1
(ln x) ln(ln x)

dx.

640
Evaluate

1
1 sin x

cos x
dx.
1 + cos x + sin x

639
Evaluate

(x + 3) xex dx.

638 1978 Chuo University


x
(x 0). Denote U the volume of the figure enclosed by the curve ,
1+x
the x axis and the line x = a, revolved around the the x axis and denote V the volume of the figure enclosed by the
curve , the y axis and th line y = b, revolved around the y axis. Whats the relation of U and V ?
Let (a, b) be a point on the curve y =

637 1978 Niigata University


For a non negative integer n, set In =

tann x dx to answer the following questions:

(1) Calculate In+2 + In .


(2) Evaluate the values of I1 , I2 and I3 .
636 1992 Tokyo University

Let a > 1 be a constant. In the xy-plane, let A(a, 0), B(a, ln a) and C be the intersection point of the curve
y = ln x and the x-axis. Denote by S1 the area of the part bounded by the x-axis, the segment BA and the curve
y = ln x.
42

(1) For 1 b a, let D(b, ln b). Find the value of b such that the area of quadrilateral ABDC is the closest to
S1 and find the area S2 .
S2
(2) Find lim

a S1
635 1975 Waseda University
Suppose that a function f (x) defined in 1 < x < 1 satisfies the following properties:
(i) f (x) is continuous.
(ii) When 1 < x < 0, f (x) < 0, f (0) = 0, and when 0 < x < 1, f (x) > 0.
(iii) f (0) = 1.
Z xp

1 + {f (t)}2 dt (1 < x < 1). If F (sin ) = c (c : constant) holds for < < , then find
Let F (x) =
2
2
0
f (x).
634 2010 Miyazaki University
Prove that :
Z

n1

(ln x) dx = (1)

 m
n
X
1
nm n!
(1)
(n = 1, 2, ).
n! + e
m!
2
m=0

633 1978 Tokyo Medical College


Let f (x) be a differentiable function. Find the value of x for which
2

{f (x)} + (e + 1)f (x) + 1 + e 2

f (t)dt 2f (x)

f (t)dt + 2
0

Z

f (t)dt
0

2

is minimized.
632 2010 Kyoto Institute of Technology
Z 1
Find lim
| sin nx|3 dx (n = 1, 2, ).
n

631
Evaluate

(x2 +



p
1
1
x4 1)
dx.
+
x2 + 1
x2 1

630
Evaluate

ln(1 + e4x )
dx.
ex

629
Evaluate

ex (1

1
dx.
+ e4x )

628 1984 Yamanashi Medical University


(1) Evaluate the following definite integrals:
Z 2
cos2 x sin x dx.
(a)
Z0 2
(b)
( 2x) cos x dx.
Z 0 2
(c)
x cos3 x dx.
0

(2) Let a be a positive constant. Find the area of the cross section cut by the plane z = sin
solid such that
x2 + y 2 + z 2 a2 , , x2 + y 2 ax, z 0,
43

of the

then find the volume of the solid.


627
Evaluate

(2 sin + 1) cos3
d.
(sin2 + 1)2

626 2010 Nara Medical University


Z 1
x ln x
dx.
Find lim
a+0 a (1 + x)3
625 2010 Kumamoto University
1
Find lim 3
t0 t

t2

ex sin

x
dx (t 6= 0).
t

624 1977 Keio University


Find the continuous function f (x) such that the following equation holds for any real number x:
Z x
sin t f (x t) dt = f (x) sin x.
0

623 1978 Shibaura Institute of Technology


Find the continuous function satisfying the following equation:
Z x
Z x
f (t) dt +
tf (x t) dt = ex 1.
0

622 2010 Nagoya Institute of Technology


For 0 < k < 2, consider two curves C1 : y = sin 2x (0 x ), C2 : y = k cos x (0 x ). Denote by S(k) the
sum of the areas of four parts enclosed by C1 , C2 and two lines x = 0, x = . Find the minimum value of S(k).
621 2010 Yokohama National University

 2
n
1X
n + (k 1)2
.
Find the limit lim
k ln
n n
n2 + k 2
k=1

620 2010 Chiba University


Let a, b be real numbers. Suppose that a function f (x) satisfies f (x) = a sin x + b cos x +
Z
{f (x)}2 dx.
the maximum value 2 for x . Find the minimum value of

f (t) cos t dt and has

619 2010 Saitama University




sin x
. Let a be the value of x for which f (x) is maximized.
0x
Consider a function f (x) =
2
2
9 + 16 sin x
Z 2
Evaluate
f (x) dx.
a

618 2010 Ibaraki University


Z
1 2
{x cos t + (1 x) sin t}2 dt.
Find the minimum value of
2
617 2010 Tohoku University
Let y = f (x) be a function of the graph of broken line connected by points (1, 0), (0, 1), (1, 4) in the x -y
Z 1
{f (x) (a|x| + b)}2 dx.
plane. Find the minimum value of
1

44

616 2010 Hirosaki University


Z 3
ln(x + 1)
dx.
Evaluate
x2
1
615 2010 Kyoto Institute of Technology

Z 2
1
1

dx.

For 0 a 2, find the minimum value of


1 + ex
1 + ea
0
614 2010 Hirosaki University School of Medicine
Z 1
3
Evaluate
{x(1 x)} 2 dx.
0

613 2010 Tokyo University of Agriculture and Technology


Find the area of the part, in the x-y plane, enclosed by the curve |ye2x 6ex 8| = (ex 2)(ex 4).
612
For f (x) =

1
(x > 0), prove the following inequality:
x
 Z t+1





1
1
1

f (x) dx
f (t) + 4f t +
+ f (t + 1)
f t+
2
6
2
t

611 2010 Kumamoto University


Let g(t) be the minimum value of f (x) = x2x in t x t + 1. Evaluate

g(t) dt.

610
Evaluate

xa 1 xax ln a
dx.
(xa 1)2

609 1972 Tokyo University of Education


Prove that for positive number t, the function F (t) =

sin x
dx always takes positive number.
1 + x2

608 2010 Gakusyuin University


For a > 0, find the minimum value of

ax2 + (a2 + 2a)x + 2a2 2a + 4


dx.
(x + a)(x + 2)

607 2010 Tokyo Institute of Technology


On the coordinate plane, Let C be the graph of y = (ln x)2 (x > 0) and for > 0, denote L() be the tangent line
of C at the point (, (ln )2 ).
(1) Draw the graph.
(2) Let n() be the number of the intersection points of C and L(). Find n().
(3) For 0 < < 1, let S() be the area of the region bounded by C, L() and the x-axis. Find S().
606 1956 Tokyo Institute of Technology
Find the area of the part bounded by two curves y =

x, x + y = 1 and the x-axis.

605 1956 Tokyo Institute of Technology


Let f (x) be a differentiable function. Find the following limit value:
 n  
ok
n
x
f (0) .
lim
f
n k
n
Especially, for f (x) = (x )(x ) find the limit value above.

45

604 1956 Tokyo Institute of Technology


n
X

kr

Let r be a positive integer. Determine the value of a for which the limit value lim k=1a
n
n
value, then find the limit value.

has a non zero finite

603 1961 Waseda University


Z 1

{ x (a + bx)}2 dx.
Find the minimum value of
0

Please solve the problem without using partial differentiation for those who dont learn it.
602 1994 Kyoto University
Prove the following inequality:
e1

n+1

(log x)n dx

(n + 1)e + 1
(n = 1, 2, )
(n + 1)(n + 2)

601
Evaluate

(tan x) 2 dx.

600
Evaluate

1
x+
1
sin x + ex e
x

dx (a > 0).

599
Evaluate

ex (sin x + cos x + cos 3x)


dx.
cos2 2x

598 1992 Tokyo University

x
(1) Find the maximum value a0 of a such that C(a) is contained to lower part of y = x, or y < x.

(2) For 0 < < , find the volume V () of the solid V obtained by revoloving the figure bounded by C(a0 ) and
2
1
about the x-axis.
three lines y = x, x = 1, x =
cos
V ().
(3) Find lim

For a constant a, denote C(a) the part x 1 of the curve y =

x2 1 +

2 0

597 1984 Tokyo University






1
1
In space given a board S shaped the equilateral triangle P QR with vertices P 1, , 0 , Q 1, , 0 ,
2
2
!
3
1
. When S is revolved about the z-axis, find the volume of the solid generated by the whole points
, 0,
R
4
4
through which S passes.
596 2009 Shimane University
Z 2
Find the minimum value of
|a sin 2x cos2 x| dx (a > 0).
0

595 2009 Kumamoto University



Z 6


4 sin x
dx.
Evaluate

3 cos x sin x

3
594

46

In the x-y plane, two variable points P, Q stay in P (2t, 2t2 + 2t), Q(t + 2, 3t + 2) at the time t. Let denote t0
as the time such that P Q = 0. When t varies in the range of 0 t t0 , find the area of the region swept by the line
segment P Q in the x-y plane.
593 1996 Osaka University
For a positive integer m, prove the following
! inequality:
r
Z 1
2
0
x + 1 x2 + 2x cos
+ 1 dx 1.
2m + 1
0
592
Prove the following inequality:

Z 4

1
3
2 1 1
ln 2 <
ln cos x dx < + ln (3 + 2 2).
4
2 4
8
2
0
591
Let a, b, c be real numbers such that a b c 1.
Z 1
3
3
Prove the following inequality:
{(1 ax)3 + (1 bx)3 + (1 cx)3 3x} dx ab + bc + ca (a + b + c) abc.
2
4
0
590
Evaluate

(cos + sin ) 2 (cos sin ) 2

d.
cos 2

589
Evaluate

dx.
2
{(2x 1) x + x + 1 + (2x + 1) x2 x + 1} x4 + x2 + 1

588
Evaluate

exe {(x + 1)ex (cos x + sin x) + cos x sin x}dx.

587
Evaluate

586

(x2 + 3x)ex (x2 3x)ex + 2


p
dx.
1 + x(ex + ex )
1

x14
dx.
+1
0
Z 1
x2n
dx (n = 1, 2, ).
(2) Evaluate In =
2
0 x +1
(1) Evaluate

x2

585 2010 Osaka University


Z ln 2
x
Evaluate
(x ln 2)e2 ln(1+e )+x+ln 2 dx.
0

584 2010 Niigata University


Z x p

Find lim
1 + e2t dt ex .
x

583 2010 Hiroshima University


Find the values of k such that the areas of the three parts bounded by the graph of y = x4 + 2x2 and the line
y = k are all equal.
582 1998 Kochi Womens University
47

Prove
inequality:
r the following

Z 2 r
3
3
1
2
+ 2<
.
1 sin x dx <
4 2
2
4
0
581 2010 Hokkaido University
For real numer c for which cx2 ln(1 + x2 ) for all real numbers x, find the value of c such that the area of the
figure bounded by two curves y = cx2 and y = ln(1 + x2 ) and two lines x = 1, x = 1 is 4.
580 2010 Hiroshima University
Let k be a positive constant number. Denote , (0 < < ) the x coordinates of the curve C : y = kx2 (x 0)
1
1
and two lines l : y = kx + , m : y = kx + . Find the minimum area of the part bounded by the curve C and two
k
k
lines l, m.
579 2010 Osaka Prefecture University
(n + 1)a + (n + 2)a + + (n + n)a
.
n
1 a + 2 a + + na

Let a be a positive real number. Find lim

578 2010 Osaka Prefecture University


Find the range of k Zfor which the following inequality holds for 0 x 1:
Z
x
x
dt
dt
p

.
k
2
3
3 + t2
(3 + t )
0
0
If necessary, you may use ln 3 = 1.10.
577 1987 Chiba University
Prove the following inequality for any integer N 4:
N
X
p2 + 2
< 5.
(p 2)4
p=4
576 2010 Fukushima Medical University
For a function f (x) = (ln x)2 + 2 ln x, let C be the curve y = f (x). Denote A(a, f (a)), B(b, f (b)) (a < b) the
points of tangency of two tangents drawn from the origin O to C and the curve C. Answer the following questions.
(1) Examine the increase and decrease, extremal value and inflection point , then draw the approximate garph of
the curve C.
(2) Find the values of a, b.
(3) Find the volume by a rotation of the figure bounded by the part from the point A to the point B and line
segments OA, OB around the y-axis.
575 2010 Kumamoto University
Z 4 x


, answer the following questions:
log4 (1 + tan t)dt 0 x
For a function f (x) =
8
x

(1) Find f (x).


(2) Find the n th term of the sequence an such that a1 = f (0), an+1 = f (an ) (n = 1, 2, 3, ).
574 2010 Tsukuba University
Let n be a positive integer. Prove that xn e1x n! for x 0.
573 2010 Tsukuba University
Find the area of
bounded by three curves:
 the figure

C1 : y = sin x 0 x <
.
2


.
C2 : y = cos x 0 x <
2


.
C3 : y = tan x 0 x <
2
572 2010 Sapporo Medical University
Z 4
(cos x)n dx.
For integer n, an is difined by an =
0

48

(1) Find a2 , a1 .
(2) Find the relation of an and an2 .
(3) Prove that a2n = bn + cn for some rational number bn , cn , then find cn for n < 0.
571 2010 Yokohama National University
Z
x sin3 x
Evaluate
dx.
2
0 sin x + 8
570 2010 Tokyo Institute of Technology
Let f (x) = 1 cos x x sin x.
(1) Show that
Z f (x) = 0 has a unique solution in 0 < x < .

(2) Let J =

|f (x)|dx. Denote by the solution in (1), express J in terms of sin .

(3) Compare the size of J defined in (2) with 2.


0

569 2010 Kyoto University


In the coordinate plane, denote by S(a) the area of the region bounded by the line passing through the point (1, 2)
with the slope a and the parabola y = x2 . When a varies in the range of 0 a 6, find the value of a such that S(a)
is minimized.
568 2010 Kyoto University
Throw n balls in to 2n boxes. Suppose each ball comes into each box with equal probability of entering in any
ln pn
.
boxes. Let pn be the probability such that any box has ball less than or equal to one. Find the limit lim
n n
567 2010 Kyoto University
Let a be a positive real numbers. In the coordinate plane denote by S the area of the figure bounded by the
curve y = sin x (0
x ) and
and denote T by the area of the figure bounded by the curves y =
 the x-axis


, y = a cos x 0 x
and the x-axis. Find the value of a such that S : T = 3 : 1.
sin x 0 x
2
2
566 2010 Kyoto University

In the coordinate space, consider the cubic with vertices O(0, 0, 0), A(1, 0, 0), B(1, 1, 0), C(0, 1, 0), D(0, 0, 1),
E(1, 0, 1), F (1, 1, 1), G(0, 1, 1). Find the volume of the solid generated by revolution of the cubic around the
diagonal OF as the axis of rotation.
565 2010 Hokkaido University
Z 1
1
Prove that f (x) =
e|tx| t(1 t)dt has maximal value at x = .
2
0
564 2010 Tokyo University
r
1
1 2
x + 2 and two distinct points
In the coordinate plane with O(0, 0), consider the function C : y = x +
2
4
P1 (x1 , y1 ), P2 (x2 , y2 ) on C.
(1) Let Hi (i = 1, 2) be the intersection points of the line passing through Pi (i = 1, 2), parallel to x axis and the
line y = x. Show that the area of OP1 H1 and OP2 H2 are equal.
(2) Let x1 < x2 . Express the area of the figure bounded by the part of x1 x x2 for C and line segments
P1 O, P2 O in terms of y1 , y2 .
563 2010 Tokyo University
Determine the pair of constant numbers a, b, c such that for a quadratic function f (x) = x2 + ax + b, the following
equation is identity
Z with respect to x:
1

f (x + 1) = c

(3x2 + 4xt)f (t) dt.

562 2010 Tokyo University


Show the following inequality for every natural number k:

49

Z 1
1
1x
1
<
dx <
.
2(k + 1)
k
+
x
2k
0
(2) Show the following inequality for every natural number m, n such that m > n:
m
X
m
mn
mn
1
< log

<
.
2(m + 1)(n + 1)
n
k
2mn
k=n+1

561
Evaluate
Z 1
1 + 2x2 + 3x4 + 4x6 + 5x8 + 6x10 + 7x12
p
dx.
(1 + x2 )(1 + x4 )(1 + x6 )
1

560 2010 School of Medicine, Keio University

1
x
, x axis and the line x = .
2
1 x2
(1) Find the volume V1 of the solid generated by rotation of K around x axis.
(2) Find the volume V2 of the solid generated by rotation of K around y axis.
Please solve question (2) without using the shell method for Japanese High School Students those who dont learn
Let K be the figure bounded by the graph of function y =

it.
559 2010 Waseda University
In xyz space, consider two points P (1, 0, 1), Q(1, 1, 0). Let S be the surface generated by rotation the line
segment P Q about x axis. Answer the following questions.
(1) Find the volume of the solid bounded by the surface S and two planes x = 1 and x = 1.
(2) Find the cross-section of the solid in (1) by the plane y = 0 to sketch the figure on the palne y = 0.
Z 1p
es es
t2 + 1 dt by substitution t =
.
(3) Evaluate the definite integral
2
0
Then use this to find the area of (2).
558 2010 Keio University
2
2
For a positive constant t, letZ, 
be the roots
quadratic
 equation x + t x 2t = 0.
  of the 
2
1
1
1
dx.
x+ 2
x+ 2 +
Find the minimum value of

557 2010 Hosei University


Find the folllowing limit:
Z 1
 
x dx
(2n + 1)
xn1 sin
2
0
lim
(n = 1, 2, ).
Z 1
 
n
n1
2
(n + 1)
x
cos
x dx
2
0
556
Prove
the following inequality:
s
Z 4
3

cos2

x cos2 (tan x) cos2 (tan(tan x)) cos2 (tan(tan(tan x)))

dx <

555 2010 Sophia University


For

1
< t < 1, find the minimum value of
e

|xex tx|dx.

554 2010 Doshisya University

d
d 2
Use
ln(2x + 4x2 + 1),
(x 4x + 1) to evaluate
dx
dx

553 2010 Doshisya University


Find the continuous function such that f (x) =

e2x
2(e 1)

50

p
4x2 + 1dx.

ey f (y)dy +

1
2

f (y)dy +
0

1
2

sin2 (y)dy.

552 2010 Tokyo Medical University


q
Find the positive value of a such that the curve C1 : x = 2y 2 +
find the equation of the tangent line of C1 at the point of tangency.

25
2

tangent to the parabola C2 : y = ax2 , then

551 2010 Tokyo Medical University


In the coordinate plane, find the area of the region bounded by the curve C : y =

x+1
and the line L : y = 1.
x2 + 1

550 2010 Shibaura Institute of Technology


Z 2
dx
.
Evaluate
(1
+
cos x)2
0
549 2010 Tokyo University of Science
n
X

k=0

Let f (x) be a function defined on [0, 1]. For n = 1, 2, 3, , a polynomial Pn (x) is defined by Pn (x) =
 
Z 1
Z 1
k
xk (1 x)nk . Prove that lim
Pn (x)dx =
f (x)dx.
Cnk f
n 0
n
0
548 2010 Ritsumeikan University
Z
X
x
x
f (x)f (x 2n)dx (n = 0, 1, 2, ).
For f (x) = e 2 cos , evaluate
2
n=0
547 2010 Kansai University
Z 1
Find the minimum value of
|ex a|dx ( < a < ).
0

546 2010 Kwansei Gakuin University


2
Z 
b
x a cos x dx.
Find the minimum value of

0
545 2010 Tokyo Womens Medical University
Z 1
2
(1) Evaluate
xex dx.
Z 01
2
x2n1 ex dx. Express In+1 in terms of In .
(2) Let In =
0

544 2010 Jyunten University


Z

x 1dx,

(x 1) dx,
(x + 1) dx.
3
3
3

Z 3
Z
(2) If a linear function f (x) satifies
3,
(x

1)f
(x)dx
=
5

(1) Evaluate

(x + 1)f (x)dx = 3 3, then we have f (x) =

A (x 1) + B (x + 1), thus we have f (x) = C .


543 1970 Waseda University

Let y be the function of x satisfying the differential equation y y = 2 sin x.


(1) Let y = ex u sin x, find the differential equation with which the function u with respect to x satisfies.
(2) If y(0) = 3, y (0) = 0, then determine y.
542 1980 Kobe University
Find continuous functions f (x), g(x) which takes positive value for any real number x, satisfying g(x) =

and {f (x)}2 {g(x)}2 = 1.


541 1986 Gakusyuin University

51

f (t) dt

Find the functions f (x), g(x) satisfying the following equations:


(1) fZ (x) = 2f (x) + 10, f (0) = 0;
x
(2)
u3 g(u)du = x4 + g(x).
0

540
Evaluate

dx.
x( x + 3 x)

539
Evaluate

sin2 x
dx.
cos3 x

538
Evaluate

x2 + 1

dx.
x x4 + 1

537

1 + sin x
dx.
Evaluate
cos x
0
Z 4
x + sin x
536 Evaluate
dx.
1
+ cos x
0
Z

535 2010 Toho University Faculty of Medicine


Let 
C be the parameterized curve for a given positive number r and 0 t ,
x = 2r(t sin t cos t)
C:
y = 2r sin2 t
When the point P moves on the curve C,
(1) Find the magnitude of acceleralation of the point P at time t.
(2) Find the length of the locus by which the point P sweeps for 0 t .
(3) Find the volume of the solid by rotation of the region bounded by the curve C and the x-axis about the x-axis.
534 1986 Oita Medical University
Z
x3
Find the indefinite integral
dx.
(x 1)3 (x 2)
533 1992 Tokyo Denki University
Let C be the circle with radius 1 centered on the origin. Fix the endpoint of the string with length 2 on the point
A(1, 0) and put the other end point P on the point P0 (1, 2). From this situation, when we twist the string around
C by moving the point P in anti clockwise with the string streched tightly, find the length of the curve that the point
P draws from sarting point P0 to reaching point A.
532 1988 Nippon University

For a curve C : y = x 9 x2 (x 0),


(1) Find the maximum value of the function.
(2) Find the area of the figure bounded by the curve C and the x-axis.
(3) Find the volume of the solid by revolution of the figure in (2) around the y-axis.
Please find the volume without using cylindrical shells for my students.
531 1967 Keio University
(1) Let f (x) be a continuous function defined on [a, b], it is known that there exists some c such that
Z b
f (x) dx = (b a)f (c) (a < c < b).
a

Explain the fact by using graph. Note that you dont need to prove the statement.
(2) Let f (x) = a0 + a1 x + a2 x2 + + an xn ,
Prove that there exists such that

52

f (sin ) = a0 +

a1
a3
an

+
+ +
, 0<< .
2
3
n+1
2

530 1996 Akita University


Answer the following questions:
Z p

2
x2 1 dx.
(1) By setting x + x 1 = t, find the indefinite integral

(2) Given two points P (p, q) (p > 1, q > 0) and A(1, 0) on the curve x2 y 2 = 1. Find the area S of the figure
bounded by two lines OA, OP and the curve in terms of p.

(3) Let S = . Express p, q in terms of .


2
Probably you consider the question (1) sounds odd, but please solve the problem without using University Mathematics for my students.
529 2006 Ochanomizu Women University
Prove that the following inequality holds for each natural number n:
2
Z 2 X
n 
61
sin kx
dx <
.
k
144
0
k=1

528 2002 Tokyo University


3

Consider a function f (x) = xex defined on any real numbers.


(1) Examine the variation and convexity of f (x) to draw the garph of f (x).
(2) For a positive number C, let D1 be the region bounded by y = f (x), the x-axis and x = C. Denote V1 (C) the
volume obtained by rotation of D1 about the x-axis. Find lim V1 (C).
C

(3) Let M be the maximum value of y = f (x) for x 0. Denote D2 the region bounded by y = f (x), the y-axis
and y = M . Find the volume V2 obtained by rotation of D2 about the y-axis.
527 2007 Osaka University of Education
Let n, m be positive integers and , be real numbers. Prove the following equalities:
Z
1
(x )(x ) dx = ( )3 ;
(1)
6
Z
n!
( )n+2 ;
(2)
(x )n (x ) dx =
(n + 2)!

Z
n!m!
(x )n (x )m dx = (1)m
(3)
( )n+m+1 .
(n
+
m + 1)!

526 1972 Tokyo Institute of Technology


For a function satisfying f (x) > 0 for a x b, let F (x) =
minimized?

|f (t) f (x)| dt. For what value of x is F (x) is

525
Let a, b be real numbers satisfying
is maximized.

(ax + b)2 dx = 1. Determine the values of a, b for which

524
EvaluateZthe following definite integral:
1
x1004 (1 x)1004 dx
0
2009
2
Z 1
.
x1004 (1 x2010 )1004 dx
0

523
Prove the following inequality:

Z 2009

2009 + 2010
1 ex2

dx < 2009 2008.


<
ln
x
2008 + 2009
2008
53

3x(ax + b) dx

522 1981 Tokyo University of Science


Compare the size of the following definite integrals:
Z 1
Z 1
Z e
3 x
3 x
A=
x e dx B =
x e dx C =
x(ln x)3 dx.
0

521 1965 Tokyo Instutitute of Technology


Let t be a positive number. Draw two tangent lines from the point (t, 1) to the parabola y = x2 . Denote S(t)
S(t)
the area bounded by the tangents line and the parabola. Find the minimum value of .
t
520
Let a, b, c be postive constants.
Z 1
2a + 3bx + 4cx2

Evaluate
dx.
2
0 2 a + bx + cx
519
Evaluate

dx.
1 + x3

518
Evaluate

cos x
 dx.
cos x 8

517 1967 Kyoto Institite of Technology


Find the values of a, b such that the area bounded by the curve y = cos x 0 x
devided into three equal parts by the curves y = a sin x, y = b sin x (a > b).


, the x-axis and y-axis are

516 1989 Kansai Gakuin University


1

(0 < x < ).
sin x 1 cos x
(1) Find the local minimum value of f (x).
Z 2
3
(2) Evaluate
f (x) dx.
Let f (x) =

515 1983 Ritsumeikan University


Find the maximum and minimum values of

(a sin x + b cos x)3 dx for |a| 1, |b| 1.

Note that you are not allowed to solve in using partial differentiation here.
514 1976 Kanazawa University
Prove the following inequalities:


;
(1) x sin x tan x x 0 x <
2 

Z x
1
sin 2x 

(2)
cos(tan t t) dt sin(sin x) +
x
0x
.
2
2
3
0
513 1968 Kyoto Institute of Technology
Find the constants a, b, c such that a function f (x) = a sin x + b cos x + c satisfies the following equation for any
real numbers x:
Z 2
(sin x + cos t)f (t) dt = f (x).
5 sin x + 3 cos x + 1 +
0

512 1968 Kyusyu University


Z n

1 sin t dt (n = 1, 2, ).
Evaluate
0

54

511 1986 Musashi Institute of Technology


Suppose Zthat f (x), g(x) are differential functions
Z and their derivatives are continuous. Find f (x), g(x) such that
x
1

{f (t) + g(t)} dt g(x) = sin x


{f (t) g (t)} dt.
f (x) =
2
0
0
510 1977 Yokahama National University
Z 2
(1) Evaluate
(x cos x + sin2 x) sin x dx.
0Z
x
et sin(x t) dt, find f (x) + f (x).
(2) For f (x) =
0

509
Evaluate

tan x
dx.
1 + sin x

508
Compare the size of the definite integrals:
Z 4
Z
Z 4
2008
2008
2009
2009
x
tan
x dx,
x
tan
x dx,
0

x2010 tan2010 x dx.

507
Evaluate

e2009

1
x

1 ln x
1+
x
ln x ln ln(ln
x)

dx.

506
Let a, b be the real numbers such that 0 a b 1. Find the minimum value of

|(x a)(x b)| dx.

505

In the xyz space with the origin O, given a cuboid K : |x| 3, |y| 3, 0 z 2 and the plane : z = 2.
Draw the perpendicular P H from P to the plane. Find the volume of the solid formed by all points of P which are
included in K such that OP P H.
504
Let a, b are positive constants. Determin the value of a positive number m such that the areas of four parts of the
region bounded by two parabolas y = ax2 b, y = ax2 + b and the line y = mx have equal area.
503 1988 Tohoku Institute of Technology
Prove the Zfollowing inequality:
1
dx
2+e
2
<
1 <
x
1
+
xe
2(1
+ e)
2
2+e
0
502 2009 National Defence Medical College

(1) For 0 < x < 1, prove that ( 2 1)x + 1 < x + 1 < 2.


R1
x 1 x2 dx
(2) Find lim a
.
3
a10
(1 a) 2
501 2009 Tokyo Institute of Technology
Find the volume of the uion A B C of the three subsets A, B, C in xyz space such that:
A = {(x, y, z) | |x| 1, y 2 + z 2 1}
B = {(x, y, z) | |y| 1, z 2 + x2 1}

55

C = {(x, y, z) | |z| 1, x2 + y 2 1}
500
Let a, b, c be positive real numbers. Prove the following inequality.
Z

xa+b+c1 [2(a + b + c) + (c + 2a)xab + (a + 2b)xbc + (b + 2c)xca + (2a + b)xac + (2b + c)xba + (2c + a)xcb ]
(xa + xb )(xb + xc )(xc + xa )
a + b + c.
499
Evaluate

( 2009 cos x +

sin x +

2009

tan x) dx.

2009

498
Let f (x) be a continuous function defined in the interval 0 x 1.
Z 1
Z
1 1
Prove that
xf (x)f (1 x) dx
{f (x)2 + f (1 x)2 } dx.
4 0
0
497 1997 Kyoto University
Consider a parameterized curve C : x = et cos t, y = et sin t 0 t
(1) Find the length L of C.
(2) Find the area S of the region bounded by C, the x axis and y axis.
Z b
1
You may not use the formula
r()2 d here.
2
a

496
a2

1
dx (a > 0).
+ a2
1
You may not use tan1 x or Complex Integral here.

Evaluate

x2

495 2009 Okayama Prefectural University


Evaluate the following definite integrals:
Z 21
x2

(1)
dx;
2
1

x
0
Z 1
1x
(2)
dx;
(1 + x2 )2
Z0 7
dx

.
(3)
3
1
+
1+x
1
Solution.
(1) Put x = sin = dx = cos d



Z 21
Z 6
Z
sin 2 6
1 6
1
x2
3

=
dx =

sin d =
1 cos 2 d =
.
=
2
2
2
2
12
8
1x
0
0
0
0
(2) Put x = tan = dx = sec2 d
Z 4
Z 1
Z
Z
1 4
1 8

1x
1 tan
dx =
d =
(cos 2 + 1 sin 2) d =
2 d = .
=
2
2
2
sec
2 0
2 0
8
0 (1 + x )
0
(3) Put 1 + x = t3 = dx = 3t2 dt
 2
2
Z 7
Z 2 2
Z 2
dx
1
t
t

= 3 ln 3 .
=
3
dt
=
3
t

1
+
dt
=
3

t
+
ln(1
+
t)
3
1+t
2
1+x
1 1 +
0 1+t
0
0
494 1988 Nippon University
Suppose the curve C : y = ax3 + 4x (a 6= 0) has a common tangent line at the point P with the hyperbola xy = 1
in the first quadrant.
56

(1) Find the value of a and the coordinate of the point P .


(2) Find the volume formed by the revolution of the solid of the figure bounded by the line segment OP and the
curve C about the axis l.
493 2008 Shibaura Institute of Technology


In the x y plane, let l be the tangent line at the point A a2 , 23 b on the ellipse
Let denote S be the area of the figure bounded by l, the x axis and the ellipse.
(1) Find the equation of l.
(2) Express S in terms of a, b.
(3) Find the maximum value of S with the constraint a2 + 3b2 = 4.

x2
a2

y2
b2

= 1 (0 < b < 1 < a).

492 1978 Gakuin University Aoyama


Find the volume formed by the revolution of the region satisfying 0 y (x p)(q x) (0 < p < q) in the
coordinate plane about the y -axis.
Z b
2x|f (x)| dx (a < b) here.
You are not allowed to use the formula: V =
a

491 1964 Muroran Institute of Technology


Let f (x) = sin 3x + cos x, g(x) = cos 3x + sin x.
Z 2
(1) Evaluate
{f (x)2 + g(x)2 } dx.
0

(2) Find the area of the region bounded by two curves y = f (x) and y = g(x) (0 x ).
490
For a positive
realnumber a > 1, provethe following inequality:


Z 1
ln a
ln(ln a + 1)
x
1
1
1
<
1
.
dx <
x
a1
a1
ln a
ln a
0 a
489 1972 Hitotsubashi University
Find the following
limit:


Z 1
x3
x2n
x2
dx.
+
+ +
lim
|x| 1 + x +
n 1
2
3
2n
488 1972 Hiroshima University
For 0 x <

prove
2,Z
1

x + ln(cos x) +

the following inequality:


t

dt .
1 + t2
4

487 1978 Tokyo Institute of Technology


Suppose two functions f (x) = x4 x, g(x) = ax3 + bx2 + cx + d satisfy f (1) = g(1), f (1) = g(1).
Z 1
Find the values of a, b, c, d such that
(f (x) g(x))2 dx is minimal.
1

486 1990 Tokyo University of Agriculture and Technology


Let H be the piont of midpoint of the cord P Q that is on the circle centered the origin O with radius 1. Suppose
the length of the cord P Q is 2 sin 2t for the angle t (0 t ) that is formed by half-ray OH and the positive direction
of the x axis. Answer the following questions.
(1) Express the coordiante of H in terms of t.
(2) When t moves in the range of 0 t , find the minimum value of x coordinate of H.

(3) When t moves in the range of 0 t , find the area S of the region bounded by the curve drawn by the
2
point H and the x axis and the y axis.
485 1980 Tokyo University
In x y palne, for the origin O, given an isosceles triangle OAB with AO = AB such that A is on the first
quadrant and B is on the x axis. Denote the area by s. Find the area of the common part of the traingle and the
region expressed by the inequality xy 1 to give the area as the function of s.
57

484 1983 Chiba University


Let C : y = ln x. For natural number n, denote An the area of the region bounded by the line passing through two
points (n, ln n), (n + 1, ln(n + 1)) on C, and let Bn be the area of the region bounded by the tangent line at (n, ln n)
on C and the line x = n + 1. Set g(x) = ln(x + 1) ln x.
(1) Express An , Bn in terms of n, g(n).
(2) Use An > 0, Bn > 0 to find the limit lim n{1 ng(n)}.
n

483 2009 Ryukyu University


Let n 2 be natural number. Answer
Z n the following questions:
x ln x dx.
(1) Evaluate the definite integral
1

(2) Prove the following inequality.


n
X
1 2
1 2
1
1
n ln n (n 1) <
k ln k < n2 ln n (n2 1) + n ln n.
2
4
2
4
k=1

(3) Find lim (11 22 33 nn ) n2 ln n .


n

482 2009 Miyazaki University


1
n n

Let n be natural number. Find the limit value of lim


2
n
1
+ + +
.
2
5
n2 + 1

481
1
(n 2).
(a + b cos )n
a
2n 3
1
n2
b
1
sin
Prove that : In = 2

In1 2

In2 2

.
a b2 n 1
a b2 n 1
a b2 n 1 (a + b cos )n1

For real numbers a, b such that |a| 6= |b|, let In =

480
Let a, b be positive real numbers.Prove that
Z 2ab p

p



3b(2a b) + 2(a 2b)x x2 3a(2b a) + 2(2a b)x x2 dx (a2 + b2 ).
3
a2b
479 1959 Tokyo University

Let a, b be
constants. Find the minimum value of the definite integral:
Z real

I(a, b) =
(1 a sin x b sin 2x)2 dx.
0

478
Evaluate

{(x sin x + 2 cos x) tan x + (x cos x 2 sin x) cot x} dx.

477
Suppose that P1 (x) =

d3
d2
d 2
(x 1), P2 (x) = 2 (x2 1)2 , P3 (x) = 3 (x2 1)3 .
dx
dx
dx
Z
1

Find all possible values for which

Pk (x)Pl (x) dx (k = 1, 2, 3, l = 1, 2, 3) can be valued.

476 1968 Tohoku University


Suppose a parabola with the axis as the y axis, concave up and touches the graph y = 1 |x|. Find the equation
of the parabola such that the area of the region surrounded by the parabola and the x axis is maximal.
475
For a positive constant number t, let denote D the region surrounded by the curve y = ex , the line x = t, the x
axis and the y axis. Let Vx , Vy be the volumes of the solid obtained by rotating D about the x axis and the y axis
respectively. Compare the size of Vx , Vy .
474 1994 Nippon Medical School
58

Calculate
the following indefinite integrals:
Z
3x + 4
dx;
(1)
2
Z x + 3x + 2

(2)

(3)

(4)

sin 2x cos 2x cos 4x dx;

xex dx;

5x dx.

473 1976 Tokyo University


For nonzero real numbers r, l and the positive constant number c, consider the curve on the xy plane :

(0 x r)
x
2
y= r
.
(r x l + r)

2
(x l 2r) (l + r x l + 2r)

Denote V the volume of the solid by revolvering the curve about the x axis. Let r, l vary in such a way that
r2 + l = c. Find the values of r, l which gives the maxmimum volume.
472 1976 Hitotsubashi University
Given a line segment P Q moving on the parabola y = x2 with end points on the parabola. The area of the figure
4
surrounded by P Q and the parabola is always equal to . Find the equation of the locus of the mid point M of P Q.
3
471
Evaluate

1 x(ex 1)
dx.
x(1 + xex ln x)

470 1995 Kwansai University


Determin integers m, n (m > n > 0) for which the area of the region bounded by the curve y = x2 x and the
37
.
lines y = mx, y = nx is
6
469
Evaluate

t
dt.
(1 + t2 )(1 + 2t t2 )

468
Evaluate

1
2

12

dx.
{(2x + 1) x2 x + 1 + (2x 1) x2 + x + 1} x4 + x2 + 1

467 1988 Nihon University

Let the curve C : y = x 9 x2 (x 0).


(1) Find the maximum value of y.
(2) Find the area of the figure bounded by the curve C and the x axis.
(3) Find the volume of the solid generated by rotation of the figure about the y axis.
466 2009 Osaka University
2

1
+y 2 = 1.
For n = 1, 2, 3, , let (pn , qn ) (pn > 0, qn > 0) be the point of intersection of y = ln(nx) and x
n
(e 1)2
(1) Show that 1 qn2
to find lim qn .
n
n2
Z p
n

ln(nx) dx.

(2) Find lim n


n

1
n

465 2009 University of Electro-communication

59

Compute

x2n+1 ex dx (n = 1, 2, ) , then use this result, prove that

X
1
= e.
n!
n=0

464
Evaluate

(1 + 2x2 ) ln x

dx.
1 + x2

463
Evaluate

e cos2 x sin x
dx.
cos3 x

462
Evaluate

(1 x + x2 ) cos ln(x +

1 + x2 ) 1 + x2 sin ln(x + 1 + x2 )

(1 + x2 ) 2

dx.

461 2009 Ohita University

1
dx (n = 1, 2, ).
1 + xn
0
(1) Find I1 , I2 .
(2) Find lim In .

Let In =

460 2009 Kumamoto University



Z 6


dx.
4 sin x

3 cos x sin x

459 2009 Kyusyu University


n
Z 1 
1
dt (x 0, n = 1, 2, ).
ln
Find lim
x ex
t
458 2009 Shimane University
Let S(t) be the area of the traingle OAB with O(0, 0, 0), A(2, 2, 1), B(t, 1, 1 + t). Evaluate

457 2009 Mie University


Z 2
1
d.
Evaluate

1
+
sin

cos
3
456 2009 Aichi University of Education

1
1
Find lim
.
+
+ +
n n
(n + 1)
(n + 2)
(n + n)

sin

sin
sin
4n
4n
4n
455 2009 Chiba University

Z 33 
1
1

dx.
(1) Evaluate

3
3
x2
1 + x2
1

(2) Find the positive real numbers a, b such that for t > 1, lim

Z

454 2009 Sinsyu University


Let a be positive constant number. Evaluate

x2 cos x + ex
dx.
ex + 1

60

dx atb
3
1 + x2

converges.

S(t)2 ln t dt.

453 2009 Saitama University


Z 2
Find the minimum value of
|x sin t cos t| dt (x > 0).
0

452 2009 Gunma University


Let a, b are postive constant
numbers.

(1) Differentiate ln(x + x2 + a) (x > 0).


Z b
4b2
1

(2) For a =
dx.
,
evaluate
2+a
(e e1 )2
x
0
451 2009 Yamagata University


n
X
ka
Find lim
ln 1 + a+1 .
n
n
k=1

450 2009 Hirosaki University


Let a, b be postive real numbers. Find lim

n
X

k=1

n
.
(k + an)(k + bn)

449 2009 Hirosaki University


n Z
X
Evaluate
(sin x cos kx)2 dx.
k=1

448
Evaluate

ln 2

e3x

2ex + 1
dx.
+ 2e2x + ex ex

447
Evaluate

x2
dx.
(1 + x tan x)(x tan x) cos2 x

446
Evaluate

445
Evaluate

6x + 5 x
p
dx.
x+ x
(1 2x)ex + (1 + 2x)ex
dx.
(ex + ex )3

444
Evaluate

sin x + cos x
ln (2 + sin 2x) dx.
1 sin 2x

443
Evaluate

e2

(e






) cos e x + e x + 4 + (e x + e x ) cos e x e x + 4

dx.
x

442
Evaluate

cos sin
d.
(1 + cos )(1 + sin )

441

61

Evaluate

(x2 ln x 1)ex
dx.
x

440 1985 Tokyo University of Science


Z a
ex
dx.
For a > 1, find lim
n 0 1 + xn
439 2009 Saitama University
Find the volume of the solid defined by the inequality x2 + y 2 + ln(1 + z 2) ln 2. Note that you may not directively
use double integral here for Japanese high school students who dont study it.
438
Evaluate

2+1

21

x4 + x2 + 2
dx.
(x2 + 1)2

437
Evaluate

1
q
p

dx.
p
x 1+ x 1+ 1+ x

436 1987 Kwansai University

Find the minimum area bounded by the graphs of y = x2 and y = kx(x2 k) (k > 0).
435
Evaluate

dx.
(sin x + cos x + 2 sin x cos x) sin x cos x

434
Evaluate

x e2x
dx.
x2 e2x

433
Evaluate

(sin x)cos x
dx.
(cos x)sin x + (sin x)cos x

432 2009 Hokkaido University


Z 1
Define the function f (t) =
(|ex t| + |e2x t|)dx. Find the minimum value of f (t) for 1 t e.
0

431 2009 The University of Electro-Communications


Z 1 p

Consider the function f () =


| 1 x2 sin |dx in the interval of 0 .
2
0
(1) Find the maximum and minimum values of f ().
Z 2
(2) Evaluate
f () d.
0

430 2009 Hokkaido University


Z
For a natural number n, let an =
(1) Find a1 .
(2) Express an+1 in terms of an .
(3) Find lim an .
n
n
X
(1)k+1
.
(4) Find lim
n
2k 1

(tan x)2n dx. Answer the following questions.

k=1

62

429 2009 Kyoto University


Find the length of the curve expressed by the polar equation: r = 1 + cos (0 ).
428 2009 Kyoto University
Let f (x) be a polynomial and C be a real number. Find the f (x) and C such that

f (y)dy +

x2 + C.

(x+ y)2 f (y)dy =

427 2009 Tokyo University


Let a be a positive real number, in Euclidean space, consider the two disks: D1 = {(x, y, z)|x2 + y 2 1, z = a},
D2 = {(x, y, z)|x2 + y 2 1, z = a}. Let D1 overlap to D2 by rotating D1 about the y axis by 180 . Note that the
rotational direction is supposed to be the direction such that we would lean the postive part of the z axis to into the
direction of the postive part of x axis. Let denote E the part in which D1 passes while the rotation, let denote V (a)
the volume of E and let W (a) be the volume of common part of E and {(x, y, z)|x 0}.
(1) Find W (a).
(2) Find lim V (a).
a

426 2009 Tokyo University


Consider the polynomial f (x) = ax2 + bx + c, with degree less than or equal to 2.
When f varies with subject to the constrain f (0) = 0, f (2) = 2, find the minimum value of S =

2
0

|f (x)| dx.

425 2009 Keio University


The coordinate of P at time t, moving on a plane, is expressed by x = f (t) = cos 2t + t sin 2t, y = g(t) =
sin 2t t cos 2t.

(1) Find the acceleration vector


of P at time t .

(2) Let l denote the line passing through the point P for the time t, which is parallel to the acceleration vector

at the time. Prove that l always touches to the unit circle with center the origin, then find the point of tangency Q.

(3) Prove that f (t) decreases in the interval 0 t .


2

(4) When t varies in the range t , find the area S of the figure formed by moving the line segment P Q.
4
2
424 2009 Aoyama Gakuin University
Let n be positive integer. For n = 1, 2, 3, n, let denote Sk be the area of AOBk such that AOBk =
n
1 X
k
, OA = 1, OBk = k. Find the limit lim 2
Sk .
n n
2n
k=1

423

2009 Keio University

Let f (x) = x2 + 3 and y = g(x) be the equation of the line with the slope a, which pass through the point (0, f (0))
Z 1
. Find the maximum and minimum values of I(a) = 3
|f (x) g(x)| dx.
1

422 2009 Waseda University


There are 10 cards, labeled from 1 to 10. Three
Z a cards denoted by a, b, c (a > b > c) are drawn from the cards at
the same time. Find the probability such that
(x2 2bx + 3c) dx = 0.
0

421 2009 Waseda University


Let f (x) = e(p+1)x ex for real number p > 0. Answer the following questions.
(1) Find the value of x = sp for which f (x) is minimal and draw the graph of y = f (x).
Z t+1
(2) Let g(t) =
f (x)etx dx. Find the value of t = tp for which g(t) is minimal.
t

(3) Use the fact 1 +

ep 1
p
p

1 + + p2 (0 < p 1) to find the limit lim (tp sp )


p0
2
p
2

420 2009 Keio University

63

Let K be the figure bounded by the curve y = ex and 3 lines x = 0, x = 1, y = 0 in the xy plane.
(1) Find the volume of the solid formed by revolving K about the x axis.
(2) Find the volume of the solid formed by revolving K about the y axis.
419 2009 Keio University
In the xy plane, the line l touches to 2 parabolas y = x2 + ax, y = x2 2ax, where a is positive constant.
(1) Find the equation of l.
(2) Find the area S bounded by the parabolas and the tangent line l.
418 2009 Kansai University Rikkyo University
Z
e2x
dx.
(1) Calculate
x
Z 1 1 + e3
2x
dx.
(2) Evaluate
2
0 1+x
417 2009 Tokyo University of Science
Z x
Z 1
x3
+1x
g(t) dt, g(x) = x
f (t) dt. Let l1 , l2 be the tangent
2
0
0
lines of the curve y = f (x), which pass through the point (a, g(a)) on the curve y = g(x). Find the minimum area of
the figure bounded by the tangent tlines l1 , l2 and the curve y = f (x).
The functions f (x), g(x) satisfy that f (x) =

416 2009 Kansai University


Answer the following questions.
(1) 0 < x 2, prove that | sin x| < x.
(2) Let f1 (x) = sin x , a be the constant such that 0 < a 2.
Z x+a
1
fn (t) dt (n = 1, 2, 3, ). Find f2 (x).
Define fn+1 (x) =
2a xa
(3) Find fn (x) for all n.

X
(4) For a given x, find
fn (x).
n=1

415 2009 Doshisya University


For a function f (x) = 6x(1 x), suppose that positive constant c and a linear function g(x) = ax + b (a, b :
Z 1
Z 1
Z 1
constants a > 0) satisfy the following 3 conditions: c2
f (x) dx = 1,
f (x){g(x)}2 dx = 1,
f (x)g(x) dx = 0.
0

Answer the following questions.


(1) Find the constants a, b, c.

Z 1
(2) For natural number n, let In =
xn ex dx. Express In+1 in terms of In . Then evaluate I1 , I2 , I3 .
0
Z 1
Z 1
x
(3) Evaluate the definite integrals
e f (x) dx and
ex f (x)g(x) dx.
0 Z
0
1
(4) For real numbers s, t, define J =
{ex cs tg(x)}2 dx. Find the constants A, B, C, D, E by setting
0

J = As2 + Bst + Ct2 + Et + F . (You dont need to find the constant F ).


(5) Find the values of s, t for which J is minimal.
414 2009 Jikei University
Evaluate

2(2+ 3)

(x2

16
dx.
+ 4)2

413 2009 Doshisya University


Find the maximum and minimum value of F (x) =

1
x+
2

412 2009 Doshisya University

64

(t x) sin t dt for 0 x .

Let the definite integral In =


(1)
(2)
(3)
(4)

Find I0 , I1 , I2 .
Find I1 .
Express In+2 in terms of In .
Find I3 , I2 , I3 .

dx
(n = 0, 1, 2, ).
(cos x)n

Z
p
2
x + 1 dx,

dx
dx in using the avobe results.
+ 1)2
0
0
You are not allowed to use the formula of integral for x2 + 1 directively here.
(5) Evaluate the definite integrals

(x2

411 2009 Sophia University


Find the area bounded by y = x2 |x2 1| + |2|x| 2| + 2|x| 7 and the x axis.
410
Evaluate

1
cos

1 + sin
d.
cos

409
Evaluate

x + x2 + 1
dx.
x2 + 1

408
Evaluate

{(1 + x)ex + (1 x)ex } ln x dx.

407
Evaluate

(x + 3) xex dx.

406 2009 Ritsumeikan University


Z 2
Find lim
x| cos(2n + 1)x| dx.
n

405
Z 2



sin x


(x
cos
x
+
1)e
dx
0

.
Calculate Z

2

cos x
(x sin x 1)e
dx

0

404

Evaluate

sin nx
dx (n = 0, 1, 2, ).
(1 + 2009x) sin x

403
Evaluate

2e2x + xex + 3ex + 1


dx.
(ex + 1)2 (ex + x + 1)2

402 1969 Nagoya City University


Consider a right circular cylinder with radius r of the base, hight h. Find the volume of the solid by revolving the
cylinder about a diameter of the base.
401 2009 Tokyo University
For real number a with |a| > 1, evaluate

d
.
(a + cos )2

65

400 2002 Kyoto University

(1) A function is defined f (x) = ln(x + 1 + x2 ) for x 0. Find f (x).


(2) Find the arc length of the part 0 for the curve defined by the polar equation: r = ( 0).
399
Evaluate

21

1 + x2
ln
1 x2

1+x
1x

dx.

398 2007 Hokkaido University


In xyz space, find the volume of the solid expressed by the sytem of inequality:
0 x 1, 0 y 1, 0 z 1
x2 + y 2 + z 2 2xy 1 0.
397
In xy plane, find the minimum volume of the solid by rotating the region boubded by the parabola y = x2 + ax + b
passing through the point (1, 1) and the x axis about the x axis.
396
Evaluate

2008


3x2 8028x + 20072 +

1
2008

dx.

395 1997 Tsuda Women University


3 points O(0, 0), P (a, a2 ), Q(b, b2 ) (a > 0, b > 0) are on the parabpla y = x2 . Let S1 be the area bounded by
the line P Q and the parabola and let S2 be the area of the triangle OP Q.
S1
.
Find the minimum value of
S2
394
1
Find lim
x0 x

(t sin 2006t + 2007t + 1004) dt.

393

Let V (a) be the volume of the solid obtained by revolvong the region bounded by the curve y = x x sin ax (0

x ) and x axis around the x axis.


a
 
1
.
For a > 0, find the minimum value of V (a) + V
a
392 2008 Tokyo Womens Medical University
Z 1
|x|
dx.
Evaluate
1
+
ex
1
391 1992 Tokyo Gakugei University
Z 2
Evaluate
sin |2x a| dx for a real number a [0, ].
0

390 1991 Sophia University


Find the polynomials f (x), g(x) such that:
Z 1
1 2 tf (t) 6g(t)

dt = f (x) g(x) + x;
0
1 t2
Z 21
8f (t) 5g (t)
6

dt = 2f (x) 3g(x) x2 + 2x.


0
1 t2
389 1985 Osaka City University

66

Find the values of t [0, 2] for which

2t

2x dx is maximal or minimal.

t3

388 2004 Tsukuba University


For f (x) = sin3 x, let p(x) be quadratic polynomial. Evaluate

p(x)f (x) dx.

387 2008 National Defence Medical College


Let l1 , l2 be the tangent and nomal line respectively at the point (p, ln(p + 1)) on the curve C : y = ln(x + 1).
T2
Denote by Ti (i = 1, 2) the areas bounded by li (i = 1, 2), C and the y axis respectively. Find the limit lim
.
p0 T1
386
For a = 2007 2009, evaluate
Z
1
6x + 5a x
p
dx.
0 4 x+a x
385

Evaluate

2
4

(2 sin x + x cos x) dx.

384
Evaluate

(sin x + cos x)

ex
dx.
sin x

383
For a positive integer m, evaluate

cosm x sin 2mx dx.

382 1992 Tokyo Medical and Dental University


For a > 0, b 0, compare:
Z b+1
1
dx
1

,
,
.
x+a
a+b
a+b+1
b
n
X
1

.
(2) Find lim
2
n
n +k
k=1
381 1993 Musashi Institute of Technogy
A functionf (x)is defined as follows for x 0:
n
sin x (n x < (n + 1)) (n = 0, 1, 2, ).
f (x) = sin
Z 4
100

Evaluate

f (x) dx.

380 1997 Hukui Medical College


Z 1
Find
Cnk (1 + x)nk (1 x)k dx (k = 0, 1, 2, n).
1

379 1986 Yamaguchi University


Let , r be real numbers such that r > 1, r 6= 3, r =
6 4.
n
X
n
r3
Find the values of , r such that lim
=
.
r
n
(n + k)
(r 1)(r 4)
k=1

378 1985 Kyoto Institute of Technology

67

Evaluate

x| sin nx| dx (n = 1, 2, ).

377 1982 Toho University


The line y = mx has 3 intersection points with the curve y = |x(x 1)|. Find the value of m such that the area
of the 2 regions bounded by the line and the curve are equal.
376
Z

Evaluate

1 + 2 sin x sin2 x
dx.
(1 + x) cos4 x

375
Prove the following inequality:
Z 2
n+5
1
1
dx <
<
(n = 2, 3, . . .).
n
n
(1
+
cos
x)
n(n
+ 1)
0
374 1996 Osaka University
Let n 2 be positive integers.

n
X

(1) Prove that n ln n n + 1 <

k=1

(2) Find lim (n!) n ln n .

ln k < (n + 1) ln n n + 1.

373
Evaluate
Z 2
n
xo
x
dx.
ex cos (sin x) cos2 + sin (sin x) sin2
2
2
0
372
Evaluate
Z 1
x
x+ex+e
ex+e
dx.
0

371
Calculate
Z 1
1

1
dx.
(1 + ex )(1 + x2 )

370
Calculate
Z /3
cos2 x + 1

dx.
cos x cos x
0
369
Calculate
Z eeee
eee

1
dx.
x ln x ln(ln x) ln(ln(ln x))

368 1995 Hosei University


Z
For a real number a, evaluate

|x(x a2 )| dx.

367 1990 Kogakuin University


Z 2
Z 1
f (t) dt.
xf (t) dt +
Let f (x) = x2 +
0

68

Evaluate

f (x + sin x) dx.

366 1989 Oita University


(1) Determine the constant numbers a, b, c, p, q, r, s such that the following equation is equality:
4x = {a(x + 1)2 + b(x + 1) + c}(x2 + 1)2 + {(px + q)(x2 + 1) + (rx + s)}(x + 1)3 .
(2) Evaluate the following definite integrals:
Z 1
Z 1
Z 1
dx
dx
dx
(c)
(b)
(a)
2
x+1
(x + 1)
(x + 1)3
Z0 1
Z0 1
Z0 1
x
dx
x
(d)
dx
dx (e)
(f )
2+1
2+1
2 + 1)2
x
x
(x
0
0
0
Z 1
Z 1
dx
4x
(g)
(h)
dx.
2
2
3
2
2
0 (x + 1)
0 (x + 1) (x + 1)
365
Let a, b be postive constant numbers.
Z 2
a cos x b sin x
dx.
Evaluate
b sin x + a cos x
0
364 2008 Hirosaki University
Z 1
Evaluate
x2 (x 1)2 e2x dx.
0

363 2008 Shibaura Institute of Technology


Z t
enx dx (n = 0, 1, 2, 3).
For t 0, let an =
0

(1) Show that a3 3a2 + 3a1 a0 0.


(2) Show that et a0 + (et 1)a1 a2 0.
362 2008 Miyazaki University
Evaluate the following definite integrals.
Z 2
x cos x dx.
(1)
Z0 2
1
(2)
dx.
x(x
+ 1)
1
Z 1
1
(3)
dx.
1 + ex
Z0 4
cos 2x cos 3x dx.
(4)
0

361 2008 Hiroshima City University


FindZ the following indefinite integrals:
sin x
dx.
(1)
2
Z 3 + sin x
(2)

e2x+e dx.

360
Evaluate

(1 + x) 2 (1 x) 2 dx.

359
Evaluate

2x4 x3 2x2 + 1

dx.
x3 x + 1 x2

358 1982 Toyama Medical and Pharmaceutical University

69

na
Let a be a postive constant number. Given a positive integer n, take an integer m such that m
< m + 1.


Z m 
1
t
Find lim
| sin t| dt.
a
n n 0
n
357 1982 Miyagi University of Education
1+a

For 0 < a < 1, let S(a) is the area of the figure bounded by three curves y = ex , y = e 1a x and y = e2x . Find
S(a)
lim
.
a0 a
356 1982 Hirosaki University
d
A continuous function f (x) satisfies that
dx

X
such that
f (2n ) = 1.

Z

f (x + t) dt

= 0. Find the necessary and sufficient condition

n=1

355 2008 Niigata University


For a positive number n, find lim n

xn ln x dx.

e n

354 2008 Yokohama City University


Z 2 p
Evaluate
x4 4 x2 dx.
2

353 2008 Kumamoto University


1 2
x and the point F (0, 1). For the origin O, take n points on the parabola
4
k
C, A1 (x1 , y1 ), A2 (x2 , y2 ), , An (xn , yn ) such that xk > 0 and OF Ak =
(k = 1, 2, 3, , n). Find
2n
n
X
1
F Ak .
lim
n n
Consider a parabola C : y =

k=1

352 2008 Kochi University


Prove the following inequality:
Z 2
sin x
1 lim
dx < 3.
n 1 x( x)
n
351 2008 Ehime University
Find the positive value of k for which

| cos x kx| dx is minimized.

350 2008 Otaru University of Commerse


Z 1
5
Evaluate
(1 x2 ) 2 dx.
0

349 2008 Saga University


Z

n+3
n

For real numbers a, b (ab 6= 0), evaluate

(a sin x + b cos x)6 dx.

348 2008 Toyama University


Find lim

n+1
n

1
n2

n+2
n

2
n2

3
n2

n+n
n

347 2008 Ibaraki University

70

n
n2

(n = 1, 2, 3, ).

For real positive numbers a, find the minimum value of

x
a

2
+ a sin x dx.

346 1981 Shibaura Institute of Technology


Suppose that two curves C1 : y = x3 x, C2 : y = (x a)3 (x a) have two intersection points.
Find the maximum area of the figure bounded by C1 , C2 .
345 1969 Kwansai Gakuin University
Given a continuous function f (x) such that

f (x) dx = 0.

Let S(x) = A0 + A1 cos x + B1 sin x, find constant numbers A0 , A1 and B1 for which

minimized.

{f (x) S(x)}2 dx is

344 1978 Putnam A3


Find the value of k (0 < k < 5) such that

2 + 4x +

3x2

xk
dx is minimal.
+ 5x3 + 3x4 + 4x5 + 2x6

343 1964 Putnam A2


Find all continuous positive functions f (x), for 0 x 1, such that:
Z 1
f (x) dx = 1
Z0 1
xf (x) dx =
Z0 1
x2 f (x) dx = 2
0

where is given real number.

342 1985 Gakusyuin University


Prove the following inequality:
Z 1
2 Z 1
1
(x2 + px + q) dx
(x2 + px + q)2 dx
.
180
0
0
341 1988 Kwansai Gakuin University
Let c be a constant number. Find the real value of x such that

|t x| dt =

|t c| dt.

340 1999 Shibaura Institute of Technology


Find the continuous function f (x) such that xf (x)

339 1963 Tokyo Metropolitan University

p
f (t) dt = x + ln( x2 + 1 x) with f (0) = ln 2.

Find the minimum area of the part bounded by the parabola y = a3 x2 a4 x (a > 0) and the line y = x.
338 2002 Tohoku University
Given a parameterized curve C : x = et et , y = e3t + e3t .
Find the area bounded by the curve C, the x axis and two lines x = 1.
337 1996 Tokyo University of Agriculture and Technology
Z 2 X
n
Find lim
sin kx sin x dx.
n

k=1

336 1989 Chiba University


s
(3n)!
(n = 1, 2, ). Find the following limits.
Let Pn = n
(2n)!

71

Pn
.
n n
P

n+2 n
.
(2) lim
n
n
(1) lim

335
Evaluate

x p
1 x2 dx.
1+x

334 2007 Tokyo University Of Science


Z 1 3
7x + 23x2 + 21x + 15
Evaluate
dx.
(x2 + 1)(x + 1)2
0
333 1982 Ritsumeikan University
Find the functions f (x), g(x) such that f (x) +

g(t) dt = sin x(cos x sin x), {f (x)}2 + {g(x)}2 = 1.

332 1970 Rikkyo University


Let f (x) be a function such that 1 f (x) = f (1 x) for 0 x 1.
Z 1
Evaluate
f (x) dx.
0

331 2007 Keio University


For a R, find the minimum value of



sin 2x


dx.

a
cos
x
1 + sin2 x

330 2008 Ochanomizu Women University


Z x
Find all real numbers x such that
t2 sin(x t) dt = x2 .
0

329 2008 Hokkaido University


Let f (x), g(x) be continuous functions defined in the range of 0 x 1.
Z 1
(1) Find f (x) such that f (x) =
ex+t f (t) dt.
0
Z 1
(2) Find g(x) such that g(x) =
ex+t g(t) dt + x.
0

328 2008 Osaka University


Let t be a negative real number and D be the part bounded by the curve y = 22x+2t , the curve y = 2x+3t and the
y axis.
(1) Find the volume V (t) generated by rotation of D about the x axis.
(2) When t moves in the range of negative real number, find the maximum value of V (t).
327 2008 Gakusyuin University
2

Let a, b be real numbers and C be the graph of the function y = ea+bx .


(1) Find the values of a, b such that C passes through the point P (1, 1) and the slope of the tangent line of C at
P is 2.
(2) For the values of a, b found in (1), find the volume of the solid generated by the revolution of the part which
lies in the right side for the y axis in the figure bounded by the parabola y = x2 and the curve C about the y axis.
326
Evaluate

(x + 1)(x + 2 ln x)
dx.
(x + ln x)2

325 1954 Nippon University

72

1
Prove that <
3

x(sin x+cos x) dx <

1
.
2

324
Evaluate

+
x ln x

ln x
x

dx.

323
Evaluate

sin x
+
x

!
x
cos x dx.
sin x

322
Evaluate

(x4 4x3 + 9x2 6x + 8)ex


dx.
x3

x2 ex x2 e2x + 2ex + 2
dx.
(ex + 1)2

321
Evaluate

320
Evaluate

ex sin x (x2 cos x + x sin x + 1) dx.

319
Evaluate

xex + ex + 1
dx.
x2 (ex + 1)2

318
Evaluate

1 + cot x
dx.
ex sin x

317
Evaluate

x+ x1

dx.
( x + 1)2

e2x e2x + 4x
dx.
(ex + ex )2

(x2 3x + 1)ex (x 1)e2x x


dx.
(x + ex )3

316
Evaluate

315
Evaluate

314
Evaluate

xsin x (1 + x cos x ln x + sin x) dx.

313 1958 Kyoto University


Find the area bounded by the graph of y =

p
p

3
3
x + x2 + 1 + x x2 + 1, the line x y 1 = 0 and the x axis.

312 2008 Tokyo Metropolitan University

Let a, b be postive real numbers. For a real number t, denote the distance between the origin and the line
(aet )x + (bet )y = 1 by d(t).
73

Let a, b vary with ab = 1, find the minimum value of

1
dt.
d(t)2

311
Prove
Z 1 the followinginequality: 
2008
1
1
2008x
dx 1
.
2008
2008
ln
2008
0
310 2008 Tokyo Institute of Technology
Z 1
Find lim
x2 | sin nx| dx (n = 1, 2, ).
n

309 2008 Yokohama National University


(1)
Z Calculate the indefinite integral:
ex sin 2x dx.
(2)
Z Evaluate the definite integral:
ex | sin 2x| dx.
0

308 2008 Osaka Prefecture University


Let a be a positive constant number. For a positive integer n, define a function In (t) by In (t) =
Answer the following questions:
Note that you may use lim tn eat = 0 without proof.

xn eax dx.

(1) Evaluate I1 (t).


(2) Find the relation of In+1 (t), In (t).
(3) Prove that there exists lim In (t) for all natural number n by using mathematical induction.
t

(4) Find lim In (t).


t

307 2008 Kyoto University


For real numbers a, b, c, let f (x) = ax2 + bx + c. Prove that :

(1 x2 ){f (x)}2 dx 6

{f (x)}2 dx.

306 2008 Tokyo Institute of Technolgy


For positive real numbers a, b, two graphs of the function : xa and ln bx have a tangency of point.
(1) Let (s, t) be the tangency of point, express this in terms of a, then express b as the function of a.
(2) For h with 0 < h < s, denote the area as A(h) of the domain bounded by the line x = h and two curves
y = xa , y = ln bx.
Express lim A(h) in terms of a.
h0

305 2008 Yokohama National University


Z
p
Find
(1 1 e2x ) dx.
0

304 2008 Tokyo University

Let , be real numbers with 0 and f (x) = x2 ( + )x + such that


Z
f (x) dx.
maximum value of

f (x) dx = 1. Find the

303 2008 Tokyo University


In
( the x y plane, find the area of the region bounded by the parameterized curve as follows:
x = cos 2t
(0 t 2).
y = t sin t

74

302 2008 Keio University


Z
|x 1|
Evaluate
dx.
(x
+
1)(x2 + 1)
0
301 2008 Waseda University

For the positive constant number a, let D be the part surrounded by the curve x + y = a and the line
x + y = a.
(1) Draw the outline of D and find the area.
(2) Find the volume of the solid by rotating D about the line x + y = a as the rotation axis.
300 2008 Waseda University
In Euclidean space, take the point N (0, 0, 1) on the sphere S with
radius 1 centered in the origin. For moving

, consider the solid figure T in which the line
points P, Q on S such that N P = N Q and P N Q = 0 < <
2
segment P Q can be passed.
(1) Show that z coordinates of P, Q are equal.
(2) When P is on the palne z = h, express the length of P Q in terms of and h.
(3) Draw the outline of the cross section by cutting T by the plane z = h, then express the area in terms of and
h.
(4) Pay attention to the range for which h can be valued, express the volume V of T in terms of , then find the
maximum V when let vary.
299 2008 Meiji University
Z x
Let In (x) =
(ln t)n dt (x > 0) for n = 1, 2, 3, .
1

(1) Prove by mathematical induction that In (x) is expressed by In (x) = xfn (ln x) + Cn (n 1) in terms of some
polynomial fn (y) with degree n and some constant number Cn .
(2) Express the constant term of fn (y) interms of n.
298 2008 Keio University
(

Let A be the region :

(x, y)|x 0, y 0,

 x  12
a

y
1a

 21

for 0 < a < 1.

(1) Find the area of A.


(2) Find the maximum volume of the solid obtained by revolving A around the x axis.
297 2008 Kwansai University

Z 6 
3
x
sin x
dx.
cos
Evaluate
2
2
0
296 2008 Ritsumeikan University
Z 2
Let an =
(1 sin t)n sin 2t dt.
0

(1) Find

an .

n=1

an
.
n
n=1

X
(3) Find
(n + 1)(an an+1 ).
(2) Find

n=1

295 2008 Doshisya University


Z 1
Let fn (x) = xn (1 x)n , In =
fn (x) dx (n = 1, 2, ).

(1)
(2)
(3)
(4)

Find a polynomial g(x) such that fn+1


(x) = (n + 1)fn (x)g(x).

Find constant numbers An , Bn such that fn+2


(x) = An fn+1 (x) + Bn fn (x).
Find An In+1 + Bn In .
Let Jn = (2n + 1)!In . Express Jn+1 in terms of Jn .

75

(5) Find In .
294
Evaluate

ln 3

ln 2

ln(ex + 1)
dx.
e2x

293 2008 Sophia University


1 2
1
x ax + a2 + a 2 and C : y = x2 + 2 for real number a in the x y plane.
4
4
(1) Find the equation of the locus of the vertex for Ca .
(2) For a = 3, find the slope of the two common tangent lines of C and Ca , then the intersection points of the lines.
(3) Suppose that Ca intersects with C at two distinct points. Find the maximum area of the figure bounded by C
and Ca .
Consider the parabolas Ca : y =

292 2008 Tokyo University of Science


(

t3 (2y 1)t2 t + 2y 1 = 0
.
xt 2t + 2y = 0
In x y plane find the area of the figure bounded by the part of curves above with x 0 and the line y = 2.

Let x, y and t be real numbers such that

291 2008 Sophia University

Consider the parabola C : y = x2 in the x y plane. Let l1 be the tangent line of C with slope 3 and l2 be the
tangent line of C with negative slope which makes angle 45 with l1 . Find the area bounded by C, l1 and l2 .
290 2008 Keio University
Let the curve C : y = |x2 + 2x 3| and the line l passing through the point (3, 0) with a slope m in x y plane.
Suppose that C intersects to l at distinct two points other than the point (3, 0), find the value of m for which the
area of the figure bounded by C and l is minimized.
289
Let p be a positive constant number, find lim

|x sin(ax2 )| dx.

288
Evaluate

(1 + 2008x2008 )ex

2008

dx.

287 2008 Kwansei Gakuin University


For constant numbers a, b, let f (x) = ex (ax + b). Suppose that the curve y = f (x) passes through two points
(t, 1), (t + 1, 1). Find the area of the part bounded by the curve y = f (x) and the line y = 1.
286
Evaluate

2008

2008

f (x) + f (x)
dx.
2008x + 1

285 2008 Jikei University School of Medicine


Z 1
2
dt for 0 x 4.
Find the minimum value of
0 |2t x| + 3
284 1990 Tohoku University
f (x) is a continuous function which takes positive values for x 0. Find f (x) such that

1
with f (1) = .
2

283 1977 Tokyo Institute of Technology

76

p
f (t) dt = x f (x)

f (x) is a continuous function with the piriodicity of 2 and c is a positive constant number. Find f (x) and c such
Z 2
that
f (t x) sin t dt = cf (x) with f (0) = 1 for all real numbers x.
0

282 1976 Chiba University


g(x) is a differentiable
function for 0Z x and g (x) is a continuous function for 0 x . Let f (x) = g(x) sin x.
Z

Find g(x) such that

{f (x)}2 dx =

{f (x)}2 dx.

281 1983 Ritsumeikan University


Find the maximum and minimum values of

(a sin x + b cos x)3 dx for |a| 1, |b| 1.

280 1982 Kobe University


1
.
Let p, q (p < q) be the x coordinates of curves x2 + y 2 = 1, y 0 and y =
4x

(1) Find , such that cos = p, cos = q (0 < < , 0 < < ).
2
2
(2) Find the area bounded by the curves.
279 1982 Kwansai University
If the function f (x) = 3x2 + 2ax + b satisfies
278 1982 Doshisya University
Z 2

Find the value of t such that sZ0

1
1

|f (x)| dx < 2, then show that f (x) = 0 has distinct real roots.

(sin x + t cos x) dx
is maximized.

(sin x + t cos x) dx

277 1982 Kansei Gakuin University


 Z
Find the fuction such that f (x) = x x 2
2

|f (t)| dt

 12

276
Prove that

1
n
1
n

1
3
dx (n = 1, 2, 3, ).
2
sin x

275 1982 Toho University


The line y = mx has three intersection points with the curve y = |x(x 1)|. Find the value of m such that the
areas of two parts bounded by the line and the curve are equal.
274 1981 Tokyo University
For real constant numbers a, b, c, d, consider the function f (x) = ax3 + bx2 + cx + d such that f (1) = 0, f (1) =
0, f (x) 1 |x| for |x| 1.
Z 1
Find f (x) for which
{f (x) x}2 dx is minimized.
1

273
Find the area bounded by 0 y

x| sin x|
, 0 x n (n = 1, 2, ).
1 + | cos x|

272 1981 Nagasaki University


Z
Find lim
ex | sin nx|dx.
n

77

271 1989 Kanazawa University


For a positive constant
a, the function f (x) satisfies the following equation:
Z number
a
1
x
f (t) dt.
f (x) = xe a +
a+1 0 Z
a

(1) Express the value of

f (t)dt in terms of a.

(2) Find the maximum and minimum value of f (x) for 1 x 1.


270 1994 Tohoku University

Let f (x) be the continuous function at 0 x 1 such that


1).

(1) For all real numbers t, find the minimum value of g(t) =

(2) Show the following equation for all real real numbers t:
Z 1
Z 1
(x t)n f (x) dx =
xn f (x) dx.
0

xk f (x) dx = 0 for integers k = 0, 1, , n 1 (n

|x t|n dx.

(3) Let M be the maximum


value of the function |f (x)| for 0 x 1.
Z
1 n

M
Show that
x f (x) dx n
.
2 (n + 1)
0
269 2001 Osaka Medical College




1
1
1
( > 0). Find the minimum area bounded by
,
Let
A(,
f
()),
B

,
f

1 + x2

the line segments OA, OB and C, where O is the origin.


1
for the problem.
Note that you are not allowed to use the integral formula of
1 + x2
For the curve C : y =

268 1986 Tokyo University


Find the constant numbers u, v, s, t (s < t) such that
f (x) = ax3 + bx2 + cx + d with the degree of 3 or less than.

f (x) dx = uf (s) + vf (t) holds for any polynomials

267 1983 Tokyo University


3
Let K be the curved surface obtained by rotating the parabola y = x2 about the y-axis.Cut K by the plane H
4
passing through the origin and forming angle 45 for the axis. Find the volume of the solid surrounded by K and H.
Note that you are not allowed to use Double Integral for the problem.
266 1982 Waseda University
x2
Find the area of the region expressed by the system of inequality
+ y 2 1,
4

!
3
x + y 1.
1+
2

265 1985 Waseda University


Z x+y
Z x
Supposed that f (x) has f (x) and for any real numbers x, y,
f (t) dt =
{f (y) cos t + f (t) cos y} dt holds.
y
0 


(1) Express f (x + y) in terms of f (x), f (y). (2) Find f x +


. Note that f
= 2. (3) Find f (x).
2
2
264 1976 Tokyo Institute of Technology

Find the area of the figure surrounded by the locus of the point P inside the square with side length a such that
the distance to the center of the square is equal to the minimum distance to the side of the square.
263 1981 Tokyo Institute of Technology
Z
1 2t
| cos 2x| dx for 0 < t 1.
Let F (t) =
t 0
(1) Find lim F (t).
t0

78

(2) Find the range of t such that F (t) 1.


262 1977 Tsukuba University
Answer the following questions for positive integer n:
(1) Find the maximum value of fn (x) = xn ex for x 0.
(2) Show that lim fn (x) = 0.
x
Z x
(3) Let In (x) =
fn (t) dt, find lim In (x).
x

261 1983 Nagoya Institute of Technology


Z

Find the continuous function f (x) such that f (x) =

x
0

{f (t) cos t cos(t x)} dt.

260
Evaluate

(sin3 cos3 cos2 )(sin + cos + cos2 )2007


d.
sin2009 cos2009

259
Evaluate

12

dx
.
(sin x + cos x)4

258 1983 Osaka City University


Find the volume of the solid formed by the revolution of the curve x = a(1 + cos ) cos , y = a(1 + cos ) sin (a >
0, < ) about the x axis.
257
Evaluate

2008

x| sin x| dx.

256 1970 The University of Electro-Communications


Z
Find the value of a for which
{ax( 2 x2 ) sin x}2 dx is minimized.
0

255 1983 Muroran Institute of Technology


Find the value of a for which the area of the figure surrounded by y = ex and y = ax + 3 (a < 0) is minimized.
254 1985 Yamanashi University
(1 ln x) ln x
In the xy plane, find the area of the figure surrounded by y =
(x 0) and the x axis, then the
x
volume of the solid of the figure obtained by revolving about the y axis.
Z b
You arent allowed to use the formula:
2xf (x) dx.
a

253
(1) 1992 Tokyo Metropolitan University Find lim n2
n

(2) 1977 Dokkyo Medical University Evaluate


(2n 1)xn1 } dx.

1
2
n+2






n
x 1
(x

1)

dx.
n+2

(1 + x + x2 + + xn1 ){1 + 3x + 5x2 + + (2n 3)xn2 +

252
(1) 2007 Sinsyu
Z University
x3
dx.
(a) Calculate
x4

79

1
dx.
cos
x
0
n
k
1 X
.
k sin
(c) Find lim 2
n n
2n
(b) Evaluate

k=1

(2) 1971 Tokyo University of Foreign Studies Compare f () =

(x+sin ) dx and g() =

for 0 2.
251 2007 Kyoto University
Z 2
2x + 1

(1) Evaluate
dx.
x2 + 4
0
Note that you arent allowed to use the formula
Z n
(2) Evaluate
ex sin4 x dx (n = 1, 2, ).

(x+cos )2 dx

dx = ....
2
x +a

250
For a positive constant number p, find lim

np+1

n1
X Z (2k+1)

xp sin3 x cos2 x dx.

2k

k=0

249
Determine the sign of

2
1
2

ln t
dt (n = 1, 2, ).
1 + tn

248
Evaluate

3
4

cos

247
Evaluate

3
8

1
sin

1
sin x

 cos

1
sin x

cos x
sin x sin2
2

1
sin x

 dx.

11 + 4 cos 2x + cos 4x
dx.
1 cos 4x

246
A eightic function y = ax8 + bx7 + cx6 + dx5 + ex4 + f x3 + gx2 + hx + i (a 6= 0) touches the line y = px + q at
x = , , , ( < < < ). Find the area of the region bounded by these graphs in terms of a, , , , .
245
A sextic function y = ax6 + bx5 + cx4 + dx3 + ex2 + f x + g (a 6= 0) touches the line y = px + q at x = , , ( <
< ). Find the area of the region bounded by these graphs in terms of a, , , .
244
A quartic function y = ax4 + bx3 + cx2 + dx + e (a 6= 0) touches the line y = px + q at x = , ( < ). Find
the area of the region bounded by these graphs in terms of a, , .
243
A cubic function y = ax3 + bx2 + cx + d (a 6= 0) intersects with the line y = px + q at x = , , ( < < ).
Find the area of the region bounded by these graphs in terms of a, , , .
242
A cubic function y = ax3 + bx2 + cx + d (a 6= 0) touches a line y = px + q at x = and intersects x = ( 6= ).
Find the area of the region bounded by these graphs in terms of a, , .
241
1. Let x = , ( < ) are x coordinates of the intersection points of a parabola y = ax2 + bx + c (a 6= 0) and
the line y = ux + v.
80

|a|
( )3 .
6
2. Let x = , ( < ) are x coordinates of the intersection points of parabolas y = ax2 + bx + c and
y = px2 + qx + r (ap 6= 0).
|a p|
Prove that the area of the region bounded by these graphs is
( )3 .
6
Prove that the area of the region bounded by these graphs is

240 1990 Kyoto University


Two curves y = x3 x and y = x2 a pass through the point P and have a common tangent line at P . Find the
area of the region bounded by these curves.
239 1993 Tokyo University of Mercantile Marine
Z
Evaluate
sin( cos x) dx.
0

238 1991 Tokyo Medical University


Z a
1
Find lim 2
log(1 + ex ) dx.
a a
0
237
Calculate

dx
.
x2008 (1 x)

236 1998 Sinsyu University


Let aZbe a positive constant. ZEvaluate the following definite integrals A, B, where

A=
eax sin2 x dx, B =
eax cos2 x dx.
0

235 1994 Tsukuba University


Z 1
Show that a function f (x) =
(1 | t |) cos(xt) dt is continuous at x = 0.
1

234 1993 Musashi Institute of Technology


 n 
sin x (n x < (n + 1)) (n = 0, 1, 2, ).
For x 0, define a function f (x) = sin
4
Z 100
Evaluate
f (x) dx.
0

233 1966 Yokohama City University


Find
Z the minimum value of the following definite integral:
(a sin x + b sin 3x 1)2 dx.
0

232 1995 Tokyo University


For f (x) = 1 sin x, let g(x) =

(x t)f (t) dt.

Show that g(x + y) + g(x y) 2g(x) for any real numbers x, y.


231
Evaluate

1
dx.
cos7 x

230 1990 Oita University


Z
n
X
(1)n 2
( ln 2)k
Prove that
+ 1.
(ln x)n dx = 2
n!
k!
1
k=1

81

229 1985 Japan Women University


Z a
sin4 x dx
0
.
Find lim
a+
a
228 1972 Tokyo Medical and Dental University
Z 2
sinn d (n = 0, 1, 2, ).
Let xn =
0

n1
xn2 .
n
(2) Find the value of nxn xn1 .
(3) Show that a sequence {xn } is monotone decreasing.
(4) Find lim nx2n .
(1) Show that xn =

227
1

Evaluate Z

2006

cos

x sin 2008x dx

226
Evaluate

x2
dx.
(cos x + x sin x)2

225 2007 Kogakuin University






1
1
1
, Q 2a,
(a > 0) are on the curve C : y = . Let l, m be the tangent lines at P, Q
Two points P a,
a
2a
x
respectively. Find the area of the figure surrounded by l, m and C.
224 1966 University of Hyogo
Z
Z
2
Let f (x) = x + |x|. Prove that
f (cos x) dx = 2
0

f (sin x) dx.

223 1964 Toyama University


Z p
(cos x + cos 2x + cos 3x)2 + (sin x + sin 2x + sin 3x)2 dx.
Evaluate
0

222

Find lim

a+1

x
dx.
x + ln x

221 2007 Nanzan University

Z 6
1 + 1 + 4x
ln
Evaluate
dx.
2
2
220 2007 Chuo University
Z 1
2

Prove that 1 <


e2x dx.
2
0
219 2007 Kochi University

x
1
Let f (x) = 1 +
(x > 0).
x
      
 n  n1
2
3
1
f
f
f
.
Find lim f
n
n
n
n
n
218 1984 Muroran Institute of Technology

82

For any quadratic functions f (x) such that f (2) = 1, evaluate

2+

f (x) sin

x
2

dx.

217
Evaluate

1
ex

dx + 2

ln(ln x) dx.

216 2007 Aoyama Gakuin University


Z an x
e 1
dx = ln n.
Let an is a positive number such that
1 + ex
0
Find lim (an ln n).
n

215 2007 Tokyo University of Agriculture and Technology


Z
sin(x t) sin(2t a) dt.
For a R, let M (a) be the maximum value of the function f (x) =
0
Z 2
Evaluate
M (a) sin(2a) da.
0

214 1956 Tokyo Institute of Technology


Find the area of the region surrounded by the two curves y =

x, x + y = 1 and the x axis.

213
(1) 2007 Japan Womens University
Z 1
Find the minimum value of f (a) =
x|x a| dx.
0

(2) 1977 Yokohama National University


Z 2
(a) Evaluate
(x cos x + sin2 x) sin x dx.
0Z
x
et sin(x t) dt, find f (x) + f (x).
(b) For f (x) =
0

212 1978 Kyoto University


Z
For integers k (0 k 5), positive numbers m, n and real numbers a, b, let f (k) =
(sin kx a sin mx

 5
5
X
1
5!
2
, E=
p(k)f (k). Find the values of m, n, a, b for which E is minimized.
b sin nx) dx, p(k) =
k!(5 k)! 2
k=0

211 1969 Ochanomizu Women University


When the parabola which has the axis parallel to y -axis and passes through the origin touch to the rectangular
hyperbola xy = 1 in the first quadrant moves, prove that the area of the figure sorrounded by the parabola and the
x-axis is constant.
210
Evaluate



6
sin x dx.
x3 ln x
x

209 1972 Yokohama National University


Let m, n be the given distinct positive integers.Z Answer the following questions:

(1) Find the real number (|| < 1) such that


sin(m + )x sin(n + )x dx = 0.

(2) Find the real number satifying the sytem of equation


Z
2

sin2 (m + )x dx = +

4m
1
Z

sin2 (n + )x dx = +

4n 1

83

208 1979 Waseda University

Find the values of real numbers a, b for which the function f (x) = a| cos x| + b| sin x| has local minimum at x =
3
Z 2
2
and satisfies
{f (x)} dx = 2.

207
Evaluate the following definite integral:

ee+1

e3

ee


1
+ ln(ln(ln x)) dx.
ln x ln(ln x)

206 1986 Oita Univesristy


Z
x3
Calculate
dx.
(x 1)3 (x 2)
205
Evaluate the following definite integral:

e2

ln x ln(x ln x) ln{x ln(x ln x)} + ln x + 1


dx.
ln x ln(x ln x)

204 1966 Himeji Institute of Technology


Z 1
x dx
.
Evaluate
2 + x + 1) 32
(x
0
203 1967 Tokyo Instutute of Technology
Let , be the distinct positive roots ofZthe equation of 2x = tan x.
1
Evaluate the following definite integral:
sin x sin x dx.
0

202
Let a, b are real numbers such that a + b = 1.
Find the minimum value of the following integral:

(a sin x + b sin 2x)2 dx.


0

201
Evaluate the following definite integral:

e2x + 1 (x + 1)(ex + ex )
dx.
x(ex 1)

200
Evaluate the following definite integral:

cos nx
dx (n = 0, 1, 2, ).
2 cos x

199
Let m, n be non negative integers. Calculate

n
X

(1)k

k=0

i (i 1) (i j + 1)
means
.
j (j 1) 2 1

n+m+1 k
C , where Cij is a binomial coefficient which
k+m+1 n

198
Compare the values of the following definite integrals:


2

Z 2 
dx

1
d.
,
and
ln x +
x 1 + x2
sin
0

197 1967 Kanazawa University

Let |a| < . Evaluate the following definite integral:


2

84

dx
.
{sin(a + x) + cos x}2

196

Calculate

Z0

ex sinn x dx

ex sinn x dx

(n = 1, 2, ).

195 1989 Waseda University


Find continuous functions x(t), y(t) such that
Z t
x(t) = 1 +
e2(ts) x(s) ds,
Z t 0
y(t) =
e2(ts) {2x(s) + 3y(s)} ds.
0

194
Evaluate

2006
XZ 1
n=0

dx
p
.
2(x + n + 1) (x + n)(x + n + 1)

193 2007 Tokyo Institute of Technology

For a > 0, let l be the line created by rotating the tangent line to parabola y = x2 , which is tangent at point

A(a, a2 ), around A by . Let B be the other intersection of l and y = x2 . Also, let C be (a, 0) and let O be the
6
origin.
(1) Find the equation of l.
(2) Let S(a) be the area of the region bounded by OC, CA and y = x2 . Let T (a) be the area of the region bounded
T (a)
by AB and y = x2 . Find lim
.
a S(a)
192 1965 Tokyo Instutute of Technology
Let t be positive number. Draw two tangent lines to the palabola y = x2 from the point (t, 1). Denote the area
S(t)
of the region bounded by these tangent lines and the parabola by S(t). Find the minimum value of .
t
191 2007 Tokyo Institute of Technology
(1) For integer n = 0, 1, 2, and positive number an , let fn (x) = an (x n)(n + 1 x). Find an such that the
curve y = fn (x) touches to the curve y = ex .
(2) For fn (x) defined in (1), denote the area of the figure bounded by y = f0 (x), y = ex and the y-axis by S0 , for
n 1, the area of the figure bounded by y = fn1 (x), y = fn (x) and y = ex by Sn . Find lim (S0 + S1 + + Sn ).
n

190 2007 Tohoku University


In xyz space, let l be the segment joining two points (1, 0, 1) and (1, 0, 2), and A be the figure obtained by
revolving l around the z axis. Find the volume of the solid obtained by revolving A around the x axis.
Note you may not use double integral.
189 2007 Osaka University

Let n be
positive integers. Denote the graph of y = x by C, and the line passing through two points (n, n)
and (n + 1, n + 1) by l. Let V be the volume of the solid obtained by revolving the region bounded by C and l
around the x axis.Find the positive numbers a, b such that lim na V = b.
n

188 2007 Kyoto University


Find the volume of the solid obtained by revolving the region bounded by the graphs of y = xe1x and y = x
around the x axis.
187 2007 Tsukuba University

For a constant a, let f (x) = ax sin x + x + . Find the range of a such that
2

85

{f (x)}2 dx f

 
.
2

186 2007 Tohoku University


Z a

( 32 +n)
xn 1 + x dx (n = 1, 2, ).
For a > 0, find lim a
a

185 2007 Yokohama National University


Evaluate the following integrals:
Z 4
dx
(1)
.
1 + sin x
Z0 2
dx

.
(2)
4 x2
x1
3
184 2007 Tokyo University
(1) For real numbers x, a such that 0 < x < a, prove the following inequality:


Z a+x
2x
1
1
1
.
<
dt < x
+
a
a+x ax
ax t
(2) Use the result of (1) to prove that 0.68 < ln 2 < 0.71.
183 2007 Tokyo University
Let n 2 be integer. On a plane there are n + 2 points O, P0 , P1 , Pn which satisfy the following conditions
as follows:

[1] Pk1 OPk = (1 k n), OPk1 Pk = OP0 P1 (2 k n).


n
1
[2] OP0 = 1, OP1 = 1 + .
n
n
X
Find lim
Pk1 Pk .
n

k=1

182 2007 Tokyo University


Find the area of the domain of the system of inequality y(y |x2 5| + 4) 0, y + x2 2x 3 0.
181 2007 Keio University
For real number a, find the minimum value of

180 2007 Waseda University




sin 2x
dx.

a
cos
x

1 + sin2 x

Let an be the area surrounded by the curves y = ex and the part of y = ex | cos x|, (n 1) x n (n =
1, 2, 3, ). Evaluate lim (a1 + a2 + + an ).
n

179
Evaluate the following integrals.
(1)
University
Z e 2007 Meiji
(log x)2
dx.
1
x
e
(2) 2007 Tokyo University of Science
Z 1 3
7x + 23x2 + 21x + 15
dx.
(x2 + 1)(x + 1)2
0
178 2007 Meiji University
Let f (x) be a differentiable function such that f (x) + f (x) = 4xex sin 2x, f (0) = 0.
n
X
f (k).
Find lim
n

k=1

177 2007 Tokyo University of Science

86

1
On xyplane the parabola K : y = x2 (d : positive constant number) intersects with the line y = x at the point
d
P that is different from the origin. Assumed that the circle C is touched to K at P and y axis at the point Q. Let
S1 be the area of the region surrounded by the line passing through two points P, Q and K, or S2 be the area of the
S1
.
region surrounded by the line which is passing through P and parallel to x axis and K. Find the value of
S2
176
(1) 1984 Chiba University
Z Let k be positive integers and f (x) be the polynomial with integers coefficients. Find k and f (x) such that
x

f (t)dt + xk = xf (x), x 1.

(2) 1989 Gunma University


n
X
sin kx
p
Let fn (x) =
.
k(k + 1)
k=1
Z 2
Find lim
{fn (x)}2 dx.
n

175
(1) 1984 Shiga Medical University
A cup of coffee is warmed in 90 C. It left in the room of 10 C for three minutes, it reached 70 C. How much
time does it takes to go down to 55 C from the first ? Note that room temperature is fixed, descending speed of
temperature is proportional to the difference of temperature to the that of surrouding.
(2) 2007 Kwansei Gakuin University

X
1
Evaluate
.
2n+1
(2n
+
1)2
n=0
174 1987 Tokyo Institute of Technology
Let a be a positive number. Assume that the parameterized curve C : x = t + eat , y = t + eat ( < t < )
is touched to x axis.
(1) Find the value of a.
(2) Find the area of the part which is surrounded by two straight lines y = 0, y = x and the curve C.
173 1999 Waseda University
Find the function f (x) such that f (x) = cos(2mx) +

f (t)| cos t| dt for positive inetger m.

172 Yokohama City University


Z 0r
1+x
Evaluate
dx.
1x
1
171
Evaluate

x2007 (1 x2 )1003 dx.

170 1986 Toyama Medical and Pharmaceutical University


Let a, bZbe constant numbers such that a2 b. Find the following definite integrals:
dx
.
(1) I =
2 + 2ax + b
x
Z
dx
.
(2) J =
(x2 + 2ax + b)2
169 1985 Osaka Women University
(1) Let f (x) be the differentiable and increasing function such that f (0) = 0. Prove that
Z 1
2
1
f (x) dx .
2
0
87

f (x)f (x) dx

2n+1

(2) gn (x) = x

+ an x + bn (n = 1, 2, 3, ) satisfies

Find an , bn .

(px + q)gn (x) dx = 0 for all linear equations px + q.

168 1980 Nagasaki University



Z 1
X
n
1


Prove that
x sin x dx diverge for x > 0.
1
n=1 n+1
167 1982 Tokyo University

In xyz plane find the volume of the solid formed by the points (x, y, z) satisfying the following system of
inequalities:
0 z 1 + x + y 3(x y)y, 0 y 1, y x y + 1.
166 1989 Toyama Medical and Pharmaceutical University
Express the following
of a definite integral and find them:
 values
 in terms
 thelimit

2
n
1
1
1+
1 +
.
ln 1 +
(1) I = lim
n n
n
n
n


p
p
p
1
(2) J = lim 2
n2 1 + n2 2 2 + + n2 n2 .
n n

p
p
1 p 2
n + 1 + 2 n2 + 2 2 + + n n2 + n2 .
(3) K = lim 3
n n
165

On x y plane, let C : y = 2006x3 12070102x2 + . Find the area of the region surrounded by the tangent line
of C at x = 2006 and the curve C.
164 Nara Womens University
For positive integers n, let
1
1
1
1
1
1
+r
+ + r
.
Sn = + + + , T n = r
n
1
2
1
1
1
1+
2+
n+
2
2
2
Tn
Find lim
.
n Sn
163
Z

Let In =
Find

tann x dx (n = 0, 1, 2, ).

{In+2 2 + (In+1 + In+3 )In+2 + In+1 In+3 }.

n=0

162
Let f (x) be the function such that f (x) > 0 at x 0 and {f (x)}2006 =
Find the value of {f (2006)}2005.

f (t)dt + 1.

161 1992 Kogakuin University


Find the differentiable function f (x) such that f (x) =

f (t) tan t dt +

x
0

160
Find the value of m (0 < m < 1) for which

| sin x mx| dx is minimized.

159 2000 Kyoto University


A function is defined by f (x) =

1
dt.
1 + t2

88

tan(t x) dt



.
|x| <
2

(1) Find the equation of normal line at x = 1 of y = f (x).


(2) Find the area of the figure surrounded
Z by the normal line found in (1), the x axis and the graph of y = f (x).
1
dx = tan1 x + C .
Note that you may not use the formula
1 + x2
158 1994 Tokyo Institute of Technology
Z
(1) Evaluate the definite integral
ex sin x dx.
0
Z n
(2) Find the limit lim
ex | sin x| dx.
n

157 1988 Tokyo University


Find the volume of the solid expressed by the following six inequaities in xyz space:
x 0, y 0, z 0, x + y + z 3, x + 2z 4, y z 1.
156 1990 Rikkyo University
For arbiterary integers n, find the continuous function f (x) which satisfies the following equation:
1
lim
h0 h

x+nh

f (t)dt = 2f (nx).

xnh

Note that x can range all real numbers and f (1) = 1.


155 2000 Kyoto University
The sequence {cn } is determined by the following equation:
cn = (n + 1)

Let be the limit value lim cn . Find lim


n

xn cos x dx (n = 1, 2, ).

cn+1
.
cn

154 2002 Kyoto University


Find the function f (x) which is defined for


x
.
2
2

Z 2
h i
such that f (x) +
,
sin(x y) f (y) dy = x + 1,
2 2

153 1980 Waseda University

y2
x2
Draw the perpendicular to the tangent line of the ellipse 2 + 2 = 1 (a > 0, b > 0) from the origin O(0, 0).
a
b
Let be the angle between the perpendicular and the positive direction of x axis. Denote the length of the
perpendicular by r().
Z 2
Calculate
r()2 d.
0

152 1970 Fukui University


Let f (x) the function such that f (0) = 0, |f (x)|

1
(x 0). Prove that
1+x

e1

{f (x)}2 dx e 2.

151 Yokohama National University


Let a, b be positive constant numbers. Find the volume of the revolution of the region surrounded by the parabola
y = ax2 and the line y = bx about the line y = bx as the axis on xy plane.
150 1987 Kanazawa University
3
Find the value of a such that lim
n 2

3 a


n

t3
1

t2 dt = 2 (n = 1, 2, ).

n
3a

89

149
148 1975 Keio University
Z 2
sin2 n
d (n = 1, 2, ).
Evaluate
sin2
0
147 1980 Fukushima Medical University
Find the area of the figure surrounded by the curve 2(x2 + 1)y 2 + 8x2 y + x4 + 4x2 1 = 0.
146 1978 Tokyo Institute of Technology
Z 1
Find the maximum value of
|x a|ex dx for |a| 1.
1

145 1980 Yamagata University



Z x+l 
1
t+
Find the minimum value of
dt (x > 0, l > 0).
t
x
144 1978 Shizuoka University
Z 




sin nx dx (n = 1, 2, ).
Evaluate lim
x+
n 0
n
143 1971 Tokyo University of Fisheries
Z 2
1 sin 2x
dx.
Evaluate
(1 + sin 2x)2
0
142 1968 Tokyo University of Fisheries
Z
sin x

dx (a > 0).
Evaluate
1

2a
cos x + a2
0
141
Evaluate

cos 4x cos 4
dx.
cos x cos

140 1998 Kagawa University


Evaluate
Z

cos x
sin x + cos x

2

dx,

sin x + cos x
cos x

2

dx.

139 2003 Osaka City University


Let a, b be real numbers. Evaluate
Z 2
0

(a cos x + b sin x)2n dx (n = 1, 2, ).

138 1968 Tokyo Institute of Technology


Let f (x) be the product of functions made by taking four functions from three functions x, sin x, cos x repeatedly.
Z 2
f (x) dx.
Find the minimum value of
0

137 1977 National Defense Medical College


Z 1
Find the value of a for which
|xex a| dx is minimized.
0

136 1992 Tokyo Institute of Technology


90

Let c be the constant number such that c > 1. Find the least area of the figure surrounded by the line passing
through the point (1, c) and the palabola y = x2 on x y plane.
135 1966 Tohoku University
Z 2
Find the value of a for which
|a sin x cos x| dx (a > 0) is minimized.
0

134 1968 Chiba University


1
1
For positive integers n, let An = {(n + 1) + (n + 2) + + (n + n)}, Bn = {(n + 1)(n + 2) (n + n)} n . Find
n
An
.
lim
n Bn
133 1968 Osaka University of Education


Let f (x) be the polynomial with respect to x, and gn (x) = n 2n2 x
Z 1
f (x)gn (x) dx.
Find lim
n




1
2
+
n

2n


x
2


1
.
2

132 1968 Osaka University


1

Find the area of the figure such that the points (x, y) satisfies the inequality lim (x2n + y 2n ) n
n

131 1966 Kochi University


For a > 0, find the minimum value of

1
a

(a3 + 4x a5 x2 )eax dx.

130 2006 Tokyo University of Science


Z a

1
3
dx = ln 2.
Find the value of a such that
x
x
e
+
4e
+
5
0
129 2006 Osaka Institute of Tecnology
The sequence {an } is defined as follows:
a1 =

, an =
4

1
2

(cos x + an1 ) cos x dx (n = 2, 3, ).

Find lim an .
n

128 2006 Miyazaki University


Prove the following inequality:

3
ln 2 +
<
3
81

ln(cos x)dx <

127
126 1977 Kanazawa University
For t > 0, find the minimum value of

x|ex t|dx.

125
Prove the following inequality for x 0:
Z

x
0

(t t2 ) sin2004 t dt <
91

1
2006

3
.
162

3 2 3 2
x + y 1.
2
2

124 2006 Kwansei University


a4
Let a > 1. Find the area S(a) of the part surrounded by the curve y = p
(0 x 1), x axis , y axis
(a2 x2 )3
and the line x = 1, then when a varies in the range of a > 1, then find the extremal value of S(a).
123 1989 Tokyo University
Let f (x) = x2 sin x2 . Prove that the volume V formed by the revolution of the figure surrounded by the part
Z 1
0 x 1 of the graph of y = f (x) and x axis about y axis is can be given as 2
xf (x) dx, then find the value of
0

V.

122 1987 Tokyo Metropolitan University




. Find the area surrounded by the curve : x = x(t), y = y(t) and
Let x(t) = tan t, y(t) = ln cos t < t <
2
2
x axis and the line x = 1.
121 1993 Kwansei University
Given the parabola C : y = x2 . If the circle centered at y axis with radius 1 has common tangent lines with C at
distinct two points, then find the coordinate of the center of the circle K and the area of the figure surrounded by C
and K.
120 2006 Hosei University
Let k be real constants.How many real roots can the following quadratic equation have?
Z 1
x2 = 2x + k +
|t + k| dt.
0

119 1976 Tokyo Institute of Technology


Find the continuous function f (x) and constant number such that

f (t) dt = e ae

2x

f (t)et dt.

118 1978 Meiji University


Let f (x) be(the function defined for x 0 which satisfies the following conditions:
x if 0 x < 1
(a) f (x) =
.
2 x if 1 x < 2
(b) f (x + 2n) = f (x) (n = 1, 2, ).
Z 2n
Find lim
f (x)ex dx.
n

117 1978 Yokohama National University


Let a be a real constant number. Evaluate lim n
n

en|ax| dx.

116 1976 Toyama University


Z 2
| sin(x + t) sin x| dx
.
Find lim
t0 0
|t|
115 1977 Chiba University
Z
Find the value of a such that

(sin x + a cos x)3 dx

4a
2

x cos x dx = 2.

114 2006 Shibaura Institute of Technology


Let a be positive numbers.For |x| a, find the maximum and minimum value of
113
92

x+a

xa

p
4a2 t2 dt.

Evaluate

sin x + cos x + 3( sin x cos x) cos 2x

dx.
sin 2x

112
Evaluate

d
, (e < 1 is a constant number).
(1 sin2 )3
e2

111
Evaluate

enx cosm x dx (m, n = 0, 1, 2, ).

110 2003 Kochi University


Prove the following inequality:
1

1
1 sin3 x dx { 2 + ln(1 + 2 )}.
2

109
Let In =

1
2006+ n

2006

Find lim nIn .

x cos2 (x 2006) dx (n = 1, 2, ).

108 1982 Tokyo Gakugei University


For x 0, find the minimum value of x for which

2t (2t 3)(x t) dt is minimized.

107
Evaluate

1 x2
dx (a > 1).
ax

106
Evaluate

1 x2
dx

.
1 + x2 1 + x4

105 1986 Miyagi University of Education


Let a, b be constant numbers such that 0 < a < b. If a function f (x) always satisfies f (x) > 0 at a < x < b, for
a < t < b, find the value of t for which the following the integral is minimized:
Z

|f (x) f (t)|x dx.

104 1989 Kyoto Prefectural University


Z x
dt

dt.
For 0 < x < 1, let f (x) =
1 t2
0
d
f ( 1 x2 ).
(1) Find
dx

1
(2) Find f .
2

(3) Prove that f (x) + f ( 1 x2 ) = .


2
103 1988 Shizuoka University
For 0 < a < 1, let f (x) =

ax
(1 < x < 1).
1 ax
93

Evaluate

1 {f (x)}6
dx.
1 x2

102 1986 Toyama Medical and Pharmaceutical University


Let a, b Zbe costant numbers such that a2 b. Find the following indefinite integrals:
dx
.
(1) I =
2
Z x + 2ax + b
dx
.
(2) J =
2
(x + 2ax + b)2
101 1985 Osaka Institute of Technology
For n > 2, prove the following inequality:
1
<
2

1
2

dx < .
n
6
1x

100
Let a, b, c be positive numbers such that abc =
Z

1
. Prove the following inequality:
16

x2
dx .
(x2 + a2 )(x2 + b2 )(x2 + c2 )

99 1980 Meiji University


Let be a constant number such that 0 . Evaluate
Z 2
sin 8x| sin(x )| dx.
0

98
Let
In =

Evaluate lim Inn .

1
1+ n

{[(x + 1) ln x + 1]ex(e

ln x+1)

+ n} dx (n = 1, 2, ).

97 2006 Keio University


Answer the following questions:
Z ee
ln(ln x)
(1) Evaluate
dx.
x ln x
e
(2) Let , be real numbers. Find the values of , for which the following equality holds for any real numbers
p, q:
Z 2
(p cos x + q sin x)(x2 + x + ) dx = 0.

96 2006 Japan Womens University


Z 1
For a 0, find the minimum value of
|x2 + 2ax| dx.
2

95 2006 Shibaura Institute of Technology


Evaluate
Z

2 sin3 x
dx.
cos5 x

94 1979 Yokohama City University


94

Let a be real numbers. Find the following limit value:


1
T T

lim

(sin x + sin ax)2 dx.

93
Evaluate

1
Z

2005

cos

x sin 2007x dx

92
Evaluate lim n2
n

1
n

(2005 sin x + 2006 cos x)|x| dx.

1
n

91
Prove the following inequality:

Z
X

n=0

1
0

n1

x4011 (1 x2006 ) 2006 dx <

2006
.
2005

90 1988 Tokyo Institute of Technology


Find lim

n
C3n
n
C2n

 n1

, where Cij is a binominal coefficient which means

89 1986 Fukushima Medical University


Z
Z
For f (x) = x4 + |x|, let I1 =
f (cos x) dx, I2 =
Find the value of

I1
.
I2

i (i 1) (i j + 1)
.
j (j 1) 2 1

f (sin x) dx.

88 1988 Meiji University


(
f (x) = f (x) (4x 2)f (x)
A function f (x) satisfies
f (0) = a, f (1) = b
Z 1
Evaluate
f (x)(x2 x) dx.

87
Find the minimum value of a (0 < a < 1) for which the following definite integral is minimized:
Z
| sin x ax| dx.
0

86 1984 Jyosai University


Prove that
Z

2
cos x
1
dx < 2 .
x

85 1991 Ochanomizu Women University


Evaluate
lim

[n sin x]
dx,
n

where [x] is the integer equal to x or less than x.

95

84 1993 Kochi University


Evaluate
lim n

enx sin2 nx dx.

83 2004 Kochi Women University


Evaluate

Z
X

n=1

2(n+1)

x sin x + cos x
dx (n = 1, 2, ).
x2

2n

82 2004 Osaka University of Education


Let 0 < a < b. Prove the following inequaliy:
1
ba

2

b
ln
dx < 2.
x

81 1978 Kyoto Institute of Technology


Prove the following inequality:

1
( 6 + 2 3)
12

ln(1 + cos 2x)dx

1
(2 3).
4

80 2005 Osaka University


Let S be the domain surrounded by the two curves C1 : y = ax2 , C2 : y = ax2 + 2abx for constant positive
numbers a, b. Let Vx be the volume of the solid formed by the revolution of S about the axis of x, Vy be the volume
Vx
.
of the solid formed by the revolution of S about the axis of y. Find the ratio of
Vy
79 1980 Shimane University
Find the area of the domain expressed by the following system inequalities:
1

x 0, y 0, x p + y p 1 (p = 1, 2, ).
78 1967 Tokyo Institute of Technology
Let , be the distinct positive roots of the equation of 2x = tan x. Evaluate
Z 1
sin x sin x dx.
0

77 1969 Gakusyuin University


Find the area of the part enclosed by the following curve:
x2 + 2axy + y 2 = 1 (1 < a < 1).
76 1992 Toyama University
The function fn (x) (n = 1, 2, ) is defined as follows:
n+1

f1 (x) = x, fn+1 (x) = 2x


Evaluate

1
x +
2
n

fn (t) dt (n = 1, 2, ).



1
.
lim fn 1 +
n
2n
96

75
(1) 1970 Tokyo Institute of Technology
A function f () satisfies the following conditions (a), (b).
(a) fZ() 0.

f () sin d = 1.

(b)

Prove the following inequality:

f () sin n d n (n = 1, 2, ).

(2) 1979 Aoyamagakuin University


Let a, b are real numbers. Evaluate
Z
|a cos nx + b sin nx|dx (n = 1, 2, ).
0

74 1978 Nagoya University


The numbers p, and q satisfies px + q ln x at a x b (0 < a < b).
Find the value of p, q for which the following definite integral is minimized and then the minimum value:
Z b
(px + q ln x) dx.
a

73 1994 Kyusyu University


Z
Find the minimum value of
(a sin x + b sin 2x + c sin 3x x)2 dx.
0

72 1992 Chiba University


Let f (x) be a continuous function satisfying f (x) = 1 + k
Z
Find the value of k for which
f (x) dx is maximized.

f (t) sin(x t) dt, where k is a constant number.

71 1993 Gakusyuin University


Z 1p
Find the minimum value of
|t x| dt.
1

70 1991 Ryukoku University


Z 2
Find the number of root for
ex cos(x + a) dx = 0 at 0 a < 2.
0

69 1993 Doshisya University


Z
1
xfn1 (t) cos3 t dt (n 2).
Let f1 (x) = x, fn (x) = x +
14 0
Find lim fn (x).
n

68 1994 Kanazawa University


Z e
a

Find the minimum value of
ln x dx (0 a e).
x
1
67

Evaluate
2005

1002

dx

+
2
2
1002 x + 10032 x2
Z 1p
1 x2 dx
0

97

1003

1002

p
10032 x2 dx

66 1985 Nagaoka University of Technology


Z 2
Find the minimum value of
| cos x a| sin x dx.
0

65 2005 Nara Medical University


Let a > 0. Find the minimum value of

x
1 + x dx.
a

64 1991 Waseda University


Let f (t) be the cubic polynomial for t such that cos 3x = f (cos x) holds for all real number x. Evaluate
Z 1
p
{f (t)}2 1 t2 dt.
0

63 1997 Hiroshima University






n
X

2k 1
2k + 1

x cos
x .
For a positive number x, let f (x) = lim
cos
n
2n
2n
k=1

Evaluate

lim

f (x)
.
x

62 1999 Kanazawa University


Z
1
1 1
|axn 1|dx + (n = 1, 2, ) and let bn be the minimum value of f (a) at a > 1.
For a > 1, let f (a) =
2 0
2
Evaluate
lim bm bm+1 b2m (m = 1, 2, 3, ).
m

61
Evaluate

2004
XZ 1

k=0

1 x2

dx.
k+1x

60 1978 Hokkaido University


Z 2
n1
sin 2t (1 sin t) 2 dt (n = 1, 2, ).
Let an =

Evaluate

n=1

(n + 1)(an an+1 ).

59
Evaluate

(cos 2x)(cos 22 x) (cos 22006 x) dx.

58 1998 Tohoku University


ex
.
+1
Prove the following equality:
Let f (x) =

ex

f (x)dx +

f (b)

f (a)

f 1 (x)dx = bf (b) af (a).

57 1996 Gakusyuin University


98

Find the value of n N satisfying the following inequality:


Z


99 2
2

<
x
sin
nx
dx

100n .
0
56 1990 Tokyo University of Agriculture and Technology

Let H be the piont of midpoint of the cord P Q that is on the circle centered the origin O with radius 1. Suppose
t
the length of the cord P Q is 2 sin for the angle t (0 t ) that is formed by half-ray OH and the positive direction
2
of the x axis. Answer the following questions:
(1) Express the coordiante of H in terms of t.
(2) When t moves in the range of 0 t , find the minimum value of x coordinate of H.

(3) When t moves in the range of 0 t , find the area S of the region bounded by the curve drawn by the
2
point H and the x axis and the y axis.
55 1993 Kyoto University
Evaluate
lim n

(1 + x)n1 ex dx (n = 1, 2, ).

54 1997 Yokohama City University


Evaluate
Z

0
1

1+x
dx.
1x

53 1999 Tokyo Gakugei University


Find the maximum value of the following integral:
Z

ex sin tx dx.

52
51 1985 The University of Electro-Communications

A function f (x) satisfies f (x) = f xc for some real number c(> 1) and all positive number x.
Z c
Z c
f (x)
f (x)
If
dx = 3, evaluate
dx.
x
x
1
1
50 2004 Tokyo University of Science
Let a, b be real numbers such that a < b. Evaluate
Z b
ln |1 + (x a)(b x)|dx
.
lim a
ba
(b a)3
49
For x 0, prove that

(t t2 ) sin2002 t dt <

1
.
2004 2005

48 1969 Yokohama City University


Evaluate
lim

X1
sin2 nx
dx
sin x
k
k=1

47 2001 Tokyo University


99

Find the condition of a, b for which the function f (x) (0 x 2) satisfying the following equality can be
determined uniquely,then determine f (x), assuming that f (x) is a continuous function at 0 x 2:
f (x) =

a
2

sin(x + y)f (y)dy +


0

b
2

cos(x y)f (y)dy + sin x + cos x.

46 1992 Tokyo Institute of Technology


Z 1
|t x|
Find the minimum value of
dt.
t+1
0
45 1999 Tokyo University of Science
Find the function f (x) which satisfies the following integral equation:
f (x) =

t(sin t cos t)dt +

et f (t) dt.

44
Evaluate

sin 2005x
dx.
sin x

43
Evaluate
Z

cos2004 x cos 2004x dx.

42 2000 Shiga University of Medical Science

Let 0 < t < .


2
Evaluate
lim

t 2

tan cos ln(cos ) d.

41 1988 Wakayama Medical University


Evaluate

p
2ax x2 dx (a > 0).

40
Evaluate
Z

x2005 ex dx.

39 1990 Tokyo Institute of Technology


Find the minimum value of the following function f (x) defined at 0 < x <
f (x) =

d
+
cos

:
2

d
.
sin

38 1998 Kyoto University


Let a be a constant number such that 0 < a < 1 and V (a) be the volume formed by the revolution of the figure
which is enclosed by the curve y = ln(x a), the x-axis and two lines x = 1, x = 3 about the x-axis. If a varies in the
range of 0 < a < 1, find the minimum value of V (a).
100

37 1989 Kwansei Gakuin University


Evaluate
Z

2
3

dx.
sin x 1 cos x

36 2004 Nagoya University


A sequence of polynomial fn (x) (n = 0, 1, 2, ) satisfies f0 (x) = 2, f1 (x) = x,
fn (x) = xfn1 (x) fn2 (x), (n = 2, 3, 4, ).
Let xn (n 2) be the maximum real root of the equation fn (x) = 0 (|x| 2)
Evaluate
Z 2
lim n2
fn (x) dx.
n

xn

35 1986 Tokyo Denki University


Z 1

Determine the value of a, b for which


( 1 x ax b)2 dx is minimized.
0

34
Let p be a constant number such that 0 < p < 1. Evaluate
2004
X

k=0

pk (1 p)2004k

xk (1 x)2004k dx

33
Evaluate
Z

ln 2

cos2 hx

dx

(a > 0).
1 2a tanh x + a2

32
Evaluate
Z

ex+e

ex

+ee +ee

dx.

31 1989 Tokyo Institute of Technology


Evaluate
lim

x2 | sin nx| dx.

30 1990 Shibaura Institute of Technology


Z 1
A sequence {an } is defined by an =
x3 (1 x)n dx (n = 1, 2, 3, ).
0

Find the constant number c such that

(n + c)(an an+1 ) =

n=1

1
.
3

29 1982 Kwansei Gakuin University


Let a be a real number. Evaluate
Z

3+a

+a

|x a | sin

28 1992 Tokyo Institute of Technology

101

x
2

dx.

Evaluate
Z

x cos 5x
dx.
cos x

27 1979 Saitama University


Let f (x) = t sin x + (1 t) cos x (0 t 1).
Find the maximum and minimum value of the following P (t):
) (Z
)
(Z
2
2
x
x
e f (x) dx
e f (x) dx .
P (t) =
0

26
Evaluate

ee

ee

dx
.
x ln x ln(ln x) ln{ln(ln x)}

eee

25 1967 Kanazawa University

Let |a| < .


2
Evaluate
Z

dx
.
{sin(a + x) + cos x}2

24 2004 Wakayama Medical University


Z
Find the minimum value of
(x y)2 (sin x)| cos x| dx.
0

23 2004 Fukushima Medical University


Evaluate
lim

a 2
a<
2

(cos x) ln(cos x) dx

0a<

22 2004 Akita Prefectural University


Evaluate
Z

(1 x2 )n dx (n = 0, 1, 2, ).

21 Entrance Examination for Japanese University 2004


Z
ln x
(1) Tokyo University of Science:
dx.
(x + 1)2
Z
5
dx.
(2) Saitama University:
3 sin x +
4 cos x
Z 3
1

dx.
(3) Yokohama City University:
2
1 Z x +1
2
sin3 x
(4) Daido Institute of Technology:
dx.
sin x + cos x
0
Z 3
4
(5) Gunma University:
{(1 + x) sin x + (1 x) cos x} dx.
0

20
Calculate
the following indefinite integrals:
Z
(1) ln(x2 1) dx;
102


.
2

(2)

ex

1
dx;
+1

(ax2 + bx + c)emx dx (abcm 6= 0);


2
Z 
1
tan x +
dx;
(4)
tan x
Z

1 sin x dx.
(5)
(3)

19
Calculate
the following indefinite integrals:
Z
3
(1) tan x dx;
Z
(2) amx+n dx (a > 0, a 6= 1, mn 6= 0);
Z
(3) cos5 x dx;
Z
(4) sin2 x cos3 x dx;
Z
dx
.
(5)
sin x
18
Calculate
the following indefinite integrals.
Z
(1) (sin x + cos x)4 dx;
Z
e2x
(2)
dx;
x
Z e +1
(3) sin4 x dx;
Z
(4) sin 6x cos 2x dx;
Z
x2
p
dx.
(5)
(x + 1)3
17

Calculate
the following indefinite integrals:
Z
dx
;
(1)
x
x
Z e e
(2) eax cos 2x dx (a 6= 0);
Z
(3) (3x 2)2 dx;
Z 4
x + 2x3 + 3x2 + 1
dx;
(4)
(x + 2)5
Z
dx
dx.
(5)
1 cos x
16
Calculate
the following indefinite integrals.
Z
(1) sin(ln x) dx;
Z
x + sin2 x
dx;
(2)
2
x
Z x sin
3
x
(3)
dx;
2
Z x +21
x

dx;
(4)
2x 1
Z
x + cos 2x + 1
(5)
dx.
x cos2 x
103

15
Calculate
the following indefinite integrals:
Z
(x2 1)2
dx;
(1)
x4
Z
e3x

dx;
(2)
ex + 1
Z

(3)

(4)
(5)

sin 2x cos 3x dx;

x ln(x + 1) dx;
x
dx.
(x + 3)2

14
Calculate
the following indefinite integrals:
Z
sin x cos x
(1)
dx;
2
Z 1 + sin x

(2)

(3)

(4)
(5)

x log10 x dx;

x
dx;
2x 1

(x2 + 1) ln x dx;
ex cos x dx.

13
Calculate
the following integrals:
Z
(1) x cos2 x dx;
Z
x1
(2)
dx;
(3x
1)2
Z
x3
(3)
dx;
(2 x2 )4 
Z 
1
1
+
(4)
dx;
2x
Z 4 x

(5)

(ln x)2 dx.

12
Calculate
the following indefinite integrals:
Z
dx
;
(1)
+ cos x
Z 1p
(2) x x2 1 dx;
Z
x
(3) a 2 dx (a > 0, a 6= 1);
Z
sin3 x
dx;
(4)
Z 1 + cos x
(5) e4x sin 2x dx.
11
Calculate
the following indefinite integrals:
Z
6x + 1

(1)
dx;
3x2 + x + 4
Z
ex
dx;
(2)
x
ax
Z e+ e 3
( x + 1)

(3)
dx;
x
104

(4)
(5)

x ln(x2 1) dx;
2(x + 2)
dx.
+ 4x + 1

x2

10
Calculate
the following indefinite integrals:
Z

(1) (2x + 1) x + 2 dx;


Z
1 + cos x
dx;
(2)
Z x + sin x
(3) sin5 x cos3 x dx;
Z
(x 3)2
(4)
dx;
x4
Z
dx
dx.
(5)
tan x
9
Calculate
the following indefinite integrals:
Z
(1) (x2 + 4x 3)2 (x + 2) dx;
Z
ln x
dx;
(2)
x(ln
x + 1)
Z
sin ( log2 x)
(3)
dx;
x
Z
dx
;
(4)
sin
x
cos2 x
Z

(5)
1 3x dx.
8
Calculate
the following indefinite integrals:
Z
(1) x(x2 + 3)2 dx;
Z
(2) ln(x + 2) dx;
Z
(3) x cos x dx;
Z
dx
(4)
;
2
Z (x + 2)
x1
dx.
(5)
2
x 2x + 3
7
Calculate
the following indefinite integrals:
Z

x( x + 1)2 dx;
(1)
Z
(2) (ex + 2ex+1 3ex+2 ) dx;
Z
(3) (sin2 x + cos x) sin x dx;
Z

(4) x 2 x dx;
Z
(5) x ln x dx.
6
Calculate
the following indefinite integrals:
Z
(1) sin x cos3 x dx;
105

1
dx;
+ x) x
Z (1p
2
(3) x x3 + 1 dx;
Z 2x
e 3ex
dx;
(4)
ex
Z
(5) (1 x2 )ex dx.
(2)

5
Calculate
the following indefinite integrals:
Z
(1) (4 5 tan x) cos x dx;
Z
1
p
dx;
(2)
3
(1

3x)2
Z
p
(3) x3 4 x2 dx;
Z


(4) ex sin x +
dx;
4
Z
(5) (3x 4)2 dx.
4
Calculate
the following indefinite integrals:
Z
x

(1)
dx;
5x
Z
2
sin x cos x
dx;
(2)
Z 1 + cos x
(sin x + cos x)2 dx;

(3)

x cos2 x
dx;
2
Z x cos x
(5) (sin x + sin 2x)2 dx.

(4)

3
Calculate
the following indefinite integrals:
Z
(1) sin x sin 2x dx;
Z
e2x
dx;
(2)
x
1
Z e
tan2 x
(3)
dx;
2x
Z cos
ex + ex
dx;
(4)
x
ex
Z e
x
e
(5)
dx.
ex + 1
2
Calculate
the following indefinite integrals:
Z


dx;
(1) cos 2x
3
Z
1
(2)
dx;
2 (3x + 4)
cos
Z

(3) (x 1) 3 x 2 dx;
Z
2
(4) x 3x +1 dx;
Z
1

dx.
(5)
1x
106

1
Calculat
atoarele integrale nedefinite:
Z , i urm
e2x
[1]
dx;
x
2
Z (e + 1)
[2] sin x cos 3x dx;
Z
[3] sin 2x sin 3x dx;
Z
1
dx;
[4]
2
Z 4x 12x + 9
[5] cos4 x dx.
Z
Z
ex
e2x
dx
=
d(ex + 1)
Solut, ie. [1]
x + 1)2
x + 1)2
(e
(e

Z x
Z 
e +11
1
1
x
=
d(ex + 1)
d(e
+
1)
=

(ex + 1)2
ex + 1 (ex + 1)2
1
= ln(ex + 1) + x
+C
e +1
[2] Using sin cos = 21 {sin( + ) + sin( )},


Z
Z
1
1
1
1
cos 4x + cos 2x + C
sin x cos 3x dx =
(sin 4x sin 2x) dx =
2
2
4
2
1
1
=
cos 2x cos 4x + C
4
8
1
[3] Folosind formula sin sin = {cos( ) cos( + )}, avem:
2


Z
Z
1
1
1
(cos x cos 5x) dx =
sin x sin 5x + C
sin 2x sin 3x dx =
2
2
5
1
1
sin x
sin 5x + C
=
2
10
Z
Z
dx
1
dx
=
=
+C
[4]
2
2
4x 12x + 9
(2x 3)
2(2x 3)
2
, avem
[5] Folosind formula cos2 = 1+cos
2
Z
Z
Z2 
1 + cos 2x
4
2
2
cos x dx = (cos x) dx =
dx

Z
Z 2
1
1 + cos 4x
1
2
=
dx
1 + 2 cos 2x +
(1 + 2 cos 2x + cos 2x) dx =
4
4
2

1
1 3
x + sin 2x + sin 4x + C
=
4 2
8
3
1
1
= x + sin 2x +
sin 4x + C
8
4
32

107

Вам также может понравиться